PDA

View Full Version : Got a Real-World Weapon, Armor or Tactics Question? Mk. XXV



Pages : [1] 2 3 4 5 6

Brother Oni
2018-01-16, 02:25 PM
Real World Weapon, Armour and Tactics Thread XXV

This thread is a resource for getting information about real life weapons, armour and tactics. The concept has always been that the information is for RPG players and DMs so they can use it to make their games better, thus it's here rather than in Friendly Banter.

A few rules for this thread:


This thread is for asking questions about how weapons, armour and tactics really work. As such, it's not going to include game rule statistics. If you have such a question, especially if it stems from an answer or question in this thread, feel free to start a new thread and include a link back to here. If you do ask a rule question here, you'll be asked to move it elsewhere, and then we'll be happy to help out with it.

Any weapon or time period is open for questions. Medieval and ancient warfare questions seem to predominate, but since there are many games set in other periods as well, feel free to ask about any weapon. This includes futuristic ones - but be aware that these will be likely assessed according to their real life feasibility. Thus, phasers, for example, will be talked about in real-world science and physics terms rather than the Star Trek canon. If you want to discuss a fictional weapon from a particular source according to the canonical explanation, please start a new thread for it.

Please try to cite your claims if possible. If you know of a citation for a particular piece of information, please include it. However, everyone should be aware that sometimes even the experts don't agree, so it's quite possible to have two conflicting answers to the same question. This isn't a problem; the asker of the question can examine the information and decide which side to go with. The purpose of the thread is to provide as much information as possible. Debates are fine, but be sure to keep it a friendly debate (even if the experts can't!).

No modern real-world political discussion. As the great Carl von Clausevitz once said, "War is merely the continuation of policy by other means," so politics and war are heavily intertwined. However, politics are a big hot-button issue and one banned on these boards, so avoid political analysis if at all possible (this thread is primarily about military hardware). There's more leeway on this for anything prior to about 1800, but be very careful with all of it, and anything past 1900 is surely not open for analysis (These are arbitrary dates but any dates would be, and these are felt to be reasonable).

No graphic descriptions. War is violent, dirty, and horrific, and anyone discussing it should be keenly aware of that. However, on this board graphic descriptions of violence (or sexuality) are not allowed, so please avoid them.


With that done, have at and enjoy yourselves!
Thread I (http://www.giantitp.com/forums/showthread.php?24294-Got-A-Weapon-or-Armor-Question)
Thread III (http://www.giantitp.com/forums/showthread.php?21318-Got-a-Real-World-Weapon-or-Armor-Question-III)
Thread IV (http://www.giantitp.com/forums/showthread.php?18302-Got-a-Real-World-Weapon-or-Armor-Question-Mk-IV)
Thread V (http://www.giantitp.com/forums/showthread.php?80863-Got-a-Real-World-Weapon-or-Armor-Question-Mk-V)
Thread VI (http://www.giantitp.com/forums/showthread.php?124683-Got-a-Real-World-Weapon-or-Armor-Question-Mk-VI)
Thread VII (http://www.giantitp.com/forums/showthread.php?168432-Got-a-Real-World-Weapon-or-Armor-Question-Mk-VII)
Thread VIII (http://www.giantitp.com/forums/showthread.php?192911-Got-a-Real-World-Weapon-or-Armor-Question-Mk-VIII)
Thread IX (http://www.giantitp.com/forums/showthread.php?217159-Got-a-Real-World-Weapon-or-Armor-Question-Mk-IX)
Thread X (http://www.giantitp.com/forums/showthread.php?238042-Got-a-Real-World-Weapons-or-Armour-Question-Mk-X)
Thread XI (http://www.giantitp.com/forums/showthread.php?255453-Got-a-Real-World-Weapons-or-Armour-Question-Mk-XI)
Thread XII (http://www.giantitp.com/forums/showthread.php?282471-Got-a-Real-World-Weapons-or-Armour-Question-Mk-XII)
Thread XIII (http://www.giantitp.com/forums/showthread.php?308462-Got-a-Real-World-Weapons-or-Armour-Question-Mk-XIII)
Thread XIV (http://www.giantitp.com/forums/showthread.php?327994-Got-a-Real-World-Weapon-or-Armor-Question-Mk-XIV)
Thread XV (http://www.giantitp.com/forums/showthread.php?347806-Got-a-Real-World-Weapon-or-Armour-Question-Mk-XV)
Thread XVI (http://www.giantitp.com/forums/showthread.php?371623-Got-a-Real-World-Weapon-Armor-or-Tactics-Question-Mk-XVI)
Thread XVII (http://www.giantitp.com/forums/showthread.php?392804-Got-a-Real-World-Weapon-Armor-or-Tactics-Question-Mk-XVII)
Thread XVIII (http://www.giantitp.com/forums/showthread.php?421723-Got-a-Real-World-Weapon-Armor-or-Tactics-Question-Mk-XVIII)
Thread XIX (http://www.giantitp.com/forums/showthread.php?454083-Got-a-Real-World-Weapon-Armor-or-Tactics-Question-Mk-XIX)
Thread XX (http://www.giantitp.com/forums/showthread.php?480058-Got-a-Real-World-Weapon-Armor-or-Tactics-Question-Mk-XX)
Thread XXI (http://www.giantitp.com/forums/showthread.php?493127-Got-a-Real-World-Weapon-Armor-or-Tactics-Question-Mk-XXI)
Thread XXII (http://www.giantitp.com/forums/showthread.php?503643-Got-a-Real-World-Weapon-Armor-or-Tactics-Question-Mk-XXII)
Thread XXIII (http://www.giantitp.com/forums/showthread.php?518251-Got-a-Real-World-Weapon-Armor-or-Tactics-Question-Mk-XXIII)
Thread XXIV (http://www.giantitp.com/forums/showthread.php?532903-Got-a-Real-World-Weapon-Armor-or-Tactics-Question-Mk-XXIV)

Brother Oni
2018-01-16, 02:36 PM
I think crossbows would be more impractical- they are heavier, the bolt is harder to keep in place when aiming down (or upside-down!), and the methods of loading that are most possible (crannequin and maybe windlass) are fiddly, and could be dangerous to the wielder to use underwing. Especially so on powerful crossbows, where attaching the crannequin wrong can result in lethal injuries. Belt hooks are obviously very difficult when flying, and hand drawing has the same draw backs as using a bow.

They had a similar problem for horseback crossbow use and they found a drop of glue or wax on the bolt held it in place just fine.

A goat's foot lever spanning method would also work just fine for a decent weight crossbow; they would just have to retreat back out of range to reload before having another go, which is basically what mounted crossbowmen did as well. The flyer could tuck the lever into their belt when not being used and for additional security, it's tied to the belt with a bit of string.

That said, the lever is a bit out of the time scale proposed by the initial scenario (they were most popular during the 14th-15th Century).

Goat's foot lever in action (https://www.youtube.com/watch?v=iIkxyjVu9gc).


Seems the consensus ranges from "it's an advantage" to "they'd crush everyone entirely."

An alternate way to limit their dominance would be non-military; Either have them culturally pacifistic or non-aggressive or have them reproduce very slowly.

Both could be tied into their biology; many birds have hollow bones and numerous other weight saving measures intrinsic to their biology, none of which are conducive to good endurance or durability in combat.
Humans are very durable for animals and injuries that just put us out of commission for a while are lethal to other species - a broken leg is a major but recoverable injury; for a horse, it's usually fatal. Tie this into the less durable skeletal structure, then a moderate injury for a human would be life threatening or lethal to them.

Second is energy expenditure - flying is very energy intensive, which is why all birds have a superior lung structure to humans for the purposes of extracting oxygen; even then large birds prefer to coast on thermals. Combat would be incredibly tiring for these flyers, so their endurance would be very limited.
This complexity of biology would also lend itself to longer gestation times, so recovery of combat losses would be more difficult.

Finally there's the heartbeat hypothesis of longevity (https://www.northcoastjournal.com/humboldt/the-heartbeat-hypothesis/Content?oid=3170875); flight again is incredibly energy intensive and assuming a standard human's cardiopulmonary system and to get blood around the body faster, their heart beats faster (rather than dynamically increase the stroke volume) so each individual lifespan is shorter. Normally this wouldn't have an effect on their combat abilities, but it would potentially give a certain impatience to their psychology and culture, making them more likely to fall for tricks and less veteran leaders to rein people in.

Dienekes
2018-01-16, 03:38 PM
So, a bit of an odd question. I know frogmouth helmets were primarily used in tournaments. However, I swear I saw some in the background of a few paintings of battlefields. Unfortunately, I can't for the life of me seem to find the painting I'm remembering.

I was wondering if any of you guys who seem to pull out pictures of painting from seemingly nowhere have any examples of them? Also, how do you organize and search through all of them. My own method of sorting them is obviously not up to snuff.

Knaight
2018-01-16, 06:05 PM
On the flying people - there are some implicit weaknesses that come with that which haven't been mentioned. A major one is food - flying is hard, energy intensive work. They'd need much more food than an ordinary human, which means they wouldn't be able to maintain the population density of regular people. If they're farming, that uses a bigger population fraction. If they're primarily hunting (which is what I'd expect), then they need to be spread even thinner.

On top of that, we're talking about flying humans here. By bird standards human vision is hot garbage, by predatory raptor standards it's somehow even worse. A small group of people could hide from them relatively easily in most terrain, making ground ambushes very much a possibility - which are all the more effective given that normal humans would basically always have a numerical advantage due to the differences in food requirements.

Galloglaich
2018-01-16, 06:54 PM
Ok so here is my take on the whole flying monkey thing - stipulating in an affluent / sophisticated part of Late Medieval Europe. I'm just going to say 'medieval' from here on in for brevity but that is what I mean, plate armor era.

The TL : DR of this in my opinion that of course flying troops would be a major asset, but in terms of direct combat the advantage would be limited.

The gist is this - I think assuming they were aware of the threat and had been for a while, a medieval army would have fairly effective short range air-defense capability. They would also have counter-measures available to protect themselves from longer range attacks (attacks from further up).

I think with a robust enough Air-Defense system, the flying troops would have a hard time doing damage against hardened military forces, and take heavy casualties. If you buy robust air defenses, an analogy would be WW1 aircraft trying to attack WW2 type AAA defenses. They might do some damage but they would get shredded. You can actually see this to some extent in WW2 where the Royal Navy had such a hard time sinking German ships with their somewhat antiquated or poorly designed aircraft (Fairey Swordfish biplanes, and Battle, Barracuda etc.) and with a combination of heavy AAA and smoke screens and so on, some of them were able to survive a ridiculously long time (see Tirpitz).

Flying troops would still be a major advantage in terms of scouting, skirmishing, communications and so on - and that is significant. Could be decisive if it were properly organized. Most of all, since there is no way to pursue them in the sky, and presumably they can find / make bases which are high on mountaintops or cliffs that are very hard for ground based forces to access, the side with the flying troops would have the operational initiative.

This is what I see the medieval army using

Rapid fire, breach-loading pintle-mounted cannon

https://i.pinimg.com/474x/d9/90/78/d9907844720e7d710a0d8d885fc8e211--conquistador-a-ship.jpg

These already existed by the 14th Century. In the last thread I think I posted a couple of videos of them in action. If you watch those you'll notice that with pre-loaded breaches, you can get multiple shots off fairly quickly. longer barrel and higher caliber weapons would be best probably, but you'd want a lot of these on your castles, city walls and towers, ships and war-wagons.

Volley guns
https://i.pinimg.com/originals/5d/ba/68/5dba6879d7f8d0f7d4fbbd703934445d.jpg
https://i.pinimg.com/236x/e6/b2/8e/e6b28ebca311d981cc13bcbaef1df1f6--medieval-weapons-innsbruck.jpg

Volley guns also go back to the 14th Century. They were fairly easy to make. The best for AAA would be those with multiple rows, you can shoot a row, aim and shoot another row, and then shoot another. Volley guns are slow to reload, but you can shoot them one row at a time. And if you have a lot of them and use them wisely, you can maintain firepower.

Bigger cannon
https://i.pinimg.com/564x/74/12/ef/7412ef915b4f9f4c3f0aea93b13d4914--cannon-drawings-of.jpg
Cannon cold be used to shoot 'shot' and possibly exploding ordinance too.

Rockets and sky-rockets
https://i.pinimg.com/736x/71/a8/60/71a8608a5dd180ea6a7dd48a42c67e45--th-century-archery.jpg

https://i.pinimg.com/736x/b6/89/f3/b689f3f49d2325458ac5548ef07a545c--medieval-weapons-heidelberg.jpg

https://upload.wikimedia.org/wikipedia/commons/thumb/5/58/Conrad_Kyeser%27s_Bellifortis_c_1405_fig_1.jpg/457px-Conrad_Kyeser%27s_Bellifortis_c_1405_fig_1.jpg

Rockets, which were plentiful in the late medieval period, could be used to lift nets, strong but thin wires, ropes or cables up in the air with hooks on them, and so on. To illuminate night-attacks, and perhaps most of all, fireworks - sky rockets, that go up and blow up in great big sprays of sparks - could be useful against flying troops. Especially if you included say some fish-hooks, lengths of wire etc. in the mix.

Other types of pyrotechnic weapons would also be helpful I think. Large formations that spray sparks and so on.

IED's
https://i.pinimg.com/originals/1a/c5/5a/1ac55a24054c5384732f3578c1baf34e.jpg

Bombs with shrapnel and submunitions could be placed in ambush positions, for example in a canyon the flying troops used as part of a repeated route, to kill and wound them.

Grenade Launchers
https://i.pinimg.com/originals/8a/f7/00/8af700ff34491012179249e7e2d4e484.jpg

Grenade launchers, or 'hand mortars' as the ones from this era were typically referred to, could be used to shoot exploding shells or just giant firecrackers up in the air. Presumably a huge boom and flash of sparks could disorient a flyer or even cause them to crash; shrapnel could make it more deadly. If you were with a small group of people and came under flying monkey attack while you didn't have a war wagon or a smoke system handy, one of these things might at least buy you some time. Apparently they were often used to shoot fireworks up in the sky to announce the arrival of a VIP into a town, for example, so this is already "ready to go"


Smoke
Many pyrotechnic devices, as well as a wide variety of much cheaper fuel sources, can make thick opaque smoke. This was used by ships in WW1 and WW2 to protect them from aircraft for example. A thick enough pall of smoke over a target can make aiming even more generalized attacks much harder.

War Wagons
https://forum.crossout.net/uploads/monthly_2017_08/guncart.jpg.f28dc2c1686032755cf078ca7ead9a5e.jpg

http://media-channel.nationalgeographic.com/media/uploads/photos/content/video/2013/01/23/ngc-9512_239.jpg

War wagons as mentioned here many times were all over the place, obviously you would need to include a roof and protect horses, but some of them were already doing that by the 15th Century.

Crossbows
Crossbows, especially larger 'wall crossbows' or siege crossbows, seem to have had the ability to shoot individual targets at a fairly long range. They were using them to hunt birds on horseback and one of the main ways to train was to shoot at a wooden 'parrot' on a 40' pole (also sometimes from horseback), so the idea of shooting targets up in the air was already well established.



This is what I see the flying monkeys using
https://i.pinimg.com/236x/ec/2e/fa/ec2efac2fdf0c4d0dc8bd160d7abb3f1--wicked-witch-brick-road.jpg

Javelins
I agree with cistenes, darts and javelins, especially heavy javelins like soliferrum, would be devastating in a flying attack. The downside is you would have to get pretty close to hit a target which makes you vulnerable. You also, presumably, couldn't carry a lot of them. Historically human peltasts etc. carried three. So you have to have a stash somewhere.

Bomblet darts, rocks etc.
Solid iron darts, tiny ones, as well as things like caltrops etc., could be effective area-attack weapons. Not sure of the proposed ceiling of these flying troops is but if you can fly high enough (thousands of feet) even just like marbles and pebbles can kill - but you do still have to hit your target. Advantage of attacking from up high is you are going to be safe from attack. The only defense would be obscuring the target with smoke or hiding some other way.

Pyrotechnic bombs, grenades, and petroleum based fire bombs
This is one of the most devastating ways that flying attackers could get their targets, but it's also quite risky. Towns would have to be fireproofed. This was already underway, but not nearly enough to deal with flaming naptha etc. you don't

Poison and disease agents
Dropping poop or contaminated substances into a castle or a town would be quite effective. Done at night it could be particularly nasty. Countermeasure would be alert sentries and the use of flares and pyrotechnics to illuminate the sky when under attack, but most of all covering food and water sources. This type of attack wasn't new though as they used to throw stuff over walls with trebuchets and so on, so presumably there were known procedures to deal with the problem already.



Oh by the way, bonus - fairly early example of firearms on horseback. Mariano di Jacopo aka 'Taccola', Siena Italy, before 1453. Presumable Liber tertius de ingeneis ac edifitiis non usitatis.Bayerische Staatsbibliothek, München, Clm 197

https://i.pinimg.com/736x/17/7f/64/177f6427ea8832c0ac869e202f682d13--medieval-armor-th-century.jpg

Kiero
2018-01-16, 07:01 PM
One other disadvantage javelins have for a flying unit, compared to a mounted one, is that there's pretty much no chance of getting your missiles back. Unless you're willing to risk coming to ground level (negating all the advantages of flight and making yourself vulnerable), they're gone once you've used them. Mounted javelineers, on the other hand, can often count on retrieving a certain proportion of their missiles.

rs2excelsior
2018-01-16, 08:49 PM
They had a similar problem for horseback crossbow use and they found a drop of glue or wax on the bolt held it in place just fine.

I was under the impression that practice was mostly done when carrying a crossbow loaded on the march, is that incorrect? Even so, I think dropping hot wax on a crossbow bolt while in the air and possibly being shot at would be tricky to say the least. Still, for the first shot it should help immensely, or if you take a shot and fly out of range to reload. That takes you out of combat for a while though.


An alternate way to limit their dominance would be non-military; Either have them culturally pacifistic or non-aggressive or have them reproduce very slowly.

Both could be tied into their biology; many birds have hollow bones and numerous other weight saving measures intrinsic to their biology, none of which are conducive to good endurance or durability in combat.
Humans are very durable for animals and injuries that just put us out of commission for a while are lethal to other species - a broken leg is a major but recoverable injury; for a horse, it's usually fatal. Tie this into the less durable skeletal structure, then a moderate injury for a human would be life threatening or lethal to them.

Second is energy expenditure - flying is very energy intensive, which is why all birds have a superior lung structure to humans for the purposes of extracting oxygen; even then large birds prefer to coast on thermals. Combat would be incredibly tiring for these flyers, so their endurance would be very limited.
This complexity of biology would also lend itself to longer gestation times, so recovery of combat losses would be more difficult.

Finally there's the heartbeat hypothesis of longevity (https://www.northcoastjournal.com/humboldt/the-heartbeat-hypothesis/Content?oid=3170875); flight again is incredibly energy intensive and assuming a standard human's cardiopulmonary system and to get blood around the body faster, their heart beats faster (rather than dynamically increase the stroke volume) so each individual lifespan is shorter. Normally this wouldn't have an effect on their combat abilities, but it would potentially give a certain impatience to their psychology and culture, making them more likely to fall for tricks and less veteran leaders to rein people in.


On the flying people - there are some implicit weaknesses that come with that which haven't been mentioned. A major one is food - flying is hard, energy intensive work. They'd need much more food than an ordinary human, which means they wouldn't be able to maintain the population density of regular people. If they're farming, that uses a bigger population fraction. If they're primarily hunting (which is what I'd expect), then they need to be spread even thinner.

On top of that, we're talking about flying humans here. By bird standards human vision is hot garbage, by predatory raptor standards it's somehow even worse. A small group of people could hide from them relatively easily in most terrain, making ground ambushes very much a possibility - which are all the more effective given that normal humans would basically always have a numerical advantage due to the differences in food requirements.

Also good points here, especially food requirements and birth rate/gestation time. Something I'll take into account as I develop the setting. Unfortunately I can't make them pacifist, the setting hinges around aforementioned flying people being a bunch of imperialistic jerks to everyone around :smallbiggrin: (Their relationship to the human nations is akin to the dynamic between ancient Greece and Persia, with the flyers taking the role of the Persians--a much larger and more centralized empire trying to subjugate smaller states that band together for mutual defense, and occasionally meddling politically in those small states where direct attacks don't work). That said, they've had their empire for a while, so it's entirely possible that their warrior elite who won the empire in the first place have morphed into a more administrative class which is happy to let the "lesser" subject races (who don't fly) do most of the unpleasant things like fighting and dying.


Ok so here is my take on the whole flying monkey thing - stipulating in an affluent / sophisticated part of Late Medieval Europe. I'm just going to say 'medieval' from here on in for brevity but that is what I mean, plate armor era.

The TL : DR of this in my opinion that of course flying troops would be a major asset, but in terms of direct combat the advantage would be limited.

The gist is this - I think assuming they were aware of the threat and had been for a while, a medieval army would have fairly effective short range air-defense capability. They would also have counter-measures available to protect themselves from longer range attacks (attacks from further up).

-Lots of awesome stuff-


This is the kind of post I was hoping to run across :smallbiggrin: I tend to agree, given that in-setting these people have been antagonists for a long time, they would have adapted means to fight. Still, though, I'm surprised (again) at the degree of sophistication that existed in real life. I mean, grenade launchers... Breech-loading guns, volley guns, combinations of the previous two, and rockets would probably be rather more prevalent here than in the actual medieval era, if I had to guess. Those seem to be the most promising from my perspective. Especially rockets/fireworks to illuminate and disorient rather than directly incapacitate, I hadn't thought of that.

I am a bit skeptical, though, of how effective explosives would be, if only because there wasn't much of a means of controlling when the shell went off, save for varying the length of the fuse, which wasn't that precise to begin with and subject to different shells coming from different manufacturers having different fuse lengths. Against a fast-moving target, I don't know how well exploding shell would do without some sort of proximity detonation. They'd be devastating if you managed to land a shot close enough, however.

Speaking of explosives...

Bigger cannon
https://i.pinimg.com/564x/74/12/ef/7412ef915b4f9f4c3f0aea93b13d4914--cannon-drawings-of.jpg
Cannon cold be used to shoot 'shot' and possibly exploding ordinance too.

Exploding shell, as far as I know, didn't really come into use as cannon ammunition into the early 1800s, and wasn't in widespread use until around the American Civil War. Most early grenades that I'm familiar with from the first part of the 1700s were clay pots rather than metal cased--which works well enough for throwing, but I wouldn't want to try to fire one out of a cannon. Was exploding cannon shot practical with this level of technology?

And, while we're on the topic: a couple of grenade designs which saw experimental use in the American Civil War, using simple impact fuses. They wouldn't fit into the medieval era, but could possibly be adapted to use a striker other than a percussion cap:

Ketchum Grenade:
https://qmcivilwar.files.wordpress.com/2012/01/img_1320.jpg

https://hmscivilwarproject.wikispaces.com/file/view/Ketchum_Grenade.jpg/74486673/500x178/Ketchum_Grenade.jpg

Haynes Excelsior Grenade:
https://userscontent2.emaze.com/images/89f835bd-46bf-4669-a1ad-fa7e5d6905f9/52ec1b74-4fbd-4f16-9621-58153d2b666f.jpg
Couldn't find a good schematic for this one. Basically the smaller sphere would be filled with black powder, percussion caps placed on all of the intrusions, and then it'd be locked inside the larger sphere. On impact the small inner sphere would strike the large outer sphere, set off one of the caps, and ignite the charge. It had the unfortunate habit of going off when thrown, however...

Incanur
2018-01-16, 09:05 PM
I think y'all are underestimating how hard it is to hit something smaller than a formation from significantly above. Sure, these flying creatures could fly completely out of arrow range, but then they'd have issues hitting their targets too. Unless they're very nimble, to effectively hit individuals and small groups they'd have to risk getting hit in return. They'd have a big advantage, of course, but they wouldn't be completely immune to shots from the ground. And I assume they'd be unarmored or lightly armored, given how flight works.

Knaight
2018-01-16, 09:28 PM
I think y'all are underestimating how hard it is to hit something smaller than a formation from significantly above. Sure, these flying creatures could fly completely out of arrow range, but then they'd have issues hitting their targets too. Unless they're very nimble, to effectively hit individuals and small groups they'd have to risk getting hit in return. They'd have a big advantage, of course, but they wouldn't be completely immune to shots from the ground. And I assume they'd be unarmored or lightly armored, given how flight works.

The whole matter of formations also seems to assume pitched battles, instead of sieges and raiding. The fliers are unlikely to use much in the way of traditional sieges for a few reasons, but raiding? It's what they'd do best, and it scales up. Cities are easier to hit than formations after all.

Mr Beer
2018-01-16, 09:39 PM
Best attacks I think they can use, barring impact fused explosives, would be as follows:

- Small iron darts.

Used from great heights, at which point they will act like a modern 50 calibre bullet and easily kill an armoured man. Only useful against formations, precise aiming is impossible. Flyer will be invulnerable at this height. Flyer can carry many darts (0.5 oz each).

- Javelins

Used as a more precision killer, perhaps from a dive, at lower heights. With added dive velocity, will likely defeat most armours. Flyer is vulnerable to counter-attack. Flyer can carry only a few javelins.

- Incendiaries

Used to start fires (duh) in fortifications and conurbations. Can be used from a great height. Flyer will be invulnerable at this height. Flyer cannot carry many such devices, perhaps only one.

Due to the limitations of what they can carry, to be decisive on the battlefield as a bombing wing, I think they need to be heavily supplied, from behind a friendly army's lines or from fortified position. Essentially fly up, deliver munitions, fly back, re-supply. Even then, it may be time-consuming to fly up to 1000 feet or so and back down again. How many missions can they fly, how rapidly and how many formations can they break doing so?

As said in the previous thread by many posters, it's the other things that they can do with intelligence gathering, command and control and manouverability that makes them so valuable.

Galloglaich
2018-01-16, 10:24 PM
This is the kind of post I was hoping to run across :smallbiggrin: I tend to agree, given that in-setting these people have been antagonists for a long time, they would have adapted means to fight. Still, though, I'm surprised (again) at the degree of sophistication that existed in real life. I mean, grenade launchers... Breech-loading guns, volley guns, combinations of the previous two, and rockets would probably be rather more prevalent here than in the actual medieval era, if I had to guess.


Thanks!

I can assure you that breach-loading cannon (usually light to medium caliber) were quite common from the 14th Century onward. Dozens have survived from the 15th Century alone. Same for organ guns. There was one single battle where the city of Ghent deployed probably at least 300 volley guns in 1382. Don't ask me to pronounce the name...

https://en.wikipedia.org/wiki/Battle_of_Beverhoutsveld

I'm not sure how prevalent rockets were on the battlefield but I do know they were mentioned as early as the (late) 13th Century, where they were apparently used by the Moors in a battle in what is now Spain. I also know by the 14th Century towns were routinely putting on big fireworks shows on major Feast days and during Royal coronations and so forth.

I did specify of course, that I'm referring to the more sophisticated and wealthy parts of Europe from in the Late medieval period, say roughly 1300-1520. I'm not talking about Albania or Finland here (no offense to either country). Or England for that matter. I know they had gear like this but I don't know how much. In Central Europe, Italy, Flanders, the usual places ... it was commonplace. You see these weapons dozens of times in the Swiss Chronicles for example.



Those seem to be the most promising from my perspective. Especially rockets/fireworks to illuminate and disorient rather than directly incapacitate, I hadn't thought of that.

Another thing i forgot to mention are 'war kites'. These things show up in many war-books like Bellifortis (1410) which is where this one is from:

https://i.pinimg.com/originals/be/50/5d/be505d78e5b7b6fdc7b42d03f4bd8385.jpg

and this one

https://c1.staticflickr.com/5/4026/4404793554_ab03ce23b4_b.jpg

In theory you could use war-kites a bit like barrage baloons, maybe embed the kite strings with hooks or coat them with glass or something, or even put hooks or barbs on the kite itself.



I am a bit skeptical, though, of how effective explosives would be, if only because there wasn't much of a means of controlling when the shell went off, save for varying the length of the fuse, which wasn't that precise to begin with and subject to different shells coming from different manufacturers having different fuse lengths. Against a fast-moving target, I don't know how well exploding shell would do without some sort of proximity detonation. They'd be devastating if you managed to land a shot close enough, however.

Keep in mind, they had the serpentine - so in theory you can have a slow-match burning, and a long string with somebody waiting. When they see a formation of flyers going by, you pull the string and boom. IED.

This is a serpentine from a war manual derivative of belifortis from ~ 1420

https://i.pinimg.com/170x/8f/68/ee/8f68eef8a3c912cf4a5be4dcef9e9c43--vienna-locks.jpg
Speaking of explosives...



Exploding shell, as far as I know, didn't really come into use as cannon ammunition into the early 1800s, and wasn't in widespread use until around the American Civil War. Most early grenades that I'm familiar

Well they did have the hand mortars I mentioned, and those had exploding shells, usually made of glass from what i gather. That is what made the military version of these so dangerous to use - if you had too much force it could blow up, if you had a misfire your grenade was still sputtering there about to blow up. Risky.

As this article notes, hand-mortars while rare, first appeared in a war-manual in 1472. Their original use seems to have mainly been as a big noisemaker, though the grenades they used also appear in the 1472 manual. They became more commonplace as weapons (as opposed to noisemakers) in the 1580's when fairly reliable wheellocks were available.

https://www.thevintagenews.com/2016/10/29/hand-mortars-an-early-grenade-launchers-used-from-the-1500s-through-the-early-1800s/



with from the first part of the 1700s were clay pots rather than metal cased--which works well enough for throwing, but I wouldn't want to try to fire one out of a cannon. Was exploding cannon shot practical with this level of technology?

It apparently existed though I would guess was quite risky. The main reason being the fuse. However, presumably under pressure from flying enemy troops the technology would have developed more rapidly, as the basic elements were all there.



And, while we're on the topic: a couple of grenade designs which saw experimental use in the American Civil War, using simple impact fuses. They wouldn't fit into the medieval era, but could possibly be adapted to use a striker other than a percussion cap:

Ketchum Grenade:
https://qmcivilwar.files.wordpress.com/2012/01/img_1320.jpg

https://hmscivilwarproject.wikispaces.com/file/view/Ketchum_Grenade.jpg/74486673/500x178/Ketchum_Grenade.jpg

Haynes Excelsior Grenade:
https://userscontent2.emaze.com/images/89f835bd-46bf-4669-a1ad-fa7e5d6905f9/52ec1b74-4fbd-4f16-9621-58153d2b666f.jpg
Couldn't find a good schematic for this one. Basically the smaller sphere would be filled with black powder, percussion caps placed on all of the intrusions, and then it'd be locked inside the larger sphere. On impact the small inner sphere would strike the large outer sphere, set off one of the caps, and ignite the charge. It had the unfortunate habit of going off when thrown, however...

Wow... see it's always more going on behind the scenes than people realize.

Actually thinking about it, I believe there is a late 14th Century Chinese military manual which has a series of pressure detonators (using techniques to generate sparks) for mines. It also has grenades and lots of other other crazy stuff.

https://en.wikipedia.org/wiki/Huolongjing

This is one of the two mines

https://upload.wikimedia.org/wikipedia/commons/f/f8/Land_mine_HLJ.jpg

And if that aint amazing enough, this is a rocket propelled bird

https://upload.wikimedia.org/wikipedia/commons/thumb/c/c7/%27Flying_Crow_With_Magic_Fire%27%2C_a_winged_rock et_bomb.jpg/369px-%27Flying_Crow_With_Magic_Fire%27%2C_a_winged_rock et_bomb.jpg

Not sure if they had this in Europe or not though.

Still, it's hard to beat the historical reality - the fantasy tropes rarely even come close.

G

Galloglaich
2018-01-16, 10:30 PM
I think y'all are underestimating how hard it is to hit something smaller than a formation from significantly above. Sure, these flying creatures could fly completely out of arrow range, but then they'd have issues hitting their targets too. Unless they're very nimble, to effectively hit individuals and small groups they'd have to risk getting hit in return. They'd have a big advantage, of course, but they wouldn't be completely immune to shots from the ground. And I assume they'd be unarmored or lightly armored, given how flight works.

I agree - I thought this was implicit in my first post but it was a bit too long already to get into it.

If they get close, swooping in Stuka style, they can kill probably even armored knights with a heavy javelin. But then they would be at serious risk of being killed by cannon, volley guns etc. or even if you presume no gunpowder, crossbows and bows.

if they attack from a safe distance, say 1000 meters, it gets a lot harder to hit. Much harder still if the target obscures itself with smoke like a WW2 pocket battleship hiding from the Fleet Air Arm.

So it's a trade off, I would assume they can't wear armor while flying. That's why I said the main advantage would be scouting and C3I and so on (which I think people would tend to understimate as an advantage, but it would be a big one) plus the ability to do a lot of amazing special ops and wipe out stragglers and so forth.

It's like light cavalry in the air, really. Properly organized it could be a bit like the Mongol Hordes. But the Mongol Hordes weren't as effective in the 15th Century as they had been in the 13th.

G

No brains
2018-01-16, 10:54 PM
Just to check, are we certain that the bones of birds are less durable than those of other animals? I've heard that the honeycombed bones actually make birds' bones stronger, which makes sense to me. If a peregrine falcon were truly delicate, how could its body survive its hunting stoops? I've also heard falconers describe that some birds could crush human hands in their talons without breaking their own bones. Do we have good sources on either data point?

Mr Beer
2018-01-16, 11:15 PM
It seems to me that an impact fuse would be easier to develop for a dropped bomb. There is no sudden acceleration when it's released and there is a massive deceleration when it impacts after a fall of dozens or even hundreds of metres. Mediaeval tech could probably develop something with flint & steel sparking 'primers' that would work.

rs2excelsior
2018-01-16, 11:18 PM
It seems to me that an impact fuse would be easier to develop for a dropped bomb. There is no sudden acceleration when it's released and there is a massive deceleration when it impacts after a fall of dozens or even hundreds of metres. Mediaeval tech could probably develop something with flint & steel sparking 'primers' that would work.

A slow match isolated from a charge of some kind of flammable liquid in a breakable container would probably work, so long as you can keep the match lit. A molotov cocktail, basically.

rrgg
2018-01-17, 12:06 AM
The chinese were using explosive iron bombs dating all the way back to the 12th century or so.

This illustration from wikipedia is of the "flying-cloud thunderclap-erupter" A cannon designed to shoot multiple small "thunderclap bombs" at once.

https://en.wikipedia.org/wiki/File:Ming_Dynasty_eruptor_proto-cannon.jpg

Carl
2018-01-17, 05:21 AM
Ok finally decided to look back into the thread, been sort of lurking the forums for a few months for various reasons and just not kept up :(. I only got to see part of this discussion in my skim so if i missed too many detail pipe up and let me know.

First and foremost, arows won't cut it for defenders. Even in an airless environment a 70m/s arrow, (which as i recall is on the fairly fast side), will only got up 250m or about 830ft. Time of flight to max altitude would be just over 7 seconds. Account for slant angle and drag i'd be shocked if you got 400 feet. Assuming an arrow fired up at an angle of 30 degrees from vertical time of fight would be 4 seconds. But in reality effective range might be half that against someone unarmoured.

This is the view from 400ft up:

https://i.ytimg.com/vi/oO3Jkg0Jspw/maxresdefault.jpg

People may be quite small but hitting massed formations or general area's on buildings is quite doable. However being that low mostly rules out dropped things. Even in a vacuum best velocity would be 48m/s which is just over 1kj per kg or mass or 100j's per 100 grams. Thats pretty heavy. Metal darts may be compact and aerodynamic but they won't be light. A bow might well make more sense as the arrows could be lighter and use the fall to maintain energy all the way down.

For fighting back, if you start bringing in crossbows, or especially gunpowder the difficulty starts to change, a crossbow can potentially get a round much higher with effective energy. And unlike IRL i suspect muzzle loading rifled guns would be very popular. I also suspect more aerodynamic bullet shapes would become popular, necessity is the mother of invention after all. All of these would be quite effective with short flight times and consequently much easier aiming, they'd still get some protection from being flyers and the sheer mobility would be incredible but it may not be so hard to hit them as some think.


The biggest issues i have TBH is that flying humanoids alongside humans only make sense if the two have been separated for some reason until now, otherwise one would have out competed the other early on. Assuming that we come onto the biological factors. Personally whilst i'm not willing to declare flying humanoids impossible without magic i am inclined to feel that they would be very unlikely to evolve naturally and their energy usage in flight would be phenomenal, a normal human mass flyer using the more efficient bird figures of energy per kg of mass would consume a marathons worth of energy every half hour of sustained flight, takeoff would be even more energetic. Quite aside from the question of how they'd get enough oxygen and store enough calories this would have severe implications for their dietary side which is probably going to be the hardest part. They'd not only have to get the usual vitamins and minerals but their caloric intake which would be enormous. To try and put it into perspective if they where to get the bulk of their caloric intake from modern day soft drinks they'd have to drink 15 litres for every hour of flight. really only something like honey or sugar water would do as thats got 15 and 10 times the energy density by weight respectively, (so honey would be best), but how the hell they'd produce it in the several kg's per day per head amounts required i have no idea.

Even accounting for all that i'd be shocked if they could fly for more than a couple of hours with any kind of extra load without refueling. Thats a serious constraint all of it's own.

Clistenes
2018-01-17, 05:29 AM
Artillery would be hard to move around the field, while the flyers could fly around the land-bound army and pick the weakest point to attack...

In order to be agile enough the land-bound army would need something like arquebusses, muskets or handgonnes, and shoot potshot. If they tried to rely on cannons, culverins, rockets, mortars... etc., the flyers would just attack before they were deployed, or wait for the land bound army to be in the move..

As for the flyers, small darts wouldn't do the trick; they would stop accelerating after achieving terminal velocity, and that wouldn't be enough to pierce any armor; you would need heavy darts for that...

The groundlings could reduce the effectiveness of heavy metal darts using open formations, that way the flyers would have to come within gun range to hit their targets... the problem is, the flyers could still concentrate 100% of their fire against a small portion of the groundling army, then move against the opposite corner unimpeded... The flyers would still have tactical advantage....

And the flyers woud have access to a devastating weapon: Put a fragmentation grenade inside a ceramic jar filled with oil, nafta or alcohol, cork them, add a wicker, and throw then from any height.... not even open formations would protect against those...

Brother Oni
2018-01-17, 07:41 AM
I was under the impression that practice was mostly done when carrying a crossbow loaded on the march, is that incorrect? Even so, I think dropping hot wax on a crossbow bolt while in the air and possibly being shot at would be tricky to say the least. Still, for the first shot it should help immensely, or if you take a shot and fly out of range to reload. That takes you out of combat for a while though.

You could pre-glue your bolts before combat and load them into a specialised quiver which holds them in place in the ideal orientation and spacing. Alternately, they could use a Zhuge Nu (Chu ku nu) magazine fed repeating crossbow, relying on their altitude and/or poison to give it lethality.


Against a fast-moving target, I don't know how well exploding shell would do without some sort of proximity detonation. They'd be devastating if you managed to land a shot close enough, however.

Assuming less durability, then the lethal range of the ground defenders' explosives are increased especially if they're shrapnel based.


Just to check, are we certain that the bones of birds are less durable than those of other animals? I've heard that the honeycombed bones actually make birds' bones stronger, which makes sense to me. If a peregrine falcon were truly delicate, how could its body survive its hunting stoops? I've also heard falconers describe that some birds could crush human hands in their talons without breaking their own bones. Do we have good sources on either data point?

Doing some digging on the subject, it's a little more complicated than I originally thought: Bone density and the lightweight skeletons of birds, Dumont, 17 Mar 10 (http://rspb.royalsocietypublishing.org/content/277/1691/2193).

Doing a direct comparison between the same functional bone between a bird and a mammal, the bird bone is always lighter, but taken overall, the bird's skeleton is of a similar weight to an equivalent sized mammal. This is because bird skeletons are both larger and the bones have been adapted to be more rigid and denser, thus giving it more strength for the equivalent amount of weight.

That said, they suffer for that rigidity with brittleness, so their bones are less durable and any damage causing a break is more likely to be catastrophic. This is not including catastrophic failure while in flight - a human suffering a broken leg doesn't have as far to fall as flying human breaking a wing while in flight!

Falcons have developed to withstand that particular type of impact in that particular body part - I seriously doubt that a falcon could take the equivalent force from an arrow or a punch to their body (not that I'm advocating falcon punching as an empirical test).

Galloglaich
2018-01-17, 11:46 AM
Artillery would be hard to move around the field, while the flyers could fly around the land-bound army and pick the weakest point to attack...

In order to be agile enough the land-bound army would need something like arquebusses, muskets or handgonnes, and shoot potshot. If they tried to rely on cannons, culverins, rockets, mortars... etc., the flyers would just attack before they were deployed, or wait for the land bound army to be in the move..

As I mentioned previously, the traits you are accurately describing, the ability to concentrate where the enemy is weak and attack by surprise, attack with harassing weapons from outside of range and so on - are all also attributable to good light cavalry, particularly cavalry archers. If you look at the real world one of the ways they adapted to this threat was war wagons.

They were used effectively by Czech mercenaries against the Golden Horde and Ottomans, by the Hungarian Black Army against the Turks (again mostly Czech mercenaries), by the Cossacks against the Mongols and Ottomans, and by the Russians. The Ottomans themselves adopted them.

Obviously it depends on the terrain, but war-wagons serve many purposes - first and perhaps foremost they give you an easy way to carry around your light swivel guns, volley-guns, rockets, and so on. And extra projectiles for crossbows, as well as gunpowder and so on.

https://upload.wikimedia.org/wikipedia/commons/thumb/7/70/Hausbuch_Wolfegg_52v_Kampfwagen.jpg/800px-Hausbuch_Wolfegg_52v_Kampfwagen.jpg

http://i41.tinypic.com/bg3cx.jpg

War wagons provide a pretty quick rallying point for both cavalry and infantry. i would imagine in an era of winged attackers wooden walls made to unfold and extend horizontally instead of vertically could help provide cover and concealment.

https://qph.ec.quoracdn.net/main-qimg-8669b47b4e05a9ec3c9ba879c29ea88c-c

In addition, you could have some means to make large quantities of smoke on your wagon. Smoke would be a good defense. And even maybe a place to anchor 'war-kites' :) no idea how effective that could be but I would love to see it.

Hand mortars (i.e. medieval grenade launchers) could be and definitely were carried on horseback, pre-loaded and ready to go. Maybe too dicey for the actual grenades (unless in the hands of a very skilled operator) but the giant firecracker mode was in use in the 15th Century and could be pretty effective too I bet at disrupting a flying attack and maybe making them crash. Guns too though that is a little dicey - crossbows would be more common in the late medieval (quite ubiquitous).



As for the flyers, small darts wouldn't do the trick; they would stop accelerating after achieving terminal velocity, and that wouldn't be enough to pierce any armor; you would need heavy darts for that...

But good for harassing fire - equivalent to flight arrows. I bet they would bother horses. You could also drop caltrops.



The groundlings could reduce the effectiveness of heavy metal darts using open formations, that way the flyers would have to come within gun range to hit their targets... the problem is, the flyers could still concentrate 100% of their fire against a small portion of the groundling army, then move against the opposite corner unimpeded... The flyers would still have tactical advantage....

But the 'groundlings' could rally to a wagonberg, which can be put together in minutes.



And the flyers woud have access to a devastating weapon: Put a fragmentation grenade inside a ceramic jar filled with oil, nafta or alcohol, cork them, add a wicker, and throw then from any height.... not even open formations would protect against those...

Even just a basic 'naptha coctail' would be pretty dangerous - I think this would indeed be one of the flyers most dangerous weapons. But this would also make them vulnerable to fireworks.

This almost sounds like it's ready to be a pretty good novel or video game. Certainly a good RPG setting.

G

Haighus
2018-01-17, 12:35 PM
Just to check, are we certain that the bones of birds are less durable than those of other animals? I've heard that the honeycombed bones actually make birds' bones stronger, which makes sense to me. If a peregrine falcon were truly delicate, how could its body survive its hunting stoops? I've also heard falconers describe that some birds could crush human hands in their talons without breaking their own bones. Do we have good sources on either data point?

Some points on bones.

The first thing to consider is that bones are not rigid rods of dead rock when they are still in a living organism. Dead bone is a considerably different material in its properties from living bone. Living bone is a very dynamic type of tissue that is constantly being removed and replaced within the body. This is important because bones (and tendons, which are related) are able to adapt to different stresses and situations reasonably quickly, and become very specialised in their functions. Because of this, bones change their internal structure (which is akin to a honeycomb in most bone) to best support the stresses they experience. In most long bones (the type of bone in fingers and talons for example) the stresses are along the length of the bone from muscle contraction and lifting loads/bodyweight. This means the internal structure of the bone is built for withstanding stresses along the bone, and it takes an enormous amount of force to damage a long bone by applying force along its length. They are not adapted for force from the side, so they are easier to snap or crush from this direction. This is why the talons of a large bird of prey may be able to crush a human finger without suffering injury themselves- they are exerting stresses in the correct way for their bones, but the finge is experiencing stresses it is not used to.

In terms of the strength of bone, as a general rule of thumb, denser bone is stronger. This is only true for organised bone, so some diseases that cause very dense bone can still result in weakened bone (such as some bony metastases in cancer). However, denser bone is obviously heavier, so living creatures have to make sacrifices. Bone density also varies based on the stresses upon the bone- astronauts lose a lot of bone density due less weight bearing when in orbit, and this is one of the most serious health effects of low gravity environments.

Mike_G
2018-01-17, 02:23 PM
Ok finally decided to look back into the thread, been sort of lurking the forums for a few months for various reasons and just not kept up :(. I only got to see part of this discussion in my skim so if i missed too many detail pipe up and let me know.

First and foremost, arows won't cut it for defenders. Even in an airless environment a 70m/s arrow, (which as i recall is on the fairly fast side), will only got up 250m or about 830ft. Time of flight to max altitude would be just over 7 seconds. Account for slant angle and drag i'd be shocked if you got 400 feet. Assuming an arrow fired up at an angle of 30 degrees from vertical time of fight would be 4 seconds. But in reality effective range might be half that against someone unarmoured.

.

So, if arrows have to obey the laws of physics, so do your flyers.

Human sized flyers would need to be either very very light, or have very very large wings, or be awkward in flight, flying like turkeys, not falcons.

So, a slow, ponderous dive bomber with a huge wingspan would be easy meat for foot archers or gunners.

Now, they still would be very useful for scouting and behind the lines raiding and high altitude bombing of soft targets

I just think the Stuka Men idea only works if you handwave the crap out of flight physics. If you handwave a 100 kg man who can fly like a hawk and shoot a bow while flapping his wings but insist that a longbow arrow can't rise high enough to hit him without making Newton cry, yeah then the flyers win.

Clistenes
2018-01-17, 02:57 PM
As I mentioned previously, the traits you are accurately describing, the ability to concentrate where the enemy is weak and attack by surprise, attack with harassing weapons from outside of range and so on - are all also attributable to good light cavalry, particularly cavalry archers. If you look at the real world one of the ways they adapted to this threat was war wagons.

They were used effectively by Czech mercenaries against the Golden Horde and Ottomans, by the Hungarian Black Army against the Turks (again mostly Czech mercenaries), by the Cossacks against the Mongols and Ottomans, and by the Russians. The Ottomans themselves adopted them.

Obviously it depends on the terrain, but war-wagons serve many purposes - first and perhaps foremost they give you an easy way to carry around your light swivel guns, volley-guns, rockets, and so on. And extra projectiles for crossbows, as well as gunpowder and so on.

https://upload.wikimedia.org/wikipedia/commons/thumb/7/70/Hausbuch_Wolfegg_52v_Kampfwagen.jpg/800px-Hausbuch_Wolfegg_52v_Kampfwagen.jpg

http://i41.tinypic.com/bg3cx.jpg

War wagons provide a pretty quick rallying point for both cavalry and infantry. i would imagine in an era of winged attackers wooden walls made to unfold and extend horizontally instead of vertically could help provide cover and concealment.

https://qph.ec.quoracdn.net/main-qimg-8669b47b4e05a9ec3c9ba879c29ea88c-c

In addition, you could have some means to make large quantities of smoke on your wagon. Smoke would be a good defense. And even maybe a place to anchor 'war-kites' :) no idea how effective that could be but I would love to see it.

Hand mortars (i.e. medieval grenade launchers) could be and definitely were carried on horseback, pre-loaded and ready to go. Maybe too dicey for the actual grenades (unless in the hands of a very skilled operator) but the giant firecracker mode was in use in the 15th Century and could be pretty effective too I bet at disrupting a flying attack and maybe making them crash. Guns too though that is a little dicey - crossbows would be more common in the late medieval (quite ubiquitous).



But good for harassing fire - equivalent to flight arrows. I bet they would bother horses. You could also drop caltrops.



But the 'groundlings' could rally to a wagonberg, which can be put together in minutes.



Even just a basic 'naptha coctail' would be pretty dangerous - I think this would indeed be one of the flyers most dangerous weapons. But this would also make them vulnerable to fireworks.

This almost sounds like it's ready to be a pretty good novel or video game. Certainly a good RPG setting.

G

About war wagons, light cavalry still is constrained by the terrain. If you expect them to attack you, you can deduce where are they going to come from, and send your own light cavalry scouts so you get an early warning...

Flyers wouldn't be constrained by terrain, they could appear literally from anywhere. And they probably can outpace your light cavalry scouts (who would be slowed by hills, woods, rivers, swamps...etc.)... so you will have trouble setting your wagonberg on time.

I also remember reading that the knightly armies opposing the hussites never tried to ambush and surprise attack the war wagon convoys before they had time to form the wagonberg, and they never tried to mount a literal siege against a wagonberg... so the effectivity of war wagons may be largely due to poor tactics from their opponents...


So, if arrows have to obey the laws of physics, so do your flyers.

Human sized flyers would need to be either very very light, or have very very large wings, or be awkward in flight, flying like turkeys, not falcons.

So, a slow, ponderous dive bomber with a huge wingspan would be easy meat for foot archers or gunners.

Now, they still would be very useful for scouting and behind the lines raiding and high altitude bombing of soft targets

I just think the Stuka Men idea only works if you handwave the crap out of flight physics. If you handwave a 100 kg man who can fly like a hawk and shoot a bow while flapping his wings but insist that a longbow arrow can't rise high enough to hit him without making Newton cry, yeah then the flyers win.

A winged humanoid just couldn't fly at all. Period.

If we accept that they are humanlike beings who can fly, then we have to assume they are human only on the outside, and that their insides are something completly alien, different not just from mammals, but from any vertebrate...

So maybe they can fly like a flesh helicopter while shooting arrows or darts. Maybe they drink ethanol and use it as effectively as a Rotax 914 does...

Mike_G
2018-01-17, 04:08 PM
A winged humanoid just couldn't fly at all. Period.

If we accept that they are humanlike beings who can fly, then we have to assume they are human only on the outside, and that their insides are something completly alien, different not just from mammals, but from any vertebrate...

So maybe they can fly like a flesh helicopter while shooting arrows or darts. Maybe they drink ethanol and use it as effectively as a Rotax 914 does...

Which is why this is a question we can really only answer if we nail down the ground rules.

If the flyers are "every bit as good as leg infantry, but they can fly" then they are going to be pretty much unbeatable.

If they are big and strong as regular humans, but can only fly low and slow and can't carry much, that's one scenario. If they are smaller and weaker and can't wear armor or use bows, but can fly high and be very maneuverable, that's a different scenario.

If you want to look at more "realism" then they need to be fairly light, and have big wings compared to their bodies and have limited carrying capacity and limited ability to wear armor. Sure you can make up anything in fantasy, but "How do we fight angry pixies" is a different question from "How do we fight Superman"

The usefulness of both your flyers and the countermeasures depend on the capabilities you give the flyers.

Galloglaich
2018-01-17, 05:27 PM
About war wagons, light cavalry still is constrained by the terrain. If you expect them to attack you, you can deduce where are they going to come from, and send your own light cavalry scouts so you get an early warning...

Flyers wouldn't be constrained by terrain, they could appear literally from anywhere. And they probably can outpace your light cavalry scouts (who would be slowed by hills, woods, rivers, swamps...etc.)... so you will have trouble setting your wagonberg on time.

There was a famous standoff between the forces of George of Podebrady and Matthias Corvinus in the 1470's - George's army was heavy in infantry and war-wagons, Matthias' army was heavy in light cavalry (Hungarian Hussars). Jan Dlugosz noted that the issue was that the light cavalry was able to get up into the hills where the war - wagons couldn't reach, but they could not risk coming to grips with the wagons. So I think the analogy is actually pretty close.

As for how long it takes to set a wagonberg -the wagons could provide cover without creating a fort. They were often used this way - the Czechs would for example send out a small group of 3-4 wagons with a banner of infantry and a few lances - the wagons are there for cover and fire support but there aren't enough of them to form a wagonberg.



I also remember reading that the knightly armies opposing the hussites never tried to ambush and surprise attack the war wagon convoys before they had time to form the wagonberg, and they never tried to mount a literal siege against a wagonberg... so the effectivity of war wagons may be largely due to poor tactics from their opponents...

No offense but I think you remember wrong. Or more precisely, most histories of the use of medieval war wagons are cursory and typically only focus on the early years of the Hussite Wars in the 1420's.

So I hope you will forgive a diversion into their history

War-wagons are documented as being used in what you might call the 'Late Medieval' manner including with guns, from back into the third quarter of 14th Century and into the 17th - they were involved in almost every kind of military situation you can think of. The Swiss used them, the Germans used them, the Poles used them, the Lithuanians used them in their Genoese financed 'Crusade' against the Golden Horde from 1397-1399. The Teutonic Knights used them in the Hunger War and in the 13 Years War and other conflicts, the Cossacks used them routinely and so on. They faced everything from knights to Ottoman Sipahi to every kind of light cavalry, horse archer etc. you can imagine.

The specific Czech / Hussite war wagon tactics (basically incrementally improved over earlier uses) were used in the initial phase of the Hussite Wars in the 1420's as you often read about, then also in the later stages including the 'Beautiful Rides' (brutal raids) into neighboring lands for hundreds of miles around Bohemia, and then in the Hussite civil war, then in the 1460's - 1470's in wars against Hungary - and by Czech mercenaries in the Fekete Sereg against the Ottomans all through the 15th Century, and so on.

In fact, not only were there dozens of cases where there were attempts to ambush columns, the wagons in several cases were actively used in moving columns to flank enemy cavalry. Not always successful but often they were. The last time I know of was by the Muscovites against the Crimean Horde at Molodi in 1572 (https://en.wikipedia.org/wiki/Battle_of_Molodi).

The Cossacks were still using War Wagons effectively against the Poles in the 17th Century.

This old 1950's communist era historical movie about the (15th Century) Hussite Wars from the Czechs gives you an idea how long it takes for a war wagon column to deploy into a berg (i.e., not long, maybe about forty five seconds to a minute? Maybe twice that to set up the guns and everything. But it's a far cry from the sort of Monty Python comedy of errors you might envision.) Their flexibility in deployment was key to their effective use. Their main opponent was cavalry after all.

https://youtu.be/WGfBUG44w8g?t=250

Of course that means you still need some advanced warning, so you would need good scouts - this was a key aspect of the Hussite system (Jan Ziska famously mounted Czech scouts on captured Crusader horses and armed them with crossbows.) Assuming you have scouts out with signaling ability (whistling arrows, noisemakers, flares etc.) and presumably, something like spyglasses or binoculars (both available in the 15th Century) and depending on the terrain, weather and so on, you should be able to be ready for an attack. Not always probably, but I would think often enough.

In this Polish movie about the Deluge (set in the 17th Century) you can similarly see a substantial Cossack army relying on their Czech style war-wagons to carry their light and medium sized guns, and to form barriers, again very rapidly. You can see the tabor being opened at 1:11. At 1:28 you can see the Cossacks retreating back into their wagonburg after a feinted sortie and preparing their swivel guns at 1:34.

https://youtu.be/4j4AYK8KGKU?t=58

G

Mr Beer
2018-01-17, 05:59 PM
As for the flyers, small darts wouldn't do the trick; they would stop accelerating after achieving terminal velocity, and that wouldn't be enough to pierce any armor; you would need heavy darts for that...

So the modern weapon I linked to last thread used gravity powered 0.5oz (13g) projectile that would have pierced armour:

https://en.wikipedia.org/wiki/Lazy_Dog_(bomb)

Shape 5, an improved basic Lazy Dog slug, had the force of a .50 caliber bullet and could penetrate 24 inches (61 cm) of packed sand. Shape 2 could penetrate 12 inches (30 cm) of sand — twice as much as a .45 caliber slug fired point blank.

Regardless of how they were released into the air, each "Lazy Dog" projectile would develop an enormous amount of kinetic energy as it fell, penetrating nearly any material upon hitting the ground. Some reports[citation needed] say that their speeds often exceeded 500 mph before impact.

But I wondered how far they'd have to fall to reach say 500mph and I used this online calculator because my physics is weak:

https://www.angio.net/personal/climb/speed

Calculator seems to ignore the effect of air resistance but should be good enough for rule of thumb with dense aerodynamic objects.

This has a 13g projectile reaching 812.68kmh (bit over 500mph) at 2,500 metres fall. It takes 23 seconds to arrive and hits with 331 joules. I assume this would pierce armour but it's a bloody long way up.

It might be better to use heavier projectile from a lower height. From say 200 metres, we can use a 150g dart to hit for 294 joules in 6 seconds. That seems like it may be more practical. But of course you get x10 as many projectiles with the lighter darts.

Either way, your human strength flyer can carry quite a few projectiles. It shouldn't be carrying just two or three very heavy darts except for specialist missions, but rather dozens or hundreds to drop on formations. I suspect there's probably a sweet spot there somewhere between the mile-high saturation attacks and precision javelin sniping from near-ground level but I don't think it's easy to determine where it lies.

Haighus
2018-01-17, 07:18 PM
So I have had a good think about possible biology for plausible (non-magical) bird humans. I am not going to go into much detail of how they evolve, because the proposed model of being like a stereotypical angel, with separate arms and wings, would be extremely implausible to evolve when all other vertebrates operate the same 4 limbs principle. A true flying humanoid is more likely to be like a bat, and not having access to the arms for operating weapons would drastically reduce their combat potential, with dive bombing and blind bombing using simple drops from handheld munitions being the only practical aerial attack methods ( I doubt they would be able to use belts/pouches whilst flying). In a bat-like scenario, they would be limited to the (still very useful) intelligence and communication roles.

First of all- size and weight.

The heaviest flying (and arguably largest) known bird ever is this beast (https://en.wikipedia.org/wiki/Argentavis), Argentavis magnificens. It is estimated to have weighed about 70kg, a height of 1-2m and have a wingspan of 5-6m. This conveniently puts it at about the size and weight of a skinny human with wings instead of arms. So as a very quick proof-of-concept, flying humans could work on a biological level.

We can expect a humanoid that is capable of reasonable function on the ground (so walking and using arms as a normal human can) will have a much greater weight burden than a bird, which is focussed on flying. Therefore our flying humans have several options:
An even greater wingspan than above (this has increasingly diminishing returns)
Some reduced function over normal humans, the easiest would be very underdeveloped legs which would result in much poorer walking ability
Reduced size of the human attached to the wings, to bring the weight down.

The info we have been given about these birdpeople is that they are similar in size to normal humans, and have similar capabilities to humans, but with the addition of flying. I think it is reasonable to expect a degree of reduced function in walking and running endurance, due to lack of use compared to a conditioned human, and that could drop the weight by 10kg or so at most.

Lets assume a slim human of roughly average height, about 50kg (7st. 12Ib) and 1.6m (5' 3"). This is without wings. The size of wings that would be needed to support this is pretty big. Birds apparently have a wing-loading between 1-20kg/m2, and the theoretical upper limit to maintain avian flight is 25kg/m2. The Argentavis bird above had a wing loading of 8.6kg/m2. At a wing loading of 20kg/m2, the wing area would have to be 2.3m2 just to support the body, not including the wings themselves. However, this seems to be for ungainly birds like waterfowl and waders, which don't need good aerodynamics. Our flying humans probably have to be fairly manoeuvrable, so lets use the wing loading for Argentavis, as it was a bird of prey and may have hunted.

Working out the weight of the wings themselves has been challenging. However, I found this source (http://my.kwic.com/~pagodavista/schoolhouse/species/birds/body.htm), which states that the wings of a bald eagle weigh about 0.9kg for a 2.3m wingspan, and I've discovered (http://journals.plos.org/plosbiology/article?id=10.1371/journal.pbio.0050197) that the similarly sized and closely related white-tailed eagle has a wing area of about 0.9m2. This means about 1kg per m2 of wing. From this, we can work out a very rough estimate of how much wing is needed to carry our birdperson with a wing loading of 8.6kg/m2. This gives 5.8m2 for our human weight, and a wing area of about 6.6m2 when including the weight of the wings. Using the size of Argentavis' wingspan compared to wing area (http://www.pnas.org/content/104/30/12398.full.pdf), we can work out a wingspan of roughly 7.6m.

This gives the total size of our bird people as about 55kg, with a wingspan of 7.6m. This is not hugely bigger than Argentavis, so we can assume our bird people are reasonably manouevrable, but would have great difficulty in taking off. This is their most vulnerable time. They also would be far less agile than falcons and eagles and hawks, and would only be able to sustain rapid flight for brief periods (flapping the huge wings). This is likely also an underestimate- I've not taken into account the differences in musculature across the shoulders and chest needed to operate the wings, and this would likely add a good few kilos more, and correspondingly more wing space (apparently (http://www.pnas.org/content/104/30/12398.full.pdf), flapping muscles require ~17% of a bird's bodyweight). In addition, the wings themselves may need to be a bit heavier near to the torso to support their own weights, so my wing weight estimates are likely too low. I may come back and redo my calculations to take into account a beefy chest for wing flapping.


Speed needed to maintain flight.

Argentavis apparently needs at least a speed of 11m/s (24.6mph, 39.6kph) to sustain flight. I would suspect a similar speed would be needed for our flying human, as we have given it the same wing loading. The cruising speed for large birds is around this speed too (http://journals.plos.org/plosbiology/article?id=10.1371/journal.pbio.0050197), so basically our bird people would likely stay at about 25-30mph the majority of the time. Interestingly, the maximum gliding speed (http://www.pnas.org/content/104/30/12398.full.pdf) increases with the size of the bird, so the bird folk could glide as fast as 20mph with little effort. With favourable conditions (slope and thermal soaring) they could reach speeds of 40mph, but this would only be sustainable as long as these conditions persist. Slope soaring requires mountains and hills, so mountainous regions would increase their mobility drastically. The upper limit of speed for birds in a dive can vary considerably, and large birds like eagles can reach 200mph, but I doubt our flying humans would be able to tolerate these speeds with their excessively large wingspan, and would struggle to pull out of such a dive without damaging something or doing it very gradually (so high off the ground). In addition, after a dive, they would struggle to regain altitude without a thermal, meaning they would be vulnerable to ground fire after they stuka-dive enemies. Related to this, the maximum altitude of such large birds is often around 5000m, especially if they can hitch lifts on thermals. Bird people would not be able to reach those heights easily, would be able to see very little, have little ability to hit even an army and would take awhile to descend. I think these heights would only be used for long-range scouting and observation. An operating height of about 500m seems much more practical for combat purposes, and if they operated their supply bases from cliff-tops, they would be able to land and take off easily. Flat plains areas would be the regions they expend most energy to operate (gaining altitude) and take the longest to get into position. In fact, it may be impossible for them to take off from flat terrain without a suitable headwind to generate lift. These are the best regions if you want to defend against them or attack. Mountains would be a nightmare to combat them. Attacking on a windless day would also make it more difficult for them.


Next up- power output and energy consumption.

So, again using Argentavis as our best available comparison, this beast requires 3.5 times more energy than it could sustainably output. The flying humans would need even more energy to maintain powered flight, and would likely be even more underpowered for flapping due to it's shared biolgy with arms. So from this, it is safe to assume our birdpeople are gliders, that can only sustain flapping for a brief period (such as takeoff) before suffering fatigue. This makes their combat potential very limited in endurance, unless they simply glide over. There is going to be no agile dives and regaining altitude quickly, at most short dives and dropping payload, before leaving the area to rearm and report back observations. Because of this lack of available power, their flight paths are also going to be considerably more predictable, because they can only sustain erratic course corrections briefly. Therefore firearms could be reasonably effective shooting on-mass.

The power estimated for Argentavis to maintain flight is 600w, but is negligible when gliding or soaring. Our bird people have larger wings and a less efficient aerodynamic weight, although are slightly lighter, so their necessary sustained power could well be considerably higher. I don't have the measurements or algorithms needed to calculate this, but I would not be surprised if it was double. Lets say 1,000w. This is 1,000J per second of active flying, so take-off and combat manouevring is going to be hugely energy intensive- 1 minute of active flying is 60kJ. A normal human requires at least 7,500kJ per day, according to the World Health Organisation, so that is the energy level for basic human metabolism plus some daily activity. Combat flying is likely to add several thousand more kilojoules of energy to that total, maybe even doubling it, if they run a lot of sorties. This is possible to eat, humans can consume 15,000kJ of energy per day, and have to in conditions like cystic fibrosis. This is going to require essentially 50% more rations for actively flying troops per day, and they are going to exhaust themselves pretty quickly in repeated attacks with their low sustainable power threshold. Launching from a clifftop would reduce the energy costs considerably and aid logistics. Simply scouting and remaining at altitude could be far more efficient than walking though, so using the flyers for reconnaisance would be logistically easier.

When laden with a combat load, taking off on the flat would be impossible without assistance (catapult maybe?) and gaining altitude would require thermals and wind with the poor sustainable power. Combat flying would also be drastically affected until munitions are offloaded. Armour at best would be thin cloth (this could prove fatal with rain), but that may be enough to protect against light arrows at the top of their arcs at higher altitudes.


Combat

Active flying largely uses the pectoralis muscle on the anterior chest wall. This muscle also acts on human arms, but is not that involved in throwing or drawing a bow. In gliding flight, it could be used to stabilise the shoulder whilst light weapons are thrown or drawn, but it would be impossible to utilise the back muscles for throwing/drawing a bow without destabilising the flight. It may be possible to accept a temporay drop and then recover into stable flight, but I think in practice gravity will be doing most of the work for projectiles. A dropped soliferrum would still be effective, although a dropped pebble is going to be fairly unimpressive. Medium sized rocks are probably the minimum for useful damage, other than caltrops. The limited munitions due to weight is also an issue, along side balance and storage. Flying with a bag would be disruptive to aerodynamics, so I wouldn't be surprised if such flyers were limited to just an item in each hand, and maybe some small gear on belts. In addition, incendiaries would be pretty much impossible to light on the wing, and fuses could only be lit prior to flying if the supply base is near to the combat. Thankfully, I think these are pretty limited in scope, unless the bird folk develop some kind of working percussion fuse in a suitably lightweight device. If they do, making the fuses unreliable could be a good way to balance this.


Summary

The bird folk could fly, and could eat enough to do it, but their combat capabilities would be limited without nearby resupply, they would tire easily in such conditions, would be very vulnerable without suitable terrain to take off, and their poor ability to dodge and enormous wingspan would make them huge targets if they come within range of the ground folks' weaponry. Huge vulnerable targets, seeing as it wouldn't take much damage to a wing to make it either lame or unable to sustain sufficient lift. Any such injury whilst flying would likely result in a greater proportion of casualties than ground troops too, so the risks are greater. Scouting, communication, observation and spying seem like by far the best options for employing flying troops, as well as ambushes in favourable regions. They would rule the mountains, but fear the plains.

Clistenes
2018-01-17, 07:22 PM
*snip*

Well, you obviously have done your research, so I have to concede that war wagons were very effective.

But if we are speaking of this thread's flyers, I still think they could still reach the convoys before they had time to prepare their defenses.


*snip*.

I find it quite surprising that a mere 13 g kinetic bombardment projectile could pack such power. I guess their aerodinamic shape helped quite a lot to reduce air resistance and achive high terminal velocity...

Haighus
2018-01-17, 07:30 PM
Well, you obviously have done your research, so I have to concede that war wagons were very effective.

But if we are speaking of this thread's flyers, I still think they could still reach the convoys before they had time to prepare their defenses.



I find it quite surprising that a mere 13 g kinetic bombardment projectile could pack such power. I guess their aerodinamic shape helped quite a lot to reduce air resistance and achive high terminal velocity...

I find it very surprising too. I wonder if they would be able to make such efficient darts in a medieval tech setting. Flying at 1000m altitude and dropping 50 of those each whilst travelling over a moving convoy could certainly cause some casualties even with the low accuracy. At least a great harassment method.

As for war wagons and time- flying enemies can be seen from further away than enemies hiding in terrain. It would be hard to ambush unless they are flying very low (and thus vulnerable to retaliation) or in mountainous terrain where they can leap off cliffs. Also, a scout picket with horns or bugles could be used to provide advance warning of incoming flyers if necessary.

rs2excelsior
2018-01-17, 08:05 PM
This almost sounds like it's ready to be a pretty good novel or video game. Certainly a good RPG setting.

G

I certainly hope that's the case :smallbiggrin:


So I have had a good think about possible biology for plausible (non-magical) bird humans.

-snip-

Summary

The bird folk could fly, and could eat enough to do it, but their combat capabilities would be limited without nearby resupply, they would tire easily in such conditions, would be very vulnerable without suitable terrain to take off, and their poor ability to dodge and enormous wingspan would make them huge targets if they come within range of the ground folks' weaponry. Huge vulnerable targets, seeing as it wouldn't take much damage to a wing to make it either lame or unable to sustain sufficient lift. Any such injury whilst flying would likely result in a greater proportion of casualties than ground troops too, so the risks are greater. Scouting, communication, observation and spying seem like by far the best options for employing flying troops, as well as ambushes in favourable regions. They would rule the mountains, but fear the plains.

Wow. I'd come here expecting a bit of discussion on medieval weaponry. I hadn't expected an in-depth analysis of the flight physics of winged humanoids. This is awesome, and also the reason I love this forum :smallsmile:

I'll be honest, I hadn't put much thought into their evolution or biology. Flying humanoids were simply a neat fantasy trope I decided to use (and, most likely, not the most biologically implausible one at that). I'll definitely work in some of this when I work up their place in the setting.

Along these lines, that indicates they'd probably focus on producing fairly high-calorie foods. Any suggestions? Primarily plants or meat? I think meat has a higher energy density, but also requires more energy to produce (since you have to feed an animal more than 1 calorie of food to produce 1 calorie of edible meat).

Interesting points about the plains as well. I'd thought about putting the centers of human resistance in a mountainous region; I might change that. Heavy forests, perhaps--negates many of the advantages of the flyers while still causing difficulties for their ground troops. For that matter, heavy forest would make it incredibly difficult for flyers to actually attack their targets, or even see them.

Mr Beer
2018-01-17, 08:06 PM
I find it quite surprising that a mere 13 g kinetic bombardment projectile could pack such power. I guess their aerodinamic shape helped quite a lot to reduce air resistance and achive high terminal velocity...

Yeah, if I'd have had to guess, I would have thought they'd need to weigh more like 100g to 200g.


I find it very surprising too. I wonder if they would be able to make such efficient darts in a medieval tech setting. Flying at 1000m altitude and dropping 50 of those each whilst travelling over a moving convoy could certainly cause some casualties even with the low accuracy. At least a great harassment method.

My guess is that medieval tech would be up to the challenge. The complexity is trivial. The metallurgy required is presumably below what was used in swords or armour. The improved darts were developed well before computer modelling, really all they needed were various designs and bombing runs to test penetration.

Carl
2018-01-17, 09:51 PM
So I have had a good think about possible biology for plausible (non-magical) bird humans. I am not going to go into much detail of how they evolve, because the proposed model of being like a stereotypical angel, with separate arms and wings, would be extremely implausible to evolve when all other vertebrates operate the same 4 limbs principle. A true flying humanoid is more likely to be like a bat, and not having access to the arms for operating weapons would drastically reduce their combat potential, with dive bombing and blind bombing using simple drops from handheld munitions being the only practical aerial attack methods ( I doubt they would be able to use belts/pouches whilst flying). In a bat-like scenario, they would be limited to the (still very useful) intelligence and communication roles.

First of all- size and weight.

The heaviest flying (and arguably largest) known bird ever is this beast (https://en.wikipedia.org/wiki/Argentavis), Argentavis magnificens. It is estimated to have weighed about 70kg, a height of 1-2m and have a wingspan of 5-6m. This conveniently puts it at about the size and weight of a skinny human with wings instead of arms. So as a very quick proof-of-concept, flying humans could work on a biological level.

We can expect a humanoid that is capable of reasonable function on the ground (so walking and using arms as a normal human can) will have a much greater weight burden than a bird, which is focussed on flying. Therefore our flying humans have several options:
An even greater wingspan than above (this has increasingly diminishing returns)
Some reduced function over normal humans, the easiest would be very underdeveloped legs which would result in much poorer walking ability
Reduced size of the human attached to the wings, to bring the weight down.

The info we have been given about these birdpeople is that they are similar in size to normal humans, and have similar capabilities to humans, but with the addition of flying. I think it is reasonable to expect a degree of reduced function in walking and running endurance, due to lack of use compared to a conditioned human, and that could drop the weight by 10kg or so at most.

Lets assume a slim human of roughly average height, about 50kg (7st. 12Ib) and 1.6m (5' 3"). This is without wings. The size of wings that would be needed to support this is pretty big. Birds apparently have a wing-loading between 1-20kg/m2, and the theoretical upper limit to maintain avian flight is 25kg/m2. The Argentavis bird above had a wing loading of 8.6kg/m2. At a wing loading of 20kg/m2, the wing area would have to be 2.3m2 just to support the body, not including the wings themselves. However, this seems to be for ungainly birds like waterfowl and waders, which don't need good aerodynamics. Our flying humans probably have to be fairly manoeuvrable, so lets use the wing loading for Argentavis, as it was a bird of prey and may have hunted.

Working out the weight of the wings themselves has been challenging. However, I found this source (http://my.kwic.com/~pagodavista/schoolhouse/species/birds/body.htm), which states that the wings of a bald eagle weigh about 0.9kg for a 2.3m wingspan, and I've discovered (http://journals.plos.org/plosbiology/article?id=10.1371/journal.pbio.0050197) that the similarly sized and closely related white-tailed eagle has a wing area of about 0.9m2. This means about 1kg per m2 of wing. From this, we can work out a very rough estimate of how much wing is needed to carry our birdperson with a wing loading of 8.6kg/m2. This gives 5.8m2 for our human weight, and a wing area of about 6.6m2 when including the weight of the wings. Using the size of Argentavis' wingspan compared to wing area (http://www.pnas.org/content/104/30/12398.full.pdf), we can work out a wingspan of roughly 7.6m.

This gives the total size of our bird people as about 55kg, with a wingspan of 7.6m. This is not hugely bigger than Argentavis, so we can assume our bird people are reasonably manouevrable, but would have great difficulty in taking off. This is their most vulnerable time. They also would be far less agile than falcons and eagles and hawks, and would only be able to sustain rapid flight for brief periods (flapping the huge wings). This is likely also an underestimate- I've not taken into account the differences in musculature across the shoulders and chest needed to operate the wings, and this would likely add a good few kilos more, and correspondingly more wing space (apparently (http://www.pnas.org/content/104/30/12398.full.pdf), flapping muscles require ~17% of a bird's bodyweight). In addition, the wings themselves may need to be a bit heavier near to the torso to support their own weights, so my wing weight estimates are likely too low. I may come back and redo my calculations to take into account a beefy chest for wing flapping.


Speed needed to maintain flight.

Argentavis apparently needs at least a speed of 11m/s (24.6mph, 39.6kph) to sustain flight. I would suspect a similar speed would be needed for our flying human, as we have given it the same wing loading. The cruising speed for large birds is around this speed too (http://journals.plos.org/plosbiology/article?id=10.1371/journal.pbio.0050197), so basically our bird people would likely stay at about 25-30mph the majority of the time. Interestingly, the maximum gliding speed (http://www.pnas.org/content/104/30/12398.full.pdf) increases with the size of the bird, so the bird folk could glide as fast as 20mph with little effort. With favourable conditions (slope and thermal soaring) they could reach speeds of 40mph, but this would only be sustainable as long as these conditions persist. Slope soaring requires mountains and hills, so mountainous regions would increase their mobility drastically. The upper limit of speed for birds in a dive can vary considerably, and large birds like eagles can reach 200mph, but I doubt our flying humans would be able to tolerate these speeds with their excessively large wingspan, and would struggle to pull out of such a dive without damaging something or doing it very gradually (so high off the ground). In addition, after a dive, they would struggle to regain altitude without a thermal, meaning they would be vulnerable to ground fire after they stuka-dive enemies. Related to this, the maximum altitude of such large birds is often around 5000m, especially if they can hitch lifts on thermals. Bird people would not be able to reach those heights easily, would be able to see very little, have little ability to hit even an army and would take awhile to descend. I think these heights would only be used for long-range scouting and observation. An operating height of about 500m seems much more practical for combat purposes, and if they operated their supply bases from cliff-tops, they would be able to land and take off easily. Flat plains areas would be the regions they expend most energy to operate (gaining altitude) and take the longest to get into position. In fact, it may be impossible for them to take off from flat terrain without a suitable headwind to generate lift. These are the best regions if you want to defend against them or attack. Mountains would be a nightmare to combat them. Attacking on a windless day would also make it more difficult for them.


Next up- power output and energy consumption.

So, again using Argentavis as our best available comparison, this beast requires 3.5 times more energy than it could sustainably output. The flying humans would need even more energy to maintain powered flight, and would likely be even more underpowered for flapping due to it's shared biolgy with arms. So from this, it is safe to assume our birdpeople are gliders, that can only sustain flapping for a brief period (such as takeoff) before suffering fatigue. This makes their combat potential very limited in endurance, unless they simply glide over. There is going to be no agile dives and regaining altitude quickly, at most short dives and dropping payload, before leaving the area to rearm and report back observations. Because of this lack of available power, their flight paths are also going to be considerably more predictable, because they can only sustain erratic course corrections briefly. Therefore firearms could be reasonably effective shooting on-mass.

The power estimated for Argentavis to maintain flight is 600w, but is negligible when gliding or soaring. Our bird people have larger wings and a less efficient aerodynamic weight, although are slightly lighter, so their necessary sustained power could well be considerably higher. I don't have the measurements or algorithms needed to calculate this, but I would not be surprised if it was double. Lets say 1,000w. This is 1,000J per second of active flying, so take-off and combat manouevring is going to be hugely energy intensive- 1 minute of active flying is 60kJ. A normal human requires at least 7,500kJ per day, according to the World Health Organisation, so that is the energy level for basic human metabolism plus some daily activity. Combat flying is likely to add several thousand more kilojoules of energy to that total, maybe even doubling it, if they run a lot of sorties. This is possible to eat, humans can consume 15,000kJ of energy per day, and have to in conditions like cystic fibrosis. This is going to require essentially 50% more rations for actively flying troops per day, and they are going to exhaust themselves pretty quickly in repeated attacks with their low sustainable power threshold. Launching from a clifftop would reduce the energy costs considerably and aid logistics. Simply scouting and remaining at altitude could be far more efficient than walking though, so using the flyers for reconnaisance would be logistically easier.

When laden with a combat load, taking off on the flat would be impossible without assistance (catapult maybe?) and gaining altitude would require thermals and wind with the poor sustainable power. Combat flying would also be drastically affected until munitions are offloaded. Armour at best would be thin cloth (this could prove fatal with rain), but that may be enough to protect against light arrows at the top of their arcs at higher altitudes.


Combat

Active flying largely uses the pectoralis muscle on the anterior chest wall. This muscle also acts on human arms, but is not that involved in throwing or drawing a bow. In gliding flight, it could be used to stabilise the shoulder whilst light weapons are thrown or drawn, but it would be impossible to utilise the back muscles for throwing/drawing a bow without destabilising the flight. It may be possible to accept a temporay drop and then recover into stable flight, but I think in practice gravity will be doing most of the work for projectiles. A dropped soliferrum would still be effective, although a dropped pebble is going to be fairly unimpressive. Medium sized rocks are probably the minimum for useful damage, other than caltrops. The limited munitions due to weight is also an issue, along side balance and storage. Flying with a bag would be disruptive to aerodynamics, so I wouldn't be surprised if such flyers were limited to just an item in each hand, and maybe some small gear on belts. In addition, incendiaries would be pretty much impossible to light on the wing, and fuses could only be lit prior to flying if the supply base is near to the combat. Thankfully, I think these are pretty limited in scope, unless the bird folk develop some kind of working percussion fuse in a suitably lightweight device. If they do, making the fuses unreliable could be a good way to balance this.


Summary

The bird folk could fly, and could eat enough to do it, but their combat capabilities would be limited without nearby resupply, they would tire easily in such conditions, would be very vulnerable without suitable terrain to take off, and their poor ability to dodge and enormous wingspan would make them huge targets if they come within range of the ground folks' weaponry. Huge vulnerable targets, seeing as it wouldn't take much damage to a wing to make it either lame or unable to sustain sufficient lift. Any such injury whilst flying would likely result in a greater proportion of casualties than ground troops too, so the risks are greater. Scouting, communication, observation and spying seem like by far the best options for employing flying troops, as well as ambushes in favourable regions. They would rule the mountains, but fear the plains.

Allright a fairly large number of comments.

1. The idea of a bird human with seperate arms, legs, and wings is completely reasonable. Tool use is literally vital to exploiting intelligence. and it's unlikely arms incorporated into wings would be anywhere near dexterous enough. There's plenty of examples of animals that have had limbs ethier split or merge in their evolutionary history so that part at least is reasonable. We don't see anything like it in nature because advanced tool use outside of humans is very rare, though a few examples have begun to turn up once people started to get past their "dumb animal" preconceptions.

2. Your wing loading figure may actually be too high still, as wing loading and flight speed are connected, and a 20mph+ flight speed could create some real issues taking off.

3. For the same reasons, (high takeoff speed), he legs might be more robust than your imagining. They'd probably still not be suited to distance running, but in short sprints probably quite good.

4. The wing weight is certainly on the low side, the closer to the end of the wing you get the less force it has on it as inner wing area's are transferring all the force from the outer sections in addition to their own.

3. Energy usage is going to vary with flight. Active flight like say a magpie assuming similar efficiencies would be around 10-11kw @ 15mph. A marathon runner uses between 10 and 12 MJ's in between 2 and 3 hours. So your looking at roughly an order of magnitude increase in oxygen intake required, and a much better means of storing large quantities of glucose. The latter is probably more reasonable, an enlarged and modified lifer or a lager secondary liver for the purpose should d it if i'm understanding the stuff i've been reading. The oxygen, thats a bit of an issue, i don't think it's insurmountable, from a purely theoretical standpoint many smaller lungs should allow a significant increase in oxygen uptake compared to humans and i'm sure there are adaptions to increase it even further, the other hard part would be avoiding oxygen narcosis. Again manageable but severely difficult.


I certainly hope that's the case :smallbiggrin:



Wow. I'd come here expecting a bit of discussion on medieval weaponry. I hadn't expected an in-depth analysis of the flight physics of winged humanoids. This is awesome, and also the reason I love this forum :smallsmile:

I'll be honest, I hadn't put much thought into their evolution or biology. Flying humanoids were simply a neat fantasy trope I decided to use (and, most likely, not the most biologically implausible one at that). I'll definitely work in some of this when I work up their place in the setting.

Along these lines, that indicates they'd probably focus on producing fairly high-calorie foods. Any suggestions? Primarily plants or meat? I think meat has a higher energy density, but also requires more energy to produce (since you have to feed an animal more than 1 calorie of food to produce 1 calorie of edible meat).

Interesting points about the plains as well. I'd thought about putting the centers of human resistance in a mountainous region; I might change that. Heavy forests, perhaps--negates many of the advantages of the flyers while still causing difficulties for their ground troops. For that matter, heavy forest would make it incredibly difficult for flyers to actually attack their targets, or even see them.

Food would almost have to be majority honey. It's the only calorie dense enough natural thing i can think of and dug up data on that works. If they had better tech they could switch to saturated sugar water with is even more energy dense. Which raises the question of what variety of insect is allowing the to produce multiple tens of kilos of honey a day per head, (thats how great the energy requirements are,. we'd probably poison ourselves if we tried to eat somthing like that in such quantities).

Also what plants are those insects getting the material, (presumably nectar), to make honey from? Theys have t be big, abundant and produce enormous quantities of nectar nearly year round. That would likely limit them to near year round sunny climates unless they reguarly migrated on the wing.

Clistenes
2018-01-18, 05:30 AM
I find it very surprising too. I wonder if they would be able to make such efficient darts in a medieval tech setting. Flying at 1000m altitude and dropping 50 of those each whilst travelling over a moving convoy could certainly cause some casualties even with the low accuracy. At least a great harassment method.

It seems that, in the tests, the flechettes where released from planes that were flying at very low altitude, but at high speed... I wonder if the lethality of the projectiles was due to the speed of the plane rather than to gravity itself (the flechettes would start their flight at high speed already...)

Brother Oni
2018-01-18, 07:59 AM
A normal human requires at least 7,500kJ per day, according to the World Health Organisation, so that is the energy level for basic human metabolism plus some daily activity. Combat flying is likely to add several thousand more kilojoules of energy to that total, maybe even doubling it, if they run a lot of sorties. This is possible to eat, humans can consume 15,000kJ of energy per day, and have to in conditions like cystic fibrosis.

7500kJ/day is only about 1800 Calories, which is very low for an active individual. I'll have to do more digging on the processing rate of people, however CF sufferers can only eat that many calories because the vast majority of isn't absorbed due to their disease condition and passes through undigested.


Along these lines, that indicates they'd probably focus on producing fairly high-calorie foods. Any suggestions? Primarily plants or meat? I think meat has a higher energy density, but also requires more energy to produce (since you have to feed an animal more than 1 calorie of food to produce 1 calorie of edible meat).

Definitely meat and fat rich diets (see my comment to Carl below).



Food would almost have to be majority honey. It's the only calorie dense enough natural thing i can think of and dug up data on that works. If they had better tech they could switch to saturated sugar water with is even more energy dense. Which raises the question of what variety of insect is allowing the to produce multiple tens of kilos of honey a day per head, (thats how great the energy requirements are,. we'd probably poison ourselves if we tried to eat somthing like that in such quantities).


Actually honey is mostly carbohydrates, which isn't that energy dense - you're looking at ~3 Calories (~12.6kJ) per gram. Fat, especially processed, is far more energy dense - rendered pig fat (lard) is ~9 Calories (37.6kJ) per gram.

Honey is a lot more bioavailable though, which would be useful for immediate actions, but for long term diet, a meat and fat heavy diet would be required.
One side effect of such a diet would be long rest periods to digest such an energy rich diet - take a look at the activity cycles of lions and other obligate carnivores as an example.

I suppose this could tie in with a more Mediterranean lifestyle for our flying humanoids, with long afternoon siestas after lunch. :smallbiggrin:

Haighus
2018-01-18, 09:56 AM
I certainly hope that's the case :smallbiggrin:



Wow. I'd come here expecting a bit of discussion on medieval weaponry. I hadn't expected an in-depth analysis of the flight physics of winged humanoids. This is awesome, and also the reason I love this forum :smallsmile:

I'll be honest, I hadn't put much thought into their evolution or biology. Flying humanoids were simply a neat fantasy trope I decided to use (and, most likely, not the most biologically implausible one at that). I'll definitely work in some of this when I work up their place in the setting.

Along these lines, that indicates they'd probably focus on producing fairly high-calorie foods. Any suggestions? Primarily plants or meat? I think meat has a higher energy density, but also requires more energy to produce (since you have to feed an animal more than 1 calorie of food to produce 1 calorie of edible meat).

Interesting points about the plains as well. I'd thought about putting the centers of human resistance in a mountainous region; I might change that. Heavy forests, perhaps--negates many of the advantages of the flyers while still causing difficulties for their ground troops. For that matter, heavy forest would make it incredibly difficult for flyers to actually attack their targets, or even see them.
Well, it is an area I am able to make some more detailed analysis in, so I decided to put in some research :) Heavy forest would likely be the best terrain for avoiding them- such a huge wingspan would be a liability in forests, and they'd have very poor visibility. Plains and deserts would be next best. Coasts and mountains are the most in favour of the flyers. I am increasingly seeing them as being ideally like flying Incas, so that may be a cool place to look for ideas. The Incas also venerated the Andean condor, so there is some cool imagery that could be drawn from them. Incas also had very intensive mountainous food production, which brings me on to my next point about energy intake. See below.

Allright a fairly large number of comments.

1. The idea of a bird human with seperate arms, legs, and wings is completely reasonable. Tool use is literally vital to exploiting intelligence. and it's unlikely arms incorporated into wings would be anywhere near dexterous enough. There's plenty of examples of animals that have had limbs ethier split or merge in their evolutionary history so that part at least is reasonable. We don't see anything like it in nature because advanced tool use outside of humans is very rare, though a few examples have begun to turn up once people started to get past their "dumb animal" preconceptions.

2. Your wing loading figure may actually be too high still, as wing loading and flight speed are connected, and a 20mph+ flight speed could create some real issues taking off.

3. For the same reasons, (high takeoff speed), he legs might be more robust than your imagining. They'd probably still not be suited to distance running, but in short sprints probably quite good.

4. The wing weight is certainly on the low side, the closer to the end of the wing you get the less force it has on it as inner wing area's are transferring all the force from the outer sections in addition to their own.

3. Energy usage is going to vary with flight. Active flight like say a magpie assuming similar efficiencies would be around 10-11kw @ 15mph. A marathon runner uses between 10 and 12 MJ's in between 2 and 3 hours. So your looking at roughly an order of magnitude increase in oxygen intake required, and a much better means of storing large quantities of glucose. The latter is probably more reasonable, an enlarged and modified lifer or a lager secondary liver for the purpose should d it if i'm understanding the stuff i've been reading. The oxygen, thats a bit of an issue, i don't think it's insurmountable, from a purely theoretical standpoint many smaller lungs should allow a significant increase in oxygen uptake compared to humans and i'm sure there are adaptions to increase it even further, the other hard part would be avoiding oxygen narcosis. Again manageable but severely difficult.



Food would almost have to be majority honey. It's the only calorie dense enough natural thing i can think of and dug up data on that works. If they had better tech they could switch to saturated sugar water with is even more energy dense. Which raises the question of what variety of insect is allowing the to produce multiple tens of kilos of honey a day per head, (thats how great the energy requirements are,. we'd probably poison ourselves if we tried to eat somthing like that in such quantities).

Also what plants are those insects getting the material, (presumably nectar), to make honey from? Theys have t be big, abundant and produce enormous quantities of nectar nearly year round. That would likely limit them to near year round sunny climates unless they reguarly migrated on the wing.
From what I was reading, large flyers invariably cannot sustain active flight, so they largely rely on environmental factors- thermals and wind, to maintain and gain altitude. Based on this, the bird people would actually have very low energy needs when gliding, which would be most of their flying, and they would not be able to sustain high energy outputs for long without fatiguing. Gliding basically only requires tilting the wing to adjust to changing wind conditions, and slope soaring and thermals can allow forward propulsion without any additional energy input from the bird. I think with this in mind, they would not need much in the way of additional oxygen capacity, because they are largely gliding and soaring. The altitude of 5,000m is within human physiological limits with some adaptation (people can live up to about 6,000m altitude permanently, and survive up to 8,000m for reasonable periods. Beyond 8,000m is when oxygen deprivation becomes short-term life threatening) Only take off would typically be energy intensive, and usually natural features would help this- even a downward slope of 10 degrees, and a head wind of 5mph is enough for Argentavis to be able to take off easily apparently, so similar conditions wold be needed for our birdpeople. I based the flight speeds off the speeds of actual birds and the big prehistoric bird. Only combat, with the additional manoeuvring needs, would be very energy intensive, and it is possible to eat the amounts of energy I posted above by simply eating more meals per day than is typical for humans. I will go into the foodstuffs themselves below.

In terms of size and morphology, I agree when looking back, I think adding 20% weight and wing span would be reasonable, so more like 80kg with a wingspan of 9m (which s enormous, but still smaller than some pterosaurs). There is no example I know of of a vertebrate from amphibians onwards which has more than 4 limbs. This would be a massive change. The way that bat wings work would be the most plausible- a flap of skin extending from the arm along the torso. This would allow the arms to be normally functioning too. You make a good point on the legs likely still being good for sprinting, I agree this is likely. Long distance endurance would be lacking, but then long distance flying is what they will do best.


7500kJ/day is only about 1800 Calories, which is very low for an active individual. I'll have to do more digging on the processing rate of people, however CF sufferers can only eat that many calories because the vast majority of isn't absorbed due to their disease condition and passes through undigested.



Definitely meat and fat rich diets (see my comment to Carl below).



Actually honey is mostly carbohydrates, which isn't that energy dense - you're looking at ~3 Calories (~12.6kJ) per gram. Fat, especially processed, is far more energy dense - rendered pig fat (lard) is ~9 Calories (37.6kJ) per gram.

Honey is a lot more bioavailable though, which would be useful for immediate actions, but for long term diet, a meat and fat heavy diet would be required.
One side effect of such a diet would be long rest periods to digest such an energy rich diet - take a look at the activity cycles of lions and other obligate carnivores as an example.

I suppose this could tie in with a more Mediterranean lifestyle for our flying humanoids, with long afternoon siestas after lunch. :smallbiggrin:
CF sufferers get dietary supplements containing pancreatic enzymes to improve their digestion and absorption of food. The main reason they eat 50% more calories than a typical human should is because of the massive metabolic demands of excessive mucous production and constant laboured breathing and coughing. Active breathing is very energy intensive, so it is one of the reasons people loose weight so quickly when they forced to breath heavy for extended periods.

The 1,800 calories what WHO suggests is the lower limit for a human to survive adequately. This is basically the costs of simply living, plus some activity. I was using it as a baseline for what the bird humans would need if they just lounged around all day. Normal recommended daily intake is about 10,000kJ, so 25% more. CF sufferers eat 50% more than that (ideally). I actually think our bird people would only need about the same intake as normal humans with typical gliding activities, it really is amazingly efficient. Only combat would require more.

Having said this, I agree that meat and fat (and alcohol... It is better than protein and only slightly worse than fat!) are the most energy dense foods. Based on their ideal mountainous location, I think their food production would be similar to the Incas. This would be intensive terraced agriculture to make the most of available terrain, with carbohydrate rich foods like potatoes and proein rich foods like beans, and large scale animal husbandry of creatures like goats, sheep and llamas, which also produce milk and (importantly) butter. Butter is incredibly energy dense with at least 80% fat content, so would be very useful for feeding campaigning armies in intensive combat. It would be most useful in cold conditions were it doesn't perish quickly of course. The Incas actually operated a very sophisticated food production system, with research "labs" to develop new varieties of potatoes and to domesticate new species. Very interesting stuff. I think this could be one of the key advantages of the bird person empire- advanced food production to support a large population of large flyers, which usually have a low population density.

Brother Oni
2018-01-18, 01:10 PM
CF sufferers get dietary supplements containing pancreatic enzymes to improve their digestion and absorption of food. The main reason they eat 50% more calories than a typical human should is because of the massive metabolic demands of excessive mucous production and constant laboured breathing and coughing. Active breathing is very energy intensive, so it is one of the reasons people loose weight so quickly when they forced to breath heavy for extended periods.

Looking at some papers, the increased energy expenditure of CF sufferers is only about 10% more than average, but I concede that any physical activity would cause that to increase exponentially.



Having said this, I agree that meat and fat (and alcohol... It is better than protein and only slightly worse than fat!) are the most energy dense foods. Based on their ideal mountainous location, I think their food production would be similar to the Incas. This would be intensive terraced agriculture to make the most of available terrain, with carbohydrate rich foods like potatoes and proein rich foods like beans, and large scale animal husbandry of creatures like goats, sheep and llamas, which also produce milk and (importantly) butter. Butter is incredibly energy dense with at least 80% fat content, so would be very useful for feeding campaigning armies in intensive combat. It would be most useful in cold conditions were it doesn't perish quickly of course. The Incas actually operated a very sophisticated food production system, with research "labs" to develop new varieties of potatoes and to domesticate new species. Very interesting stuff. I think this could be one of the key advantages of the bird person empire- advanced food production to support a large population of large flyers, which usually have a low population density.

Running with the Incan aesthetic even more, it would give a good reason for their stepped pyramids - if they were living quarters as well, it would make it easier for the flyers to take off, with the highest social rank people living towards the top and the slaves or lowest ranked people living on the ground floor tier.

rs2excelsior
2018-01-18, 01:23 PM
Well, it is an area I am able to make some more detailed analysis in, so I decided to put in some research :) Heavy forest would likely be the best terrain for avoiding them- such a huge wingspan would be a liability in forests, and they'd have very poor visibility. Plains and deserts would be next best. Coasts and mountains are the most in favour of the flyers. I am increasingly seeing them as being ideally like flying Incas, so that may be a cool place to look for ideas. The Incas also venerated the Andean condor, so there is some cool imagery that could be drawn from them. Incas also had very intensive mountainous food production, which brings me on to my next point about energy intake. See below.

...

Having said this, I agree that meat and fat (and alcohol... It is better than protein and only slightly worse than fat!) are the most energy dense foods. Based on their ideal mountainous location, I think their food production would be similar to the Incas. This would be intensive terraced agriculture to make the most of available terrain, with carbohydrate rich foods like potatoes and proein rich foods like beans, and large scale animal husbandry of creatures like goats, sheep and llamas, which also produce milk and (importantly) butter. Butter is incredibly energy dense with at least 80% fat content, so would be very useful for feeding campaigning armies in intensive combat. It would be most useful in cold conditions were it doesn't perish quickly of course. The Incas actually operated a very sophisticated food production system, with research "labs" to develop new varieties of potatoes and to domesticate new species. Very interesting stuff. I think this could be one of the key advantages of the bird person empire- advanced food production to support a large population of large flyers, which usually have a low population density.


Running with the Incan aesthetic even more, it would give a good reason for their stepped pyramids - if they were living quarters as well, it would make it easier for the flyers to take off, with the highest social rank people living towards the top and the slaves or lowest ranked people living on the ground floor tier.

I like the idea of an Incan aesthetic. Unfortunately I don't know all that much about the Incas. Off to do some research! :smallbiggrin: Again, thanks for the discussion, everyone. Lots of really helpful stuff here.

Haighus
2018-01-18, 02:55 PM
Looking at some papers, the increased energy expenditure of CF sufferers is only about 10% more than average, but I concede that any physical activity would cause that to increase exponentially.



Running with the Incan aesthetic even more, it would give a good reason for their stepped pyramids - if they were living quarters as well, it would make it easier for the flyers to take off, with the highest social rank people living towards the top and the slaves or lowest ranked people living on the ground floor tier.

Oh, fair enough, I thought it was larger. That means they don't digest 40% of the extra calories. Hmm, perhaps 50% extra calories may be too much for a human to digest without constant grazing.

Oooh I like the pyramid idea. That is a really interesting concept. Sun temples take on a whole new aspect with their importance for thermals and flying too.

Max_Killjoy
2018-01-18, 03:12 PM
Not sure if it's come up in these discussions since we jumped threads, but I would note again that much of the power for flight comes from the chest and back muscles of birds, rather than from muscles in the wings themselves. Thus the deep chests and large "cuts of meat" therefrom even on domestic meat birds that barely ever fly.

A winged humanoid capable of flight would likely have a significantly different torso build for the same biomechanical reasons.

Haighus
2018-01-18, 03:34 PM
Not sure if it's come up in these discussions since we jumped threads, but I would note again that much of the power for flight comes from the chest and back muscles of birds, rather than from muscles in the wings themselves. Thus the deep chests and large "cuts of meat" therefrom even on domestic meat birds that barely ever fly.

A winged humanoid capable of flight would likely have a significantly different torso build for the same biomechanical reasons.

I agree, based on what I've read (and discussed above), you would have a slender humanoid, with a massively bulky torso (17% of the weight in the torso, which would be about 15kg of 80kg in my modified theoretical), and really huge wings (9m wingspan), which dwarf the rest of the body in size (but not weight). I also suspect the head and neck would be structured a bit differently, to be more streamlined whilst flying and give a more forward looking head position.

spineyrequiem
2018-01-18, 03:42 PM
Depending on how much you're willing to sacrifice ground ability I think you could trim off a bit more leg weight. According to this (http://www.exrx.net/Kinesiology/Segments.html) legs are about 35% of body weight. Reduce that by two thirds and our 70kg man becomes around 52kg. Obviously in this case they'd either have to waddle slowly or wingwalk, but hey, you've got to make some sacrifices. We could also take off the weight of the arms as those're being replaced, but I'm inclined not to so you can factor in the extra weight from flight muscles. 70kg also seems pretty high as an average weight, I'd guess it's based around a 6'-ish man. If we instead base it on the Incas (just to rip off a bit more of their civilisation), with an average male height of 5'2", we get a weight of about 54.5 kg, or 41kg with shorter legs. Note that my maths is not even close to accurate, but it does make for a very slightly more reasonable wingspan.

Haighus
2018-01-18, 04:00 PM
Depending on how much you're willing to sacrifice ground ability I think you could trim off a bit more leg weight. According to this (http://www.exrx.net/Kinesiology/Segments.html) legs are about 35% of body weight. Reduce that by two thirds and our 70kg man becomes around 52kg. Obviously in this case they'd either have to waddle slowly or wingwalk, but hey, you've got to make some sacrifices. We could also take off the weight of the arms as those're being replaced, but I'm inclined not to so you can factor in the extra weight from flight muscles. 70kg also seems pretty high as an average weight, I'd guess it's based around a 6'-ish man. If we instead base it on the Incas (just to rip off a bit more of their civilisation), with an average male height of 5'2", we get a weight of about 54.5 kg, or 41kg with shorter legs. Note that my maths is not even close to accurate, but it does make for a very slightly more reasonable wingspan.

Well, instead of replacing the arms, our humanoids have arms AND wings, which adds a degree of weight. They are supposed to be roughly analogous to humans, but with flying abilities (thus allowing them to operate weapons and fly simultaneously). If you look at my calculations on the previous page, and near the top of this page, I started with a lightweight 50kg human, and added ~20% more weight for flying muscles, plus the weight of the wings. That came out as roughly 80kg altogether.

If we start adjusting the bodies to become more adapted for flying, we loose some of the human capabilities. Depends on what the original poster actually wants for their flying humans really, but I tried to get as close as possible to a plausible human with wings scenario.

Clistenes
2018-01-18, 05:18 PM
It seems that, in the tests, the flechettes where released from planes that were flying at very low altitude, but at high speed... I wonder if the lethality of the projectiles was due to the speed of the plane rather than to gravity itself (the flechettes would start their flight at high speed already...)

I have re-read the Wikipedia's Lazy Dog entry... and I have doubts.

-The flechette weighs 13 gr. only, around the same some .30-06 Springfield bullets. That's very little for a kinetic weapon...

-The planes that released them in the tests were flying at an altitude of around 23 meters, and were flying at a speed of 400 knots (740.8 km/h = 205.78 m/s).

-The speed of an object released from 23 meters would be around 21 m/s, but that's just vertical speed. The flechettes conserved the horizontal speed of the plane, of around 205.78 m/s. Most of the energy they carried came mostly form the plane's own speed.

-Using the Pythagorean theorem, the speed of the flechette was, discarding air resistance, of around 206.85 m/s...

-That means the flechettes carried a kinetic energy of around 556.23 Joules.

-By comparision, a Winchester .45 ACP cartridge carries packs a kinetic energy of around 483 to 796 Joules, and a 9×19mm Parabellum cartridge of 481 to 588 Joules. 556.23 Joules is still pretty deadly, though...

But my point is, most of the kinetic energy carried by the flechette came from the speed of the plane, not from gravity, so the Lazy Dog tests aren't relevant for our current thought experiment.

I have sought information about WWI kinetic bombardment flechettes, and they were 60 grams of weight, pencil-shaped, and were thrown from a high altitude. That sounds more plausible for our flying freaks...

Brother Oni
2018-01-18, 06:50 PM
Oh, fair enough, I thought it was larger. That means they don't digest 40% of the extra calories. Hmm, perhaps 50% extra calories may be too much for a human to digest without constant grazing.

Even with additional pancreatic enzymes to pre-digest their food, it's very difficult for nutrients to be absorbed through the thick mucus in their intestines - it's most likely digested, it simply doesn't get into their system in the time it passes through their GI tract. I once attended a talk by a CF sufferer on her average day - the enzymes you mentioned was one she always took as she otherwise got extremely bad diarrhoea (she was on something like 28 different medications every day and after a while, she just stopped bothering to take all of them as all those treatments took so long to do, it detracted more from her quality of life than she gained - nebulisers were the worst as they could take up to an hour from setup to finishing washing up afterwards; imagine taking that three times a day).

Back on topic, the highest calorie intakes I know about are for activities in the extreme cold; living in the Antarctic can require from 2750 Calories when working inside buildings to 6500 Calories when manhandling sleds (11.5MJ to 27.2MJ). Royal Marines during Arctic survival training can consume up to 8000 Calories a day (33.5MJ).

While all those extra calories are pretty much all going to thermogenesis rather than aerobic exercise, it does give an upper limit of calorific intake and energy expenditure without turning into a walking blubber ball.

Carl
2018-01-18, 06:57 PM
7500kJ/day is only about 1800 Calories, which is very low for an active individual. I'll have to do more digging on the processing rate of people, however CF sufferers can only eat that many calories because the vast majority of isn't absorbed due to their disease condition and passes through undigested.



Definitely meat and fat rich diets (see my comment to Carl below).



Actually honey is mostly carbohydrates, which isn't that energy dense - you're looking at ~3 Calories (~12.6kJ) per gram. Fat, especially processed, is far more energy dense - rendered pig fat (lard) is ~9 Calories (37.6kJ) per gram.

Honey is a lot more bioavailable though, which would be useful for immediate actions, but for long term diet, a meat and fat heavy diet would be required.
One side effect of such a diet would be long rest periods to digest such an energy rich diet - take a look at the activity cycles of lions and other obligate carnivores as an example.

I suppose this could tie in with a more Mediterranean lifestyle for our flying humanoids, with long afternoon siestas after lunch. :smallbiggrin:

Yes and no.

First my understanding is that outside of late summer/autumn in climates where animals hibernate the fat to meat ratio of wild game is quite poor, this severely curtails the energy density of meat.

Second, meat is fairly energy dense but not anywhere near as volume dense, this limits meal size which limits daily intake.

Third, meat as you noted takes a long time to digest whilst honey being mostly sugars processes faster from what i understand.

That dosen;t mean i'd see bird people as pure honey eater,s just that it would be the most critical part of their diet IMO as it wold provide quick easy fuel. fat risch game meat would probably be more of the "extra food and might well constitute a delicacy the way many meats did historically.


Well, it is an area I am able to make some more detailed analysis in, so I decided to put in some research :) Heavy forest would likely be the best terrain for avoiding them- such a huge wingspan would be a liability in forests, and they'd have very poor visibility. Plains and deserts would be next best. Coasts and mountains are the most in favour of the flyers. I am increasingly seeing them as being ideally like flying Incas, so that may be a cool place to look for ideas. The Incas also venerated the Andean condor, so there is some cool imagery that could be drawn from them. Incas also had very intensive mountainous food production, which brings me on to my next point about energy intake. See below.

From what I was reading, large flyers invariably cannot sustain active flight, so they largely rely on environmental factors- thermals and wind, to maintain and gain altitude. Based on this, the bird people would actually have very low energy needs when gliding, which would be most of their flying, and they would not be able to sustain high energy outputs for long without fatiguing. Gliding basically only requires tilting the wing to adjust to changing wind conditions, and slope soaring and thermals can allow forward propulsion without any additional energy input from the bird. I think with this in mind, they would not need much in the way of additional oxygen capacity, because they are largely gliding and soaring. The altitude of 5,000m is within human physiological limits with some adaptation (people can live up to about 6,000m altitude permanently, and survive up to 8,000m for reasonable periods. Beyond 8,000m is when oxygen deprivation becomes short-term life threatening) Only take off would typically be energy intensive, and usually natural features would help this- even a downward slope of 10 degrees, and a head wind of 5mph is enough for Argentavis to be able to take off easily apparently, so similar conditions wold be needed for our birdpeople. I based the flight speeds off the speeds of actual birds and the big prehistoric bird. Only combat, with the additional manoeuvring needs, would be very energy intensive, and it is possible to eat the amounts of energy I posted above by simply eating more meals per day than is typical for humans. I will go into the foodstuffs themselves below.

In terms of size and morphology, I agree when looking back, I think adding 20% weight and wing span would be reasonable, so more like 80kg with a wingspan of 9m (which s enormous, but still smaller than some pterosaurs). There is no example I know of of a vertebrate from amphibians onwards which has more than 4 limbs. This would be a massive change. The way that bat wings work would be the most plausible- a flap of skin extending from the arm along the torso. This would allow the arms to be normally functioning too. You make a good point on the legs likely still being good for sprinting, I agree this is likely. Long distance endurance would be lacking, but then long distance flying is what they will do best.

The problem is i'm not sure a humanoid non-active flyer could evolve. You said it yourself they're going to be poor diving hunters and their size makes that worse s they're more visible to potential prey. That means wings would be far less useful to them than the arms, so why do they have wings still evolutionary speaking. An active flyer has a lot more ways to benefit from wings than a passive one which makes them more likely to keep their wings rather than ditch them for the tool utility. There's also the collory issue that passive flight is a fairly slow hunting technique. On a bird that works. On a Bird man with a brain it may be a serious issue as the brain uses quite a bit of power. Not as much as active flight, (in fact nowhere near), but enough that it may make passive flight based hunting or long distance foraging unworkable.

Also since we're talking about military applications active flyers are far more useful.

The reason large birds are passive fliers mostly comes down to the aerobic exercise limit, they just can't take in enough oxygen to handle the energy expenditure required. Thats the real headache with bird people.

None of thats to say they won't use passive flight in a lot of situations, and long distance travel will likely be somthing of a combination, short periods of active flight interspaced by long glides. But i can't see them working evolutionary as pure passive fliers.

Haighus
2018-01-18, 07:12 PM
Even with additional pancreatic enzymes to pre-digest their food, it's very difficult for nutrients to be absorbed through the thick mucus in their intestines - it's most likely digested, it simply doesn't get into their system in the time it passes through their GI tract. I once attended a talk by a CF sufferer on her average day - the enzymes you mentioned was one she always took as she otherwise got extremely bad diarrhoea (she was on something like 28 different medications every day and after a while, she just stopped bothering to take all of them as all those treatments took so long to do, it detracted more from her quality of life than she gained - nebulisers were the worst as they could take up to an hour from setup to finishing washing up afterwards; imagine taking that three times a day).

Back on topic, the highest calorie intakes I know about are for activities in the extreme cold; living in the Antarctic can require from 2750 Calories when working inside buildings to 6500 Calories when manhandling sleds (11.5MJ to 27.2MJ). Royal Marines during Arctic survival training can consume up to 8000 Calories a day (33.5MJ).

While all those extra calories are pretty much all going to thermogenesis rather than aerobic exercise, it does give an upper limit of calorific intake and energy expenditure without turning into a walking blubber ball.
CF does have a terrible quality of life in many ways- there is definitely a cost to the increased life expectancy nowadays.
Hmm, I believe artic explorers essentially pull sleds of butter too. Those higher end figures will be operating at a loss though, they won't be able to sustain that for more than a few weeks at most before starving/freezing, and will lose weight over this time. Cyclists in the Tour de France essentially eat optimum dietary intake for maximal short term energy input, and they lose weight and muscle mass over three weeks.

Yes and no.

First my understanding is that outside of late summer/autumn in climates where animals hibernate the fat to meat ratio of wild game is quite poor, this severely curtails the energy density of meat.

Second, meat is fairly energy dense but not anywhere near as volume dense, this limits meal size which limits daily intake.

Third, meat as you noted takes a long time to digest whilst honey being mostly sugars processes faster from what i understand.

That dosen;t mean i'd see bird people as pure honey eater,s just that it would be the most critical part of their diet IMO as it wold provide quick easy fuel. fat risch game meat would probably be more of the "extra food and might well constitute a delicacy the way many meats did historically.



The problem is i'm not sure a humanoid non-active flyer could evolve. You said it yourself they're going to be poor diving hunters and their size makes that worse s they're more visible to potential prey. That means wings would be far less useful to them than the arms, so why do they have wings still evolutionary speaking. An active flyer has a lot more ways to benefit from wings than a passive one which makes them more likely to keep their wings rather than ditch them for the tool utility. There's also the collory issue that passive flight is a fairly slow hunting technique. On a bird that works. On a Bird man with a brain it may be a serious issue as the brain uses quite a bit of power. Not as much as active flight, (in fact nowhere near), but enough that it may make passive flight based hunting or long distance foraging unworkable.

Also since we're talking about military applications active flyers are far more useful.

The reason large birds are passive fliers mostly comes down to the aerobic exercise limit, they just can't take in enough oxygen to handle the energy expenditure required. Thats the real headache with bird people.

None of thats to say they won't use passive flight in a lot of situations, and long distance travel will likely be somthing of a combination, short periods of active flight interspaced by long glides. But i can't see them working evolutionary as pure passive fliers.

No flyer of this kind could evolve full stop anyway, so I'm handwaving that to get close to a model that works for the OP's specifications. Theoretically, what we have so far could be genetically engineered, and function. It is one of those examples that is useful as a fully formed entity, but couldn't get there through an evolutionary path.

There are no known examples of active flyers this large, that could sustain active flight, so I went with passive overall, and short term active. This also gives the flyers a palpable weakness, that stops them being as dominating as they could otherwise be. I reckon the passive flying for large birds also comes down to diminishing returns on increased muscle mass against increasing weight- the huge bird I linked to above was only capable of outputting a third of the power it needed to sustain active flight. That isn't just an aerobic limit, but a literal muscle power limit too.

I don't think the human brain will be a huge problem*, because humans are already adapted to consuming that much energy. It does mean a passive bird person is going to use more energy than a passive bird with a smaller brain, but the advantages of being smart is probably going to outweigh that.

*As a note related to this, humans have a degree of obligate carbohydrate consumption- they die without any glucose at all (brain and red blood cells being the most important requirers of glucose). This means they will need to eat some carbs ideally, although protein can substitute for carbs (it isn't ideal for this).

rs2excelsior
2018-01-19, 01:03 AM
I agree about winged humanoids not really being practically able to evolve. The skeletal structure alone to accommodate a double-shoulder would be...implausible to say the least. And yet, they're a fantasy trope I decided to run with, biology be damned. I mean, you can have house-sized, double-shouldered dragons flying around without a care in the world.

Regarding the energy density of meat, they will definitely have domesticated animals, so the fat levels of wild game isn't really an issue. They can raise animals which are fattier to suit their needs.

Also, for anyone who might be interested to see where this goes, here (http://www.giantitp.com/forums/showthread.php?548688-Reign-of-the-Corven-(Very)-WIP) is where I'm developing my ideas further. It's very much a rough draft--much of the detail is still open ended and I haven't posted everything I have nailed down yet--but I'm planning on continuing to flesh things out as I go. [/shameless plug] :smallbiggrin:

Carl
2018-01-19, 04:24 AM
CF does have a terrible quality of life in many ways- there is definitely a cost to the increased life expectancy nowadays.
Hmm, I believe artic explorers essentially pull sleds of butter too. Those higher end figures will be operating at a loss though, they won't be able to sustain that for more than a few weeks at most before starving/freezing, and will lose weight over this time. Cyclists in the Tour de France essentially eat optimum dietary intake for maximal short term energy input, and they lose weight and muscle mass over three weeks.


No flyer of this kind could evolve full stop anyway, so I'm handwaving that to get close to a model that works for the OP's specifications. Theoretically, what we have so far could be genetically engineered, and function. It is one of those examples that is useful as a fully formed entity, but couldn't get there through an evolutionary path.

There are no known examples of active flyers this large, that could sustain active flight, so I went with passive overall, and short term active. This also gives the flyers a palpable weakness, that stops them being as dominating as they could otherwise be. I reckon the passive flying for large birds also comes down to diminishing returns on increased muscle mass against increasing weight- the huge bird I linked to above was only capable of outputting a third of the power it needed to sustain active flight. That isn't just an aerobic limit, but a literal muscle power limit too.

I don't think the human brain will be a huge problem*, because humans are already adapted to consuming that much energy. It does mean a passive bird person is going to use more energy than a passive bird with a smaller brain, but the advantages of being smart is probably going to outweigh that.

*As a note related to this, humans have a degree of obligate carbohydrate consumption- they die without any glucose at all (brain and red blood cells being the most important requirers of glucose). This means they will need to eat some carbs ideally, although protein can substitute for carbs (it isn't ideal for this).

It's very unlikely somthing like this could evolve, but if it's physically possibble the it could actually evolve. As i pointed out previously evolution is full of creatures where limbs joined or separated limbs at some point in their evolution so having arms seperate out from the wings is a long way from impossible. The odds against it are fairly long, but it's not outright impossible.

The problem i have with passive flight is thats it's simply incompatible with intelligance when you get right down to it. Passive flight is an evolutionary choice made because in general large bird active flight is simply more energy intensive than any naturally occurring environment, (on earth, it's not set in stone that a sufficiently energy rich environment could not exist, though it would likely have many other implications), can provide for. Since they can't get enough food they don't evolve, (or lose if they started out smaller), any active flight features, and as nature continues to select for passive flight this reinforces itself emphasising reduction of any features that make it less effective. And thats where the high energy cost of a brain becomes an issue. Bigger brains with bigger energy costs harm passive flight as a successful survival strategy so it actually would result in selection against intelligance without another competing factor. And the limited food selection afforded by passive flight would tend to select against tool use, (though thats not definitive, it merely reduces the range of avenues by which it could develop), and especially the large ranges involved would tend to limit population density which would select against communication.

The more i think about it the most sensible evolutionary path is soe smaller bird that preys on a primary pry that needs more manipulation than beak and talons alone would provide result in use of the wing as a pinning tool. This could then develop into wingtip ips of a different design, which could allow for other wing uses which could develop wing tip hands and eventual splitting of the limb. If they're allready communal this could reasonably be expected to create a positive benefit of more intelligance and that would likely lead to steady increases in absolute size to support the more advanced brain structure.

I''m not sure what the point of the * comment was though. Yes humans need carb's, duh. That doesn't make honey invalid since it's >80% carbs by weight, in fact depending on the honey it's between 15% and 35% pure glucose by weight, (most of the rest is fructose, another sugar type carbohydrate directly absorbed by the blood). That said some thinking on the evolution side makes me think Nuts, sweet fruits, and of course meats would be more important secondary foods alongside than i'd first assumed. (In the same way most humans eat a lot of meats, but still eat fruit and veg alongside to varying degrees). Having a wider array of food sources is very useful to a communal species because it helps larger groups survive from the same hunting/foraging range area. If there's suitable large scale honey producing insects aroudn it's possible, perhaps even probable that the nectar producing plants would be larger which might leads to Nectar being a viable thing to gather direct as well. Would make for interesting agriculture.

Carl
2018-01-19, 04:28 AM
I agree about winged humanoids not really being practically able to evolve. The skeletal structure alone to accommodate a double-shoulder would be...implausible to say the least. And yet, they're a fantasy trope I decided to run with, biology be damned. I mean, you can have house-sized, double-shouldered dragons flying around without a care in the world.

Regarding the energy density of meat, they will definitely have domesticated animals, so the fat levels of wild game isn't really an issue. They can raise animals which are fattier to suit their needs.

Also, for anyone who might be interested to see where this goes, here (http://www.giantitp.com/forums/showthread.php?548688-Reign-of-the-Corven-(Very)-WIP) is where I'm developing my ideas further. It's very much a rough draft--much of the detail is still open ended and I haven't posted everything I have nailed down yet--but I'm planning on continuing to flesh things out as I go. [/shameless plug] :smallbiggrin:

Forgot to quote this above because reasons. DoH!

Having domesticated animals that can be as fatty as they like won't make it a primary food source. meat is such a big source for humans because thanks to our hunter gatherer history we went through a sage where wild meat was a major source. In short both in terms of our physiology and subtle hand me down psychological stuff where inclined towards high meat intake. But bird people won't necessarily have that. Anyway gotta go got some more comments on ways a passive flight model could work for bird people but need to go out and then sleep so probably 12 hours or so from now at the earliest.

Brother Oni
2018-01-19, 08:00 AM
That dosen;t mean i'd see bird people as pure honey eater,s just that it would be the most critical part of their diet IMO as it wold provide quick easy fuel. fat risch game meat would probably be more of the "extra food and might well constitute a delicacy the way many meats did historically.

I disagree - while I concede that volume for volume, honey is more energy dense (~1.4 g/ml compared to 0.6-0.9 g/ml for meat) trying to keep down that much honey would both be difficult and poor for glycemic control (take a look at sugar-crashed children after Halloween).

I don't disagree that honey would be good for a quick fix and indeed a meal of mead could form part of their cultural pre-battle preparations, I don't think subsisting primarily from honey is efficient nor good for long term health.

Bear in mind that animals with a heavy meat/fat heavy diet tend to sleep or nap more, thus lowering their daily metabolic requirements. Daily activity and hence energy expenditure isn't spread evenly throughout the day, so quick energy spikes from honey may not be necessary. This also isn't counting the additional water intake due to the high glucose/fructose content of honey as it's turned into glycogen as an intermediary before storage as fat.


I''m not sure what the point of the * comment was though. Yes humans need carb's, duh. That doesn't make honey invalid since it's >80% carbs by weight, in fact depending on the honey it's between 15% and 35% pure glucose by weight, (most of the rest is fructose, another sugar type carbohydrate directly absorbed by the blood). That said some thinking on the evolution side makes me think Nuts, sweet fruits, and of course meats would be more important secondary foods alongside than i'd first assumed. (In the same way most humans eat a lot of meats, but still eat fruit and veg alongside to varying degrees).

Actually while we have evolved to primarily burn carbohydrates, we technically don't need it as both protein and fats can be burnt to provide energy (fats undergo lipolysis and the constituents are chucked into the Kreb's Cycle, proteins can either be processed into pyruvate like fats or just burnt as ketone bodies).

There's a growing hypothesis that humans brains developed because we invented cooking, thus raising the bioavailability of calories from the foodstuffs we ate and providing the additional calories required.

Haighus
2018-01-19, 08:46 AM
It's very unlikely somthing like this could evolve, but if it's physically possibble the it could actually evolve. As i pointed out previously evolution is full of creatures where limbs joined or separated limbs at some point in their evolution so having arms seperate out from the wings is a long way from impossible. The odds against it are fairly long, but it's not outright impossible.

The problem i have with passive flight is thats it's simply incompatible with intelligance when you get right down to it. Passive flight is an evolutionary choice made because in general large bird active flight is simply more energy intensive than any naturally occurring environment, (on earth, it's not set in stone that a sufficiently energy rich environment could not exist, though it would likely have many other implications), can provide for. Since they can't get enough food they don't evolve, (or lose if they started out smaller), any active flight features, and as nature continues to select for passive flight this reinforces itself emphasising reduction of any features that make it less effective. And thats where the high energy cost of a brain becomes an issue. Bigger brains with bigger energy costs harm passive flight as a successful survival strategy so it actually would result in selection against intelligance without another competing factor. And the limited food selection afforded by passive flight would tend to select against tool use, (though thats not definitive, it merely reduces the range of avenues by which it could develop), and especially the large ranges involved would tend to limit population density which would select against communication.

The more i think about it the most sensible evolutionary path is soe smaller bird that preys on a primary pry that needs more manipulation than beak and talons alone would provide result in use of the wing as a pinning tool. This could then develop into wingtip ips of a different design, which could allow for other wing uses which could develop wing tip hands and eventual splitting of the limb. If they're allready communal this could reasonably be expected to create a positive benefit of more intelligance and that would likely lead to steady increases in absolute size to support the more advanced brain structure.

I''m not sure what the point of the * comment was though. Yes humans need carb's, duh. That doesn't make honey invalid since it's >80% carbs by weight, in fact depending on the honey it's between 15% and 35% pure glucose by weight, (most of the rest is fructose, another sugar type carbohydrate directly absorbed by the blood). That said some thinking on the evolution side makes me think Nuts, sweet fruits, and of course meats would be more important secondary foods alongside than i'd first assumed. (In the same way most humans eat a lot of meats, but still eat fruit and veg alongside to varying degrees). Having a wider array of food sources is very useful to a communal species because it helps larger groups survive from the same hunting/foraging range area. If there's suitable large scale honey producing insects aroudn it's possible, perhaps even probable that the nectar producing plants would be larger which might leads to Nectar being a viable thing to gather direct as well. Would make for interesting agriculture.
I agree with all your points on evolution and intelligence, although I still do not know of any examples of splitting limbs in vertebrates- if you could give some that would be excellent. It could happen, but I think the timespans involved could be enormous. I essentially handwaved it, because it would almost certainly not generate the type of flyer the OP was looking for, but rather something more like a corvid in size- the "angry pixie" mentioned by Galloglaich. Physically possible doesn't necessarily mean evolutionarily plausible, because open there ar insumountable hurdles to reaching a finished, useful evolutionary product.

The * bit was more just personal musings on the topic, than replying to any specific point. I was basically thinking an omnivorous diet would be best. It certainly doesn't rule out honey. I agree with Brother Oni below that honey would be best as a supplement for periods of intensive activity. If we look at athletes for guides on energy intake, complex carbohydrates like starch are better for sustained physical activity, with easily bioavailable sugars used as a top-up during activity. So I think eating rice and potatoes and wheat-based products as general calories, with honey best used as a boost everytime they return from an attack sortie to re-arm and recover energy, or even taking small pots to use when on the wing over long journeys. Would still require a massive honey industry if using for rapid recovery in warfare. This would give them some interesting logistical considerations.

I disagree - while I concede that volume for volume, honey is more energy dense (~1.4 g/ml compared to 0.6-0.9 g/ml for meat) trying to keep down that much honey would both be difficult and poor for glycemic control (take a look at sugar-crashed children after Halloween).

I don't disagree that honey would be good for a quick fix and indeed a meal of mead could form part of their cultural pre-battle preparations, I don't think subsisting primarily from honey is efficient nor good for long term health.

Bear in mind that animals with a heavy meat/fat heavy diet tend to sleep or nap more, thus lowering their daily metabolic requirements. Daily activity and hence energy expenditure isn't spread evenly throughout the day, so quick energy spikes from honey may not be necessary. This also isn't counting the additional water intake due to the high glucose/fructose content of honey as it's turned into glycogen as an intermediary before storage as fat.



Actually while we have evolved to primarily burn carbohydrates, we technically don't need it as both protein and fats can be burnt to provide energy (fats undergo lipolysis and the constituents are chucked into the Kreb's Cycle, proteins can either be processed into pyruvate like fats or just burnt as ketone bodies).

There's a growing hypothesis that humans brains developed because we invented cooking, thus raising the bioavailability of calories from the foodstuffs we ate and providing the additional calories required.
Eating a huge amount of honey can also result in diarrhoea, if you eat beyond the level of sugar that can be absorbed by the gut.

We don't need to eat carbs, although we do need them to survive- fat alone is not sufficient because only the glycerol in triglycerides can be turned into glucose. Fats can be burned directly by most cells as you mention, but the brain can only utilise ketones from fats, and needs at least 30% glucose to function alongside the ketones. Apparently the ability to utilise ketones is unique to human brains as an adaption for having such a large brain, but I have never seen proper evidence supporting this theory. Interesting if true. Red blood cells can only use glucose. This is why overweight diabetics can die of ketoacidosis in hypoglycaemia. Protein can be turned into glucose (well, some amino acids can, not all), but comes with byproducts which make it less desirable than eating some carbs to supplement it. In a starvation state, once glycogen reserves are used up, protein from tissues is used to create glucose for obligate tissues (red blood cells and brain), whilst fats are used to fuel everything else until they are exhausted. Protein is generally the limiting energy supply in starvation, and people who are adequately hydrated but no food intake, loss of protein is what kills them (apparently usually through the diaphragm becoming too weak to sustain breathing and rupturing).

What this means is that a human can live off a low-carbs, high-protein diet, but it isn't ideal, and some carbs are needed to be healthy. Inuits have adapted to a high protein and fat, exclusively meat diet, but they are unusual in this regard, and it doesn't work well for unadapted people. The bird people could be similarly adapted, but I suspect omnivorous feeding habits to be the best for their function.

The cooking thing is awesome. I can see the logic behind the theory.

Tobtor
2018-01-20, 04:32 AM
What this means is that a human can live off a low-carbs, high-protein diet, but it isn't ideal, and some carbs are needed to be healthy. Inuits have adapted to a high protein and fat, exclusively meat diet, but they are unusual in this regard, and it doesn't work well for unadapted people. The bird people could be similarly adapted, but I suspect omnivorous feeding habits to be the best for their function.


As you say, Inuit live almost exclusively on meat. So adaptation is relatively simple (compared to the rest of the bird ecology anyway).

In contrast to above suggestions about mountains, I think some ocean-culture is better suited for the bird people. The sea simply provides so much nutrient rich sources of food that it would greatly help the bird people for their higher energy consumption. Also if the were based on island(s) of the coast, they would have a secure homebase, which could explain why the humans does not focus solely on the bird-peoples home territory: Attacking flying creatures living on an Island would be immensely difficult (as hitting ships from above with "fire-bombs" would be devastating for any attacker).

So you could have the bird people living on (mountainous) islands of the coast, who have colonised/occupied the coast, and made coastal humans fight for them. Then you could have more inland humans fighting against this "bird"-empire.

For food you could supplement plant food, with rich marine sources (migratory whales for high fat intake?, otherwise fish, seals, shellfish, eggs of seabirds and so on is easily gathered/hunted). This can easily be combined with "terraced agriculture", used for stable food (and possible fermenting to alcohol or similar for easier consumption of high energy intake).

Carl
2018-01-20, 05:18 AM
I disagree - while I concede that volume for volume, honey is more energy dense (~1.4 g/ml compared to 0.6-0.9 g/ml for meat) trying to keep down that much honey would both be difficult and poor for glycemic control (take a look at sugar-crashed children after Halloween).

I don't disagree that honey would be good for a quick fix and indeed a meal of mead could form part of their cultural pre-battle preparations, I don't think subsisting primarily from honey is efficient nor good for long term health.

Bear in mind that animals with a heavy meat/fat heavy diet tend to sleep or nap more, thus lowering their daily metabolic requirements. Daily activity and hence energy expenditure isn't spread evenly throughout the day, so quick energy spikes from honey may not be necessary. This also isn't counting the additional water intake due to the high glucose/fructose content of honey as it's turned into glycogen as an intermediary before storage as fat.



Actually while we have evolved to primarily burn carbohydrates, we technically don't need it as both protein and fats can be burnt to provide energy (fats undergo lipolysis and the constituents are chucked into the Kreb's Cycle, proteins can either be processed into pyruvate like fats or just burnt as ketone bodies).

There's a growing hypothesis that humans brains developed because we invented cooking, thus raising the bioavailability of calories from the foodstuffs we ate and providing the additional calories required.

My main thought with the honey was basically this:

Digestive systems vary from species to species so a digestive system optimized for whatever's best makes sense. That makes the real limitation the size of the stomach sacks, (and the volume pressure this puts on other organs from an evolutionary standpoint). For active flight 1 liter of honey will power just half an hours flight, so they'd need to consume several litters before a multi-hour flight session and then let it digest.With that kind of feeding requirements meats have the problem that a good meal of slowly digested meat might only let them fly for an hour or two each day. How valuable is that going to be evolutionary? You could well see evolution start to select against flight in those circumstances unless flight was absolutely vital in some fashion.

The cooking angle is interesting because there are a number of cooking techniques that can absolutely enhance nutrition of course.


I agree with all your points on evolution and intelligence, although I still do not know of any examples of splitting limbs in vertebrates- if you could give some that would be excellent. It could happen, but I think the timespans involved could be enormous. I essentially handwaved it, because it would almost certainly not generate the type of flyer the OP was looking for, but rather something more like a corvid in size- the "angry pixie" mentioned by Galloglaich. Physically possible doesn't necessarily mean evolutionarily plausible, because open there ar insumountable hurdles to reaching a finished, useful evolutionary product.

The * bit was more just personal musings on the topic, than replying to any specific point. I was basically thinking an omnivorous diet would be best. It certainly doesn't rule out honey. I agree with Brother Oni below that honey would be best as a supplement for periods of intensive activity. If we look at athletes for guides on energy intake, complex carbohydrates like starch are better for sustained physical activity, with easily bioavailable sugars used as a top-up during activity. So I think eating rice and potatoes and wheat-based products as general calories, with honey best used as a boost everytime they return from an attack sortie to re-arm and recover energy, or even taking small pots to use when on the wing over long journeys. Would still require a massive honey industry if using for rapid recovery in warfare. This would give them some interesting logistical considerations.

Eating a huge amount of honey can also result in diarrhoea, if you eat beyond the level of sugar that can be absorbed by the gut.

We don't need to eat carbs, although we do need them to survive- fat alone is not sufficient because only the glycerol in triglycerides can be turned into glucose. Fats can be burned directly by most cells as you mention, but the brain can only utilise ketones from fats, and needs at least 30% glucose to function alongside the ketones. Apparently the ability to utilise ketones is unique to human brains as an adaption for having such a large brain, but I have never seen proper evidence supporting this theory. Interesting if true. Red blood cells can only use glucose. This is why overweight diabetics can die of ketoacidosis in hypoglycaemia. Protein can be turned into glucose (well, some amino acids can, not all), but comes with byproducts which make it less desirable than eating some carbs to supplement it. In a starvation state, once glycogen reserves are used up, protein from tissues is used to create glucose for obligate tissues (red blood cells and brain), whilst fats are used to fuel everything else until they are exhausted. Protein is generally the limiting energy supply in starvation, and people who are adequately hydrated but no food intake, loss of protein is what kills them (apparently usually through the diaphragm becoming too weak to sustain breathing and rupturing).

What this means is that a human can live off a low-carbs, high-protein diet, but it isn't ideal, and some carbs are needed to be healthy. Inuits have adapted to a high protein and fat, exclusively meat diet, but they are unusual in this regard, and it doesn't work well for unadapted people. The bird people could be similarly adapted, but I suspect omnivorous feeding habits to be the best for their function.

The cooking thing is awesome. I can see the logic behind the theory.

Actually physically possibble pretty much does mean evolutionarily possibble. The catch is some things require such far out there factors that as a matter of practise the odds are exceedingly low. The thing is in fiction the odds can be as long as you want, and still come up trumps.


I did say i'd talk about how passive flight might work out. Whilst it's just completely incompatible with what the OP wanted the easiest way would be to make the passive flight pure transport and have them do their hunting and gathering on the ground. I'm not sure what evolutionary pressures precisely could produce such a need for efficient long distance transit but it would also probably look closest to the traditional "winged human" archetype as the heavy groundside work would tend to push for a more human limb set whilst whatever evolutionary pressure needs that excellent long distance efficient movement capability would tend to push the desire to keep the wings, whilst the poor hunting and gathering efficiency of passive flight pushes the need to do hunting and gathering ground side with a body better adapted to it.

Tobtor
2018-01-20, 06:23 AM
I did say i'd talk about how passive flight might work out. Whilst it's just completely incompatible with what the OP wanted the easiest way would be to make the passive flight pure transport and have them do their hunting and gathering on the ground. I'm not sure what evolutionary pressures precisely could produce such a need for efficient long distance transit but it would also probably look closest to the traditional "winged human" archetype as the heavy groundside work would tend to push for a more human limb set whilst whatever evolutionary pressure needs that excellent long distance efficient movement capability would tend to push the desire to keep the wings, whilst the poor hunting and gathering efficiency of passive flight pushes the need to do hunting and gathering ground side with a body better adapted to it.

Transport between different "living" quarters could be a reason. Sort of like with of the Draco (https://en.wikipedia.org/wiki/Draco_(genus)) lizards, which have developed "wing"-like tisue independent of their 4 limps (they use the "wings" to move from tree to tree without going to the bottom of the forrest):

Draco Volans https://upload.wikimedia.org/wikipedia/commons/9/95/Draco_volans_01.JPG

These liazards doesn't really have flight (just gliding), but perhaps we could imagine our "bird" people stating out this way and then developing more "flight" ability? Anyway Draco lizards are the closest I have seen to a vertibrae with four limps +"wings".

They could have developed the early flight to move between mountains (or islands with sea cliffs), as a way of moving about. The gradually improved it with more powered flight to use in hunting/fighting?

Brother Oni
2018-01-20, 06:52 AM
My main thought with the honey was basically this:

Digestive systems vary from species to species so a digestive system optimized for whatever's best makes sense. That makes the real limitation the size of the stomach sacks, (and the volume pressure this puts on other organs from an evolutionary standpoint). For active flight 1 liter of honey will power just half an hours flight, so they'd need to consume several litters before a multi-hour flight session and then let it digest.With that kind of feeding requirements meats have the problem that a good meal of slowly digested meat might only let them fly for an hour or two each day. How valuable is that going to be evolutionary? You could well see evolution start to select against flight in those circumstances unless flight was absolutely vital in some fashion.

The oral LD50 for fructose in rats is 4g/kg bodyweight, so ignoring species differences for now and a 50kg human, they'd need 200g fructose to kill half the population.

Assuming that honey is 38.2% fructose, you'd need 524g of honey, which adjusted for density is 374mL or just over a third of a stomach's worth.

Even if you account for adaptations, the fructose processing of the bird folk will have to be over 3 times more efficient than regular humans and given that we're essentially former tree dwelling mammals who used to gorge ourselves on fruit at the end of every summer, our fructose processing rate isn't too shabby.

I'm very much agreeing with Haigus in that these bird folk primarily use passive flight with only minimal active flight activities; regardless I think we're just going to have to disagree on this subject - we've presented both sides of the argument, so rs2excelsior has a choice of options (it could be even both, with a highly physically active warrior caste preferring a high honey diet and the more sedentary castes preferring meat and fat).

Carl
2018-01-20, 08:35 AM
The oral LD50 for fructose in rats is 4g/kg bodyweight, so ignoring species differences for now and a 50kg human, they'd need 200g fructose to kill half the population.

Assuming that honey is 38.2% fructose, you'd need 524g of honey, which adjusted for density is 374mL or just over a third of a stomach's worth.

Even if you account for adaptations, the fructose processing of the bird folk will have to be over 3 times more efficient than regular humans and given that we're essentially former tree dwelling mammals who used to gorge ourselves on fruit at the end of every summer, our fructose processing rate isn't too shabby.

I'm very much agreeing with Haigus in that these bird folk primarily use passive flight with only minimal active flight activities; regardless I think we're just going to have to disagree on this subject - we've presented both sides of the argument, so rs2excelsior has a choice of options (it could be even both, with a highly physically active warrior caste preferring a high honey diet and the more sedentary castes preferring meat and fat).


I agree we've more or less reached the limit but you got me curious with the stuff on fructose and the irony of what i found is doubly worth sharing. According to the wiki page for fructose one of the most frusose dense (in terms of fraction of total sugars and carbs), fruits is the fig. Something several species of birds will happily subsist entirely on whilst they're abundant. I figured there was probably something in nature that subsisted on a high fructose diet, but thats a little too perfect :smalltongue:.

Anyway nice having the discussion, i love getting into little details like this.


Probably going to have a question of my own in the near future. Off to do some research though see if a quick search turns anything up.

Mr Beer
2018-01-20, 03:20 PM
I think it's reasonable that bird-people can cope with a diet that would be terrible for a human because adaptation is a thing. A plausible bird-people would be consuming a lot of calorie-dense food, that's the important take out.

Re. the cooking thing, I read a book where the author claimed cooking was likely core to humanity, that we had been cooking for a very long time, as in millions of years.

He went into a lot of detail about the physiological differences between humans and chimps and how we are adapted to cooked food and they are adapted to raw food e.g. mouth size, jaw strength, size of intestines etc.

He also spent time talking about how raw food diets are basically terrible for humans. Allegedly women who go on a raw food diet tend to stop menstruating. That's with access to modern supermarkets and an effectively unlimited food budget. They're not having to go and physically hunt down food.

Mike_G
2018-01-20, 04:23 PM
Cooked food is a lot easier to digest, and cooking makes a lot of food that would be inedible edible.

So it probably did have a huge effect on our evolution and allowed us to get a lot more benefit from the food we could get our hands on.

Galloglaich
2018-01-21, 01:57 PM
The honey bee thing has some kind of historical context;

Slavs in the period roughly 8th-13th Century (and maybe later) used to incorporate bee hives in the defensive rings (usually made of a combination of stones and logs) around their villages. This served two purposes, it helped in defense and as an early-warning system, and it provided a substantial part of their industry. Honey for food and wax for candles and other stuff. Wax in particular remained a very important trade commodity in the region for centuries.

The whole thing also calls into mind honey harvesting in the Himalayas

http://www.ancient-origins.net/sites/default/files/field/image/collecting-honey-nepal.jpg

In general on the military level it seems like you end up with a trade off between a kind of 'flying light cavalry' - phenomenally mobile but also highly vulnerable and almost incapable (more so than any cavalry) of combat in close quarters unless the target is already fleeing or almost defenseless.

If you are sticking to anything like real-world physics I imagine their huge wingspans would make them even more vulnerable and the wings would be one of the best target for ground forces. I was thinking something like bolas would be particularly dangerous for these flying creatures.

Incorporating bolas or chain shot ammunition at various levels might be a particularly effective way for experienced ground based defenders to fight off attacking bird men.

On the simplest level, either hand thrown or stick-assisted thrown bolas could possibly entangle low-flying birdmen. I assume they would have to fly low to hit anyone with a javelin. You might also be able to incorporate something like a bola into a rocket or a 'hand mortars' grenade - in the latter it would be relatively harmless to people on the ground in case of a misfire, say two or three pebbles with a very long cord connecting them which is safer than some kind of flammable or exploding shrapnel based projectile- but quite dangerous for a flying beast or bird man.

On a more prepared position, like a gun on a war-wagon, you could have stuff like chain shot, but use longer chains.

https://upload.wikimedia.org/wikipedia/commons/thumb/6/68/Boulet_ram%C3%A9_mg_5230.jpg/480px-Boulet_ram%C3%A9_mg_5230.jpg

https://en.wikipedia.org/wiki/Chain-shot

This was used to destroy rigging on ships but I suspect could also be quite effective against winged creatures.


So all this sets up a kind of chess game.

Bird men would be hard-pressed to attack well prepared troops for example in forts, on warships, or traveling with war-wagons. Or carrying 'hand mortar' type grenade launchers. These kinds of targets would only really be vulnerable to high-altitude attacks like from steel darts and incendiary devices as has been discussed, but they could protect themselves from that by creating large volumes of smoke.

https://www.bismarck-class.dk/tirpitz/gallery/pictures/galltiroperparavane/galltiroperparavane01.jpg

Less prepared but still armed targets, like cavalry armed with bows or crossbows, merchant ships armed with light cannon, and small lightly defended forts could be vulnerable.

Unprepared 'soft' targets, civilians lacking in serious military weapons - might still try to defend themselves with bolas or darts, but would be much more vulnerable to attack.

The birdmen would also be able to provide any allies with excellent C3I and should be able to often observe without themselves being seen (especially if they could use camoflage)

Presumably, ground based forces could try to launch dangerous expeditions up into mountain fastnesses to attack and harass birdmen settlements. I don't know about the physiology / physics of birdmen but apparently condors could fly as high as 15,000 feet, astoundingly, so perhaps the birdmen would have some huge mountain they lived on. Could make for an interesting expedition!

G

Max_Killjoy
2018-01-21, 02:21 PM
Presumably, ground based forces could try to launch dangerous expeditions up into mountain fastnesses to attack and harass birdmen settlements. I don't know about the physiology / physics of birdmen but apparently condors could fly as high as 15,000 feet, astoundingly, so perhaps the birdmen would have some huge mountain they lived on. Could make for an interesting expedition!


There are cranes and geese that migrate over the Himalayas, and raptors that prey on them. There are anecdotal reports of bar-headed geese flying above the highest peaks.

Varying by species, birds have a remarkably more efficient respiratory system than mammals -- they're effectively pushing air in one direction through their lungs while they breath in and out, if I understand the physiology correctly, so that there are no pauses and the entire lung is refreshed continually, instead of getting stale air built up at the "bottom" of each breath that has to be pushed out.

Martin Greywolf
2018-01-21, 03:41 PM
The honey bee thing has some kind of historical context;

Slavs in the period roughly 8th-13th Century (and maybe later) used to incorporate bee hives in the defensive rings (usually made of a combination of stones and logs) around their villages. This served two purposes, it helped in defense and as an early-warning system, and it provided a substantial part of their industry.

Depends on the Slavs we're talking about, those in kingdoms that adopted knightly army systems (Poland, Hungary, Croatia) stopped using this, most likely because villages weren't meant to be defensible any more, since castles overtook that role.

That said, bees and wasps were still used militarily on occasion (by throwing or using catapults), I recall at least one incident from Hussite wars, and a lot of details like this simply weren't recorded.



If you are sticking to anything like real-world physics I imagine their huge wingspans would make them even more vulnerable and the wings would be one of the best target for ground forces. I was thinking something like bolas would be particularly dangerous for these flying creatures.

Incorporating bolas or chain shot ammunition at various levels might be a particularly effective way for experienced ground based defenders to fight off attacking bird men.

On the simplest level, either hand thrown or stick-assisted thrown bolas could possibly entangle low-flying birdmen. I assume they would have to fly low to hit anyone with a javelin. You might also be able to incorporate something like a bola into a rocket or a 'hand mortars' grenade - in the latter it would be relatively harmless to people on the ground in case of a misfire, say two or three pebbles with a very long cord connecting them which is safer than some kind of flammable or exploding shrapnel based projectile- but quite dangerous for a flying beast or bird man.

On a more prepared position, like a gun on a war-wagon, you could have stuff like chain shot, but use longer chains.

This was used to destroy rigging on ships but I suspect could also be quite effective against winged creatures.

I don't think these would be that effective - if the bird people are in the short range, they need to rethink their strategy, if they are at longer ranges and the bolas can even reach them, you have the issue of aiming. Unless you managed to put out a tremendous RoF and walk your shots, hitting a moving, flying target whose position can vary in three directions is a tall order. If you have cannon, air bursting a shrapnel shot is probably your best option, but making that effective with, say, Napoleonic artillery is not easy.

What I think is a better option, especially with lower tech levels, is something like trick shooting - these avian menaces don't wear armor, so even a relatively light bow will do, and that means you can fill the air with a LOT of arrows, if you have good supply of those. Accuracy by volume, if you will. Something like Chinese cho ko nu is a pretty good fit.



Bird men would be hard-pressed to attack well prepared troops for example in forts, on warships, or traveling with war-wagons. Or carrying 'hand mortar' type grenade launchers. These kinds of targets would only really be vulnerable to high-altitude attacks like from steel darts and incendiary devices as has been discussed, but they could protect themselves from that by creating large volumes of smoke.


I'm not sure how well and how long you'd be able to maintain a smoke screen with pre-industrial chemistry.

One thing I can see happening if a change to how fortresses are designed - the more important ones would almost certainly have steel slanted roofs with sandbags on them. That would bring the need for stronger walls, but with sand slowing the darts down, they could well be impervious to anything the birdmen can carry short of explosives. And even those would be made a lot less effective by the sand, as shown by some of the western front forts in WW1.

Mike_G
2018-01-21, 04:24 PM
I don't think these would be that effective - if the bird people are in the short range, they need to rethink their strategy, if they are at longer ranges and the bolas can even reach them, you have the issue of aiming. Unless you managed to put out a tremendous RoF and walk your shots, hitting a moving, flying target whose position can vary in three directions is a tall order.
.

If these fliers are anywhere near human sized, they will not be jinking around in flight. If they have 20 foot wingspans and fly at a slow glide, which is the only realistic way to look at this short of magic or lethal doses of Handwavium, then they won't be all that hard to hit.

I don't think they'd be all that fearsome in massed battles, until they get scary stuff to drop. Like explosives.

They'd be very very effective in raiding supply lines or doing sabotage and commando stuff deep inside your territory. Think of how much of your food supply is vulnerable in fields and barns. How vulnerable your important officials might be to an airborne commando kidnapping or assassination strike. How easy it would be to torch your cavalry stables or stampede your herds by dropping bad things on them. You can't put roofs and volley guns on everything.

Just the cost to the groundlings of having to spend resources on all that air defense will take a lot away from what they can put toward other stuff, like putting together expeditions to attack the birdmen homeland.

In short, I think they'd be better used for strategic bombing and commando raids. More like heavy bombers and paratroops than divebombers or CAS.

Martin Greywolf
2018-01-22, 04:32 AM
If these fliers are anywhere near human sized, they will not be jinking around in flight. If they have 20 foot wingspans and fly at a slow glide, which is the only realistic way to look at this short of magic or lethal doses of Handwavium, then they won't be all that hard to hit.


They don't have to dodge. Analogous situation would be hitting a WW1 airplane with a rifle - if you happen to aim correctly, the airplane doesn't have time to dodge, simply because a bullet is so much faster, the problem is that you need to know where to point the rifle in the first place.

If our birdman is making slow turns as he flies, you won't be able to even walk your shots all that well, and walking shots is only possible with a bow or light crossbow anyway, not a ballista. Add to that that you're shooting at your far range, and it becomes kinda like sniping - to hit the target, you need to point your weapon at a point that's not anywhere near the target. With ground targets, you only need to correct your orientation and distance, but flyers add altitude to the equation, and that gets messy fast.

As an example, there are those clay pigeons, usually shot with a shotgun. Can you do that with a bow? At a range over 100 yards? While the targets are slowly changing their position? Maybe you'd have some soldiers used to combat these guys that really are that good, but that's a very specialized and hard to learn skill.



I don't think they'd be all that fearsome in massed battles, until they get scary stuff to drop. Like explosives.


Incendiary payloads could work in pre-gunpowder era, greek fire combined with aerial delivery is some scary stuff. The question is, where would they get the naphta needed for that?

Haighus
2018-01-22, 04:51 AM
One thing I can see happening if a change to how fortresses are designed - the more important ones would almost certainly have steel slanted roofs with sandbags on them. That would bring the need for stronger walls, but with sand slowing the darts down, they could well be impervious to anything the birdmen can carry short of explosives. And even those would be made a lot less effective by the sand, as shown by some of the western front forts in WW1.

Dover castle could be an interesting example here. It has a rather unusual solid stone roof, where most castles have a typical wooden beamed and tiled roof. This presumably is to make it more resistant to bombardment, although considering how high up the castle is, it was probably overengineered. I don't know of any other castles with a solid stone roof. The weight is supported by a vaulted ceiling below, so it draws on the architectural innovations of gothic cathedrals.

I imagine this kind of roof would be impervious to pretty much anything the bird people could drop, except from extreme (innacurate) altitude. Even gunpowder bombs would likely be ineffective in the size they would be able to carry, as much of the force would explode away from the roof, and gunpowder is not a great explosive. TNT and C4 etc would be fine, but this is well beyond medieval tech. Th e only reasonable way to damage them is to crack the stone through sheer kinetic impact, although a layer of sand/earth would be easy to apply to mitigate this, and we know castles did this in sieges, so why not to stave off aeriel bombardments?

I reckon such roofs would become far more common, even with the expense of creating them.

Haighus
2018-01-22, 04:57 AM
They don't have to dodge. Analogous situation would be hitting a WW1 airplane with a rifle - if you happen to aim correctly, the airplane doesn't have time to dodge, simply because a bullet is so much faster, the problem is that you need to know where to point the rifle in the first place.

If our birdman is making slow turns as he flies, you won't be able to even walk your shots all that well, and walking shots is only possible with a bow or light crossbow anyway, not a ballista. Add to that that you're shooting at your far range, and it becomes kinda like sniping - to hit the target, you need to point your weapon at a point that's not anywhere near the target. With ground targets, you only need to correct your orientation and distance, but flyers add altitude to the equation, and that gets messy fast.

As an example, there are those clay pigeons, usually shot with a shotgun. Can you do that with a bow? At a range over 100 yards? While the targets are slowly changing their position? Maybe you'd have some soldiers used to combat these guys that really are that good, but that's a very specialized and hard to learn skill.



Incendiary payloads could work in pre-gunpowder era, greek fire combined with aerial delivery is some scary stuff. The question is, where would they get the naphta needed for that?
Hmm, they would likely be in greater numbers than WWI biplanes though, otherwise they wouldn't have much impact on the groundtroops. One bird isn't going to have much impact. They will also be much slower than biplanes (probably less than 60mph in most circumstances), so hitti g them will be easier. I suspect they will fly as a formation, which makes them more vulnerable.

Also, if I were designing an AA system, I,d use the (likely) superior numbers of ground troops to essentially fill the air with shot/arrows. So no one directly aims at the bird, everyone aims at their segment and fills the sky with lethal projectiles. This wasn't so helpful in WWI because very powerful artillery forces the troops to be more dispersed.

snowblizz
2018-01-23, 06:29 AM
What I think is a better option, especially with lower tech levels, is something like trick shooting - these avian menaces don't wear armor, so even a relatively light bow will do, and that means you can fill the air with a LOT of arrows, if you have good supply of those. Accuracy by volume, if you will. Something like Chinese cho ko nu is a pretty good fit.


The Hwacha. Almost like it was made for this.:smallbiggrin: It's not going to accurately hit anything but it's going to make area denial possible and with the slight inaccuracy in trajectories (the one the Mythbusters tested went all over the place) the arrows will be hard to dodge as you can't really predict where anything in the swarm of arrow-rockets are going to end up. It should be very effective as a deterrent I'd imagine. Adn it's not exactly very high-tech if you only have rudimentary gunpowder.

https://upload.wikimedia.org/wikipedia/commons/thumb/0/04/Hwacha-Shinkigeon_Style.jpg/800px-Hwacha-Shinkigeon_Style.jpg

Combine with highly skilled archers that could probably have a fair go at hitting such a target at fair distance if you only get solitary fliers coming at you.

Brother Oni
2018-01-23, 07:27 AM
Adn it's not exactly very high-tech if you only have rudimentary gunpowder.

We are heading into the Late Middle Ages though (the hwacha was first developed in the 15th Century), with the most famous version of the hwacha seeing their most extensive use during the 16th Century Imjin Wars, well into the Early Modern Period.

By the time you're got matchlock arquebuses and muskets, flying opponents are a lot less insurmountable, even if they are similarly equipped.

2D8HP
2018-01-23, 12:01 PM
The Battle of Cerignola in 1503 was largely won by Spain through the use of matchlock firearms, but the English warship The Mary Rose (http://www.maryrose.org/meet-the-crew/soldiers-and-gunners/archery/), which sunk in 1545 during Henry the 8th reign, still had longbows for war use, as the bone structure of skeletons recovered with the wreck indicate (they showed lifelong practice pulling the great draw weights), just as they were 100 years earlier, but in the time that Henry the 8th daughter Elizabeth was Queen, longbow were hardly used for war, even by the British.

I've read how the longbow was more effective than early firearms (rate of fire, etc.), but that lifelong training was required, but that was true compared to crossbows as well.

Now I'm wondering why only the British had the lifelong training to use l ongbows effectively in war, and why didn't they continue the training?

Kiero
2018-01-23, 12:16 PM
Now I'm wondering why only the British had the lifelong training to use l ongbows effectively in war, and why didn't they continue the training?

Probably because the "craze" which had first made lots of longbowmen available to English kings had long since passed. It's a highly specific skill with few peacetime uses (smaller self bows are perfectly sufficient for hunting) that requires a lot of time and effort dedicated to it. They passed laws making Sunday practise compulsory in an attempt to maintain the customs, but once people had lost interest, they were fighting a losing battle.

ExLibrisMortis
2018-01-23, 02:39 PM
Thanks to everyone here for a very interesting discussion on flying humanoids. I wonder if we might proceed to the next step. To paraphrase Mythbusters: "Winged humans with hawk-like maneuverability? Busted! But we're not giving up until we get some hawk-like humans in the air. We're going to replicate the result". (By which I mean: I have annoying and difficult questions and I'm not going to replicate anything myself, but I welcome answers of any kind.)


1) What sort of mechanical tricks might a flying humanoid (as calculated to be plausible) use to improve its flight? I'd expect things like spyglasses and topographical maps to be a speciality of flying hominids, but what would they use to fly? After all, humans use flippers and running shoes (and human-powered aircraft), so so should our flying cousins. Would they use, say, wingsuits? Steering vanes on the legs? Pole-vault take-offs? Cargo kites?


2) I'm not confident the mechanical tricks will be enough, but in a world with dragons, there's (arguably) the magical to fall back on. Specifically, magic as a force applied to the body of the flier, with a certain distribution, for some time, in some direction. You have to eat to sustain this application of force, but it's applied with 100% efficiency (that's why it's magic). Oh, and the force should be smaller than the force of gravity on your body (as small as possible, really), else we're levitating, and that's not the point of the game.

If this force is evenly distributed, always-on, and pointing straight up (acting directly against gravity), it's effectively a reduction of weight (but not mass, so inertia is unchanged, so it shouldn't affect vulnerability to high winds, I think?). This should really help reduce required wing area, with all the energy savings (or wing power gains) that entails, but it's a bit inefficient to have this reduced weight while resting, and it may even be a disadvantage on the ground (less grip).

If you can vary the magnitude through time, you can turn it off when not flying, and max it out when taking off. That should reduce your daily energy consumption by half, at least (assuming twelve hours of non-flying per day).

If you can vary the direction through time, you can use it for propulsion, generating lift. Would it be more efficient to generate additional lift, or would you rather spend your magic mitigating gravity? You could also use magic for power-dives or charges on the ground, compensating for reduced ground-optimized musculature compared to non-flying humans (this might help preserve the totally-human-but-winged aesthetic).

If you can vary the distribution through time, you get to questions I don't even know how to begin to search for the answers to. For example: would you rather have a heavier wing and a lighter body during the downstroke, and the reverse during the upstroke, or the opposite? Would it be useful to mitigate the weight of your legs to maintain flying posture? Is it useful to increase the weight of your hand while throwing a javelin down onto something?

Of course, combining the three gives you a neat superpower, and would reduce the force required to the absolute minimum. I think that once you have autokinesis, selection pressure would be towards the ability to manipulate it as described above, and the fully evolved magical bird-humans would have something like it (incidentally also useful to explain dragons, elves, dwarves, and whatnot, if you care to give them just-out-of-reach natural abilities).


For each of these, what sort of power do you need to make flying raptor-like humanoids possible? Just how out-of-bounds is the idea?


P.S. Sorry to leave you with the calculations, but my physics was never up to this, and it's rusty to boot)

rrgg
2018-01-23, 06:25 PM
We are heading into the Late Middle Ages though (the hwacha was first developed in the 15th Century), with the most famous version of the hwacha seeing their most extensive use during the 16th Century Imjin Wars, well into the Early Modern Period.

By the time you're got matchlock arquebuses and muskets, flying opponents are a lot less insurmountable, even if they are similarly equipped.

Rocket propelled arrows seem to date back to at least the 11th century or so in china. Its sort of hard to tell how common they were since Chinese sources tend to use the term "fire arrow" to refer both to rocket powered arrows and incendiary arrows launched by a bow or crossbow, but they would have been available.

Incanur
2018-01-23, 10:13 PM
English use of bows was an anomaly in 16th-century Europe, but archery persisted longer elsewhere in the world. In and around China, both cavalry and infantry archers appear to have been effective against opposing troops equipped with firearms, including European ones with European guns (https://bowvsmusket.com/2015/06/15/battle-of-tai-bay-1661/), at least into the 2nd half of the 17th century.

By the 19th century, particularly the 2nd half of the 19th century, Chinese/Manchu bows were obviously inferior to decent European firearms, but it's unclear exactly when this shift occurred.

Carl
2018-01-24, 01:51 AM
Thanks to everyone here for a very interesting discussion on flying humanoids. I wonder if we might proceed to the next step. To paraphrase Mythbusters: "Winged humans with hawk-like maneuverability? Busted! But we're not giving up until we get some hawk-like humans in the air. We're going to replicate the result". (By which I mean: I have annoying and difficult questions and I'm not going to replicate anything myself, but I welcome answers of any kind.)


1) What sort of mechanical tricks might a flying humanoid (as calculated to be plausible) use to improve its flight? I'd expect things like spyglasses and topographical maps to be a speciality of flying hominids, but what would they use to fly? After all, humans use flippers and running shoes (and human-powered aircraft), so so should our flying cousins. Would they use, say, wingsuits? Steering vanes on the legs? Pole-vault take-offs? Cargo kites?


2) I'm not confident the mechanical tricks will be enough, but in a world with dragons, there's (arguably) the magical to fall back on. Specifically, magic as a force applied to the body of the flier, with a certain distribution, for some time, in some direction. You have to eat to sustain this application of force, but it's applied with 100% efficiency (that's why it's magic). Oh, and the force should be smaller than the force of gravity on your body (as small as possible, really), else we're levitating, and that's not the point of the game.

If this force is evenly distributed, always-on, and pointing straight up (acting directly against gravity), it's effectively a reduction of weight (but not mass, so inertia is unchanged, so it shouldn't affect vulnerability to high winds, I think?). This should really help reduce required wing area, with all the energy savings (or wing power gains) that entails, but it's a bit inefficient to have this reduced weight while resting, and it may even be a disadvantage on the ground (less grip).

If you can vary the magnitude through time, you can turn it off when not flying, and max it out when taking off. That should reduce your daily energy consumption by half, at least (assuming twelve hours of non-flying per day).

If you can vary the direction through time, you can use it for propulsion, generating lift. Would it be more efficient to generate additional lift, or would you rather spend your magic mitigating gravity? You could also use magic for power-dives or charges on the ground, compensating for reduced ground-optimized musculature compared to non-flying humans (this might help preserve the totally-human-but-winged aesthetic).

If you can vary the distribution through time, you get to questions I don't even know how to begin to search for the answers to. For example: would you rather have a heavier wing and a lighter body during the downstroke, and the reverse during the upstroke, or the opposite? Would it be useful to mitigate the weight of your legs to maintain flying posture? Is it useful to increase the weight of your hand while throwing a javelin down onto something?

Of course, combining the three gives you a neat superpower, and would reduce the force required to the absolute minimum. I think that once you have autokinesis, selection pressure would be towards the ability to manipulate it as described above, and the fully evolved magical bird-humans would have something like it (incidentally also useful to explain dragons, elves, dwarves, and whatnot, if you care to give them just-out-of-reach natural abilities).


For each of these, what sort of power do you need to make flying raptor-like humanoids possible? Just how out-of-bounds is the idea?


P.S. Sorry to leave you with the calculations, but my physics was never up to this, and it's rusty to boot)

1. Nothing. The difference between an activer and a passive flyer is exactly the same as that between an airplane and a glider. And the core thing required to swap from one to the other is exactly the same. An artificial means of propulsion. Te technology to do that won;t exist for roughly a thousand years after the technology level defined for the birdmen. And even once it does, the birdmen will have been building and operating far higher performance aircraft long before they get to that point.

Tha said active flight bridmen are far from definitely busted as you put it, the problem is we genuinely can't say if nature can produce the necessary lung system, we know nature has a whole slew of ways of improving things like lung efficiency and i'm pretty sure no single creature combines all of them. So despite the fact that no such creature has ever existed, we can;t say for certain that such a creature cannot exist. The catch is such a creature if it is possibble would be supremely unlikely to occur, it would take very specific evolutionary pressures at various stages of the creatures evolution to produce such a creature.

2. Again Nothing. If they don't have the lung capacity for active flight they don;t have the lung capacity to turn food stores in their body into magical energy. Such capability would have enormous implications in other area's, but if passive flight is allready stretching their lung capacity then they're not going to have the oxygen to provide the thrust via magic. If they do have sufficient lung capacity then unless they've evolved to use magic in the place of muscles, (a not unreasonable adaption), there's no reason for maic, and you've still got te issue in that case of the unlikly evolutinary path to the necessary lungs.

Clistenes
2018-01-24, 05:08 AM
English use of bows was an anomaly in 16th-century Europe, but archery persisted longer elsewhere in the world. In and around China, both cavalry and infantry archers appear to have been effective against opposing troops equipped with firearms, including European ones with European guns (https://bowvsmusket.com/2015/06/15/battle-of-tai-bay-1661/), at least into the 2nd half of the 17th century.

By the 19th century, particularly the 2nd half of the 19th century, Chinese/Manchu bows were obviously inferior to decent European firearms, but it's unclear exactly when this shift occurred.

The Chinese had an advantage of 17 to 1, and the Dutch stood their ground until their rear was taken by surprise... if that battle proves something, it is the superiority of guns over bows...

I mean, if a sword-wielding guy keeps at bay 17 other guys armed with bricks in socks until one of them manages to sneak from behind and hit the back of his head, that doesn't mean the brick and sock are more powerful than the sword, it just means that he lost to an overwhelming numerical advantage and a surprise attack...

Haighus
2018-01-24, 06:46 AM
1. Nothing. The difference between an activer and a passive flyer is exactly the same as that between an airplane and a glider. And the core thing required to swap from one to the other is exactly the same. An artificial means of propulsion. Te technology to do that won;t exist for roughly a thousand years after the technology level defined for the birdmen. And even once it does, the birdmen will have been building and operating far higher performance aircraft long before they get to that point.

Tha said active flight bridmen are far from definitely busted as you put it, the problem is we genuinely can't say if nature can produce the necessary lung system, we know nature has a whole slew of ways of improving things like lung efficiency and i'm pretty sure no single creature combines all of them. So despite the fact that no such creature has ever existed, we can;t say for certain that such a creature cannot exist. The catch is such a creature if it is possibble would be supremely unlikely to occur, it would take very specific evolutionary pressures at various stages of the creatures evolution to produce such a creature.
Could they not also make gliding more efficient for less weight, meaning more of the available power to the bird can be used for active flying, rather than simply staying airborne. In addition, I am sure there are aids that could help direct the bird in flight, that are mechanical and use very little energy to power. Something like rudders with levers could help to steer rapidly for less energy.


2. Again Nothing. If they don't have the lung capacity for active flight they don;t have the lung capacity to turn food stores in their body into magical energy. Such capability would have enormous implications in other area's, but if passive flight is allready stretching their lung capacity then they're not going to have the oxygen to provide the thrust via magic. If they do have sufficient lung capacity then unless they've evolved to use magic in the place of muscles, (a not unreasonable adaption), there's no reason for maic, and you've still got te issue in that case of the unlikly evolutinary path to the necessary lungs.
Magic also adds no weight premium- muscle has weight, so using magic instead of more muscle means a much better power to weight ratio. Oxygen intake is not the only limit of power output, muscle and wing weight are too. Based on this, I think magic would be the superior option if it could be done efficiently.

Mabn
2018-01-24, 07:08 AM
so I've only glanced at the flying people discussion since the beginning of this thread, so I may have missed some things, but off the top of my head I question whether birds are the most practical model to use. A more pterosaur-like design allows for several advantages.

Firstly, it allows quite a bit more flexibility in weight scales, pterosaurs were simply able to grow bigger and be able to fly. Some of that was obviously aided by their living in a hotter, more oxygen-rich time, but the difference is substantial enough to merit attention. A humanoid flyer wouldn't have to be as big or heavy as a pterosaur could be, but the more inefficiencies you add, the smaller a flyer can be, so it allows us to add in things like arms and not be so at risk of becoming a dynamic impossibility.

Secondly, take off would be vastly easier because they would push off with their much stronger and more durable forelimbs rather than their legs, which lets the legs have a much more practical percentage of body weight.

Thirdly, pterosaur heads and necks were huge. The weight and blood flow of a human head would be trivial in comparison, birds heads and necks are much lighter than a humans proportionately, generally speaking. Not having to move through a neck half a dozen times longer than your entire torso also means the respiratory system of a pterasaur with a human head would have much less dead air with every breath it took, and since pterosaurs already had one hell of a respiratory system the improvement might actually mean our flying monkeys were the most efficient oxygen processors.

Fourthly, going back to the legs, human legs are a hell of a drag factor on a bird. Storks and the like make it work, but it isn't free. Pterosaurs long hind legs (compared to body size) actually serve a vital function in their flight. Their feet are still tiny because of the drag thing, but their wings act like absolutely ripped hands-free crutches, so balance shouldn't be a factor.

Finally as an aside, both birds and pterosaurs have fused spines and very different muscle attachments on their rib-cage equivalents, both of which I believe would make the addition of arms much more feasible. Still makes shoulder blades a nightmare to work in with wings and it would be far easier to just have the flying people have a significantly restricted range of upper arm motion.

Edit: this is going quite a way from the initial premise, but being able to balance on their forelimbs and having prehensile feet is a far easier structure to work with than 6 limbs. It would mean only being able to hold things in one of their "hands" at a time while moving on land (power crutches making hopping on one leg perfectly doable) and even then would rule out incorporating the legs into the wing structure like pterasaurs or bats if they wanted to be able to not get tangled up and to hold stuff while flying. The weight savings and otherwise increased functionality are quite nice however and these guys were never going to be great porters anyway.

ExLibrisMortis
2018-01-24, 08:39 AM
1. Nothing. The difference between an activer and a passive flyer is exactly the same as that between an airplane and a glider. And the core thing required to swap from one to the other is exactly the same. An artificial means of propulsion. The technology to do that won;t exist for roughly a thousand years after the technology level defined for the birdmen. And even once it does, the birdmen will have been building and operating far higher performance aircraft long before they get to that point.
So the extra wing area from using a wingsuit or flaps won't help at all, but you could use a booster rocket to take off (disregarding the obvious risk involved)? Why wouldn't an extra square metre or two of wing area help active flight; it should produce extra lift, right?


2. Again Nothing. If they don't have the lung capacity for active flight they don;t have the lung capacity to turn food stores in their body into magical energy. Such capability would have enormous implications in other area's, but if passive flight is allready stretching their lung capacity then they're not going to have the oxygen to provide the thrust via magic. If they do have sufficient lung capacity then unless they've evolved to use magic in the place of muscles, (a not unreasonable adaption), there's no reason for maic, and you've still got te issue in that case of the unlikly evolutinary path to the necessary lungs.
Right, you're assuming (sensibly) that magic is applied as you burn energy. I was thinking a little more along the lines of already-available magic built up with leftover metabolic capacity during downtime. Instead of storing energy in fat and sugar and then metabolizing it as the muscles do work, you let unspecified organs (maybe the muscles, why not) do work during downtime, and the result of said work is magic, which you store (at no weight cost) and apply during flight.

However, even if oxygen intake limits the use magic, it's still fairly obvious that replacing muscles with magic involves big efficiency and weight gains, so the question stands. The idea of the question is to find the amount of "no weight/no loss" power output you need to get good (hawk-like) flight performance. It'd give you an indication of the efficiency gains required to make it happen non-magically, too.

Mike_G
2018-01-24, 10:06 AM
Magic can be whatever you want it to be. If dragons the size of plesiosaurs with armored scales can fly, then winged humanoids can fly.

Without magic, it's tough. Humans need a significant redesign to get them to fly, so you either need to make them very different, or accept certain things just are fantasy.

The idea of lighter, slender humans with a large wingspan compared to body size at least allows for the appearance of plausibility.

So I guess the question ios how much real physics do you want and how much magic/handwaving is ok?

Because you have Hawkman on one end of the spectrum, and an intelligent, tool using pterasaur or were-condor on the other.

Galloglaich
2018-01-24, 10:51 AM
English use of bows was an anomaly in 16th-century Europe, but archery persisted longer elsewhere in the world. In and around China, both cavalry and infantry archers appear to have been effective against opposing troops equipped with firearms, including European ones with European guns (https://bowvsmusket.com/2015/06/15/battle-of-tai-bay-1661/), at least into the 2nd half of the 17th century.

By the 19th century, particularly the 2nd half of the 19th century, Chinese/Manchu bows were obviously inferior to decent European firearms, but it's unclear exactly when this shift occurred.

Interesting and nicely detailed anecdote there, but 4,000 men vs. 240 hardly seems to make a decisive case for the superiority of bow over musket.

G

Carl
2018-01-24, 11:03 AM
Could they not also make gliding more efficient for less weight, meaning more of the available power to the bird can be used for active flying, rather than simply staying airborne. In addition, I am sure there are aids that could help direct the bird in flight, that are mechanical and use very little energy to power. Something like rudders with levers could help to steer rapidly for less energy.


Magic also adds no weight premium- muscle has weight, so using magic instead of more muscle means a much better power to weight ratio. Oxygen intake is not the only limit of power output, muscle and wing weight are too. Based on this, I think magic would be the superior option if it could be done efficiently.

For the first part see my reply below. For the second whilst ExLiberMortis has clarified somthing the atrophy of muscles only changes things in so much as it lightens the loads somewhat, but unless your going to have a birdman be much smaller than a human there is, (particularly accounting for aerodynamic concerns), a limit on how far that can go.


So the extra wing area from using a wingsuit or flaps won't help at all, but you could use a booster rocket to take off (disregarding the obvious risk involved)? Why wouldn't an extra square metre or two of wing area help active flight; it should produce extra lift, right?


Right, you're assuming (sensibly) that magic is applied as you burn energy. I was thinking a little more along the lines of already-available magic built up with leftover metabolic capacity during downtime. Instead of storing energy in fat and sugar and then metabolizing it as the muscles do work, you let unspecified organs (maybe the muscles, why not) do work during downtime, and the result of said work is magic, which you store (at no weight cost) and apply during flight.

However, even if oxygen intake limits the use magic, it's still fairly obvious that replacing muscles with magic involves big efficiency and weight gains, so the question stands. The idea of the question is to find the amount of "no weight/no loss" power output you need to get good (hawk-like) flight performance. It'd give you an indication of the efficiency gains required to make it happen non-magically, too.


First bear in mind what i'm saying next is based on reading up, my engineering training didn't cover fixed wing flight and that leaves me with at least one serious gap in my knowledge in the sense that i know a particular principle is true, (and i can thus apply it), but there's an aspect to how it "makes sense" under the laws of physics that i don't understand.

However the core principle you need to know is that at whatever sped the fliers lift exceeds its weight the thrust must also exceed the drag or velocity will be lost, this reduces lift eventually resulting i9n lift no longer exceeding mass and decent. However the very act of losing altitude (in simple terms), produces energy which becomes forward velocity. In effect gliders trade altitude to keep velocity constant so they can generate sufficient lift to overcome gravity. Yes there really is a difference between falling, (however stylish), and flight. An active flier flies at a constant speed by generating more thrust than drag. The thin is passive flier bird basically generate no thrust whatsoever measured over any appreciable period of time. So whilst a wing suit might improve lift to drag allowing a slower glide speed, (though this would be worked against by the fact that the natural wing would be optimized for a higher speed), it dosen;t overcome the relative lack of forward thrust.

The additional problem is that turning takes energy, a lot of energy usually so maneuvering flight require a yet larger increase in thrust. and a wingsuit dosen't do a thing to help there.

As far as pre-storing, it comes down to the amount they can pre-store. It also comes back to dietary questions, if they're not optimised to be able to take in very large caloric volumes, (and not able to take i enough oxygen to process it), with appropriate food sources it doesn't matter how much they can store because it's going to take too long to store up. Certainly a stored system may be more practical as i imagine a magic force based thrust system would be more efficient than typical flapping flight, but energy costs are still going to be high.

As an aside i'm not sure of the precise energy input required, you'd need to know the average drag of high maneuvering bird flight without flapping which is a pretty big unknown, i'd assume high however.

EDIT: To be clear i assume high because maneuvering by its nature has significant drag increasing problems which means the more maneuverable you want to be the more energy intensive. Wiki has a good description of why on it's wing loading article (https://en.wikipedia.org/wiki/Wing_loading).

ExLibrisMortis
2018-01-24, 12:54 PM
So I guess the question ios how much real physics do you want and how much magic/handwaving is ok?
I more-or-less precisely defined what sort of magic I mean in my post (http://www.giantitp.com/forums/showsinglepost.php?p=22779099&postcount=70). In the simplest form, it works out to the ability to counteract some percentage of gravity (less than 100%, of course).


As an aside i'm not sure of the precise energy input required, you'd need to know the average drag of high maneuvering bird flight without flapping which is a pretty big unknown, i'd assume high however.
Right, that makes sense, I guess that's what I was after. From what you say, I imagine a wingsuit would increase your ability to travel long distances efficiently by employing thermals and gliding, but doesn't appreciably improve your in-combat performance, which is largely thrust-dependant.

On the topic of caloric requirements, I believe top rowers and cyclists eat around 13 000 kJ/day even years after retiring, down from as high as 25 000 kJ/day during their professional careers, due to long-term changes in base metabolic rate and generally being active people. Given that a human can achieve those numbers (at least for men, but not large men (cyclists)), I'd expect that bird people can, with some evolutionary and cultural adjustment, achieve that 13 000 kJ/day average, being higher for active soldiers and lower for civilians and elderly. Supplying them would be annoying, but I think they can eat enough.

Mr Beer
2018-01-24, 02:06 PM
Magic is whatever you want, the thing is that 'nearly flying' or 'enhance flying' is very close to 'flying' and if flying magic is doable, then birdmen are not special because any jackass with mana can fly. So you need to consider how and why this ability is limited.

I would have flying magic do things like allow the birdman to carry more stuff (Lift Master) or fly faster (Fleet Wings) or be harder to hit (Ariel Stealth)...that kind of stuff. Not so much worrying about the exact forces it's creating.

Then I would use the magic system to make it relatively common/easy for birdmen spellusers to enhance flight. Only powerful (rare) non-birdmen wizards can fly under their own volition and magic objects that allow flight would be extremely rare/expensive.

Yora
2018-01-24, 02:17 PM
Are there any major cities in recent centuries that were so badly destroyed that they were never rebuild to something of comparable size? I can't really think of any and only of a very few back in antiquity.

Berenger
2018-01-24, 02:43 PM
Are there any major cities in recent centuries that were so badly destroyed that they were never rebuild to something of comparable size? I can't really think of any and only of a very few back in antiquity.

You probably already found it, but there is a thread at reddit on this subject: https://www.reddit.com/r/history/comments/4ld04u/are_there_any_citiestowns_that_were_destroyed_in/

Spiryt
2018-01-24, 03:21 PM
Are there any major cities in recent centuries that were so badly destroyed that they were never rebuild to something of comparable size? I can't really think of any and only of a very few back in antiquity.

Depends on what you mean by 'recent centuries', and what do you mean by 'destroyed' I guess?

Mongols had destroyed quite a lot of great urban centers in Kharezm, from what I gather. They've got rebuilt later, but subsequent razing could leave the city abandoned.

https://en.wikipedia.org/wiki/Konye-Urgench

https://en.wikipedia.org/wiki/Tus,_Iran



Kungahälla was, as name implies, the hall of the Konungs, in modern Sweden, but after it got destroyed by Pomeranians in 1136, it never recovered.

https://en.wikipedia.org/wiki/Kungahälla

Now it's pretty much known only from Snorri Sturluson's saga.

Haighus
2018-01-24, 06:58 PM
For the first part see my reply below. For the second whilst ExLiberMortis has clarified somthing the atrophy of muscles only changes things in so much as it lightens the loads somewhat, but unless your going to have a birdman be much smaller than a human there is, (particularly accounting for aerodynamic concerns), a limit on how far that can go.




First bear in mind what i'm saying next is based on reading up, my engineering training didn't cover fixed wing flight and that leaves me with at least one serious gap in my knowledge in the sense that i know a particular principle is true, (and i can thus apply it), but there's an aspect to how it "makes sense" under the laws of physics that i don't understand.

However the core principle you need to know is that at whatever sped the fliers lift exceeds its weight the thrust must also exceed the drag or velocity will be lost, this reduces lift eventually resulting i9n lift no longer exceeding mass and decent. However the very act of losing altitude (in simple terms), produces energy which becomes forward velocity. In effect gliders trade altitude to keep velocity constant so they can generate sufficient lift to overcome gravity. Yes there really is a difference between falling, (however stylish), and flight. An active flier flies at a constant speed by generating more thrust than drag. The thin is passive flier bird basically generate no thrust whatsoever measured over any appreciable period of time. So whilst a wing suit might improve lift to drag allowing a slower glide speed, (though this would be worked against by the fact that the natural wing would be optimized for a higher speed), it dosen;t overcome the relative lack of forward thrust.

The additional problem is that turning takes energy, a lot of energy usually so maneuvering flight require a yet larger increase in thrust. and a wingsuit dosen't do a thing to help there.

As far as pre-storing, it comes down to the amount they can pre-store. It also comes back to dietary questions, if they're not optimised to be able to take in very large caloric volumes, (and not able to take i enough oxygen to process it), with appropriate food sources it doesn't matter how much they can store because it's going to take too long to store up. Certainly a stored system may be more practical as i imagine a magic force based thrust system would be more efficient than typical flapping flight, but energy costs are still going to be high.

As an aside i'm not sure of the precise energy input required, you'd need to know the average drag of high maneuvering bird flight without flapping which is a pretty big unknown, i'd assume high however.

EDIT: To be clear i assume high because maneuvering by its nature has significant drag increasing problems which means the more maneuverable you want to be the more energy intensive. Wiki has a good description of why on it's wing loading article (https://en.wikipedia.org/wiki/Wing_loading).

From what I was reading with andean condors and Argentavis, an improved gliding efficiency means a lower active energy cost to maintain active flying, because less forward velocity is needed to prevent a loss in altitude. This would largely only help in long-distance powered flight I would suspect. The way you word it above makes a lot of sense though- from what I can gather passive flying is essentially deriving the thrust and energy from elsewhere (thermals or wind)? For manouevring, would levers operating rudders not have the potential to make some of the motions more mechanically efficient? This a stretch, but I am mainly considering plausibility here.

spets
2018-01-24, 06:58 PM
1) What sort of mechanical tricks might a flying humanoid (as calculated to be plausible) use to improve its flight? I'd expect things like spyglasses and topographical maps to be a speciality of flying hominids, but what would they use to fly? After all, humans use flippers and running shoes (and human-powered aircraft), so so should our flying cousins. Would they use, say, wingsuits? Steering vanes on the legs? Pole-vault take-offs? Cargo kites?

Don't know about flight, but I had some fun thinking up other things a quasi-realistic flyer might use (I'm imagining birdmen with four limbs and some form of gripping system at the 'hand');

Running poles - our flyers might not be vastly inferior runners to the earthhuggers but they're probably only good in a straight line; ski poles might help them turn corners sharply when moving on foot, particularly with a cultural tendency to use arms in movement rather than for carrying weapons/items and the rocky, mountainous home terrain.

Polearms - hooked bill-like weapons sharpened on both sides. The curved side makes a fine slashing weapon when used in flight, held downward like a boat pole and swiped with a twist of the shoulder in a hit-and-run attack - the curve prevents the blade sticking much like a cavalry scimitar. While our troops prefer to fight on the wing, the hooked side allows them to defend their mountain passes on foot passably well against the goblin-folk, pulling their shields aside or yanking them upwards by the neck. A polearm is counterweighed on the wing by javelins or a weight in the off hand. Cheaply made and carrying no prestige, they are dropped before a flying retreat - on foot it is never dropped, for use as a running pole should escape be necessary.

Armour - padded jacks are used by cliff guards, giving them an almost human resistance to blunt trauma, though the heat is prohibitive for any time on the wing. Strapped versions allow a knot to be cut at the back and the jack to be shed instantly for unencumbered skyward retreat.

Screamers - banners tied to the waist are popular amongst our flyers, but screamers are used solely for night or terror raids; vented whistles and noisemakers on waists and ankles that, when unstoppered during a battle, create a piercing shriek when our fighters go in for a dive. Scatters animals and conscripts alike.

Haighus
2018-01-24, 07:13 PM
Are there any major cities in recent centuries that were so badly destroyed that they were never rebuild to something of comparable size? I can't really think of any and only of a very few back in antiquity.

I have a minor example on a much smaller scale :smallbiggrin: It isn't a city, but there was a small town (http://www.shropshirehistory.org.uk/html/search/verb/GetRecord/theme:20061130104059) located near where I grew up. It started as a Norman motte-and-bailey after the conquest of England, and became a small but bustling little borough. There was then a period of decline due to neglectful lords, but the final deathnell was when the owner of the castle supported the Royalists in the English civil war, and the castle and what was left of the town were destroyed. The location now boasts a wooded hill with a farm next to it, and some limited ruins. Many of the local houses were built from pilfered stone (including my parents' house). It is interesting to note that the settlement entirely ceased to exist.

Caus castle today:

https://upload.wikimedia.org/wikipedia/commons/5/5a/Caus_Castle_-_geograph.org.uk_-_487557.jpg



In an example that is perhaps more relevant to your question, Angkor (https://en.wikipedia.org/wiki/Angkor#End_of_the_Angkorian_period) could be a contender? Went from being probably the largest city on the planet in its heyday to forgotten ruins in the jungle. Generally considered to be abandoned following a sacking in 1431.

Yora
2018-01-25, 01:04 AM
There are plenty of ancient and medieval cities, and an endless number of villages, and of course pretty much all population centers in the Americas in the 16th century.
But it seems to me that since 1600, every location that had a major city on it, got pretty much immediately rebuild, even if the destruction of the buildings was more or less complete.

rrgg
2018-01-25, 01:33 AM
The Battle of Cerignola in 1503 was largely won by Spain through the use of matchlock firearms, but the English warship The Mary Rose (http://www.maryrose.org/meet-the-crew/soldiers-and-gunners/archery/), which sunk in 1545 during Henry the 8th reign, still had longbows for war use, as the bone structure of skeletons recovered with the wreck indicate (they showed lifelong practice pulling the great draw weights), just as they were 100 years earlier, but in the time that Henry the 8th daughter Elizabeth was Queen, longbow were hardly used for war, even by the British.

I've read how the longbow was more effective than early firearms (rate of fire, etc.), but that lifelong training was required, but that was true compared to crossbows as well.

Now I'm wondering why only the British had the lifelong training to use l ongbows effectively in war, and why didn't they continue the training?

Honestly a lot of the reason the English were so good at archery basically boils down to "because the English were known for being really good at archery." It started in perhaps the early 1300s when English archers did pretty well in the war against the Scots and then over the course of the HYW the longbow became more and more ingrained into English culture and the English national identity.

It's not really the case that just being english turned someone into a super archer (only around 1/5th of the english skeletons from the Mary Rose had changes to the shoulder blades consistant with intensive practice from a young age), but because of the culture everyone was at least familiar with archery: they grew up learning stories about Agincourt and Crecy, it was a very popular pasttime among all classes, it was a very cheap pasttime, the virtues of the longbow even made its way into church sermons, etc. Basically because everyone is exposed to archery those who happen have a knack for it find out early on and can start practicing more seriously, and even if you do need to suddenly train up a company of longbowmen since everyone has at least some archery experience you don't have to worry about starting from scratch. Archery didn't completely dissapper from the continent in the 1400s and there occasionally were really good archers serving in continental armies, it's just that the English typically had way more of them.

By the 1540s though most of western europe had already transferred to firearms almost completely and it was quickly becoming apparent that arquebuses and muskets now outmatch the English longbow in pretty much every way (range, accuracy, power etc.) except for rate of fire, and even then the longbow couldn't really match a musket loaded with a handful of pistol bullets. So the rest of the 16th century basically saw England desperately working to modernize its military with modern pikes and firearms while the old medieval "bows and bills" were increasingly relegated to the status of last resort milita weapons, until 1595 when the Privy council basically stated outright that the longbow was obsolete and no longer fit for any military service.

rrgg
2018-01-25, 04:20 AM
English use of bows was an anomaly in 16th-century Europe, but archery persisted longer elsewhere in the world. In and around China, both cavalry and infantry archers appear to have been effective against opposing troops equipped with firearms, including European ones with European guns (https://bowvsmusket.com/2015/06/15/battle-of-tai-bay-1661/), at least into the 2nd half of the 17th century.

By the 19th century, particularly the 2nd half of the 19th century, Chinese/Manchu bows were obviously inferior to decent European firearms, but it's unclear exactly when this shift occurred.

A better example would probably be the battle of Sarhu in 1618, when Manchu forces armed with essentially no firearms whatsoever defeated a much larger Ming army including at least 10,000 korean musketeers. Even then though I think it more showcases the advantage Manchu horses provided moreso than their bows.

Carl
2018-01-25, 06:17 AM
I more-or-less precisely defined what sort of magic I mean in my post (http://www.giantitp.com/forums/showsinglepost.php?p=22779099&postcount=70). In the simplest form, it works out to the ability to counteract some percentage of gravity (less than 100%, of course).


Right, that makes sense, I guess that's what I was after. From what you say, I imagine a wingsuit would increase your ability to travel long distances efficiently by employing thermals and gliding, but doesn't appreciably improve your in-combat performance, which is largely thrust-dependant.

On the topic of caloric requirements, I believe top rowers and cyclists eat around 13 000 kJ/day even years after retiring, down from as high as 25 000 kJ/day during their professional careers, due to long-term changes in base metabolic rate and generally being active people. Given that a human can achieve those numbers (at least for men, but not large men (cyclists)), I'd expect that bird people can, with some evolutionary and cultural adjustment, achieve that 13 000 kJ/day average, being higher for active soldiers and lower for civilians and elderly. Supplying them would be annoying, but I think they can eat enough.

The problem is 25,000kj is peanuts to a bird sized active flier, they'd use 270,000-360,000Kj's over a 6-8 hour flight time.


From what I was reading with andean condors and Argentavis, an improved gliding efficiency means a lower active energy cost to maintain active flying, because less forward velocity is needed to prevent a loss in altitude. This would largely only help in long-distance powered flight I would suspect. The way you word it above makes a lot of sense though- from what I can gather passive flying is essentially deriving the thrust and energy from elsewhere (thermals or wind)? For manoeuvring, would levers operating rudders not have the potential to make some of the motions more mechanically efficient? This a stretch, but I am mainly considering plausibility here.

Birds are allready amazingly efficient gliders when they've got updrafts, thermals, or for dynamic soarers like the albatross wind to exploit. The Albatross is particularly crazy, they can travel upto 16,000km in a single flight, (only 2 airliners in the worlds can claim to mach or exceed this), pending over a month on the wing whilst doing so and have been known to circumnavigate the globe in as little as 46 days. A birdman able to carry a small supply of suitable rations and with the advantage of intelligance to better exploit wind conditions that other birds don't because they never encounter them sufficiently often to learn how could probably do a round the world trip in less time and without ever landing. Long distance flight they can do, it's the active maneuvering that would give them hell.

snowblizz
2018-01-25, 06:32 AM
There are plenty of ancient and medieval cities, and an endless number of villages, and of course pretty much all population centers in the Americas in the 16th century.
But it seems to me that since 1600, every location that had a major city on it, got pretty much immediately rebuild, even if the destruction of the buildings was more or less complete.

Generally speaking that's probably true. But consider what are the options? You had a city in that place because it made sense for various reasons. These tend to remain so you would want the town to remain and you've got to do somethign with the populace, or what's left of it, usually the simplest path is to rebuild their own home. Even execessively ruined cities can have some semblance of basic infrastrucutre, like roads and underground infrastructures that can be valuable in rebuilding. In Europe at least resettling a major city just can't be done very easily because there were reasons the cities were there in the first place.

Some exceptions exists. Fukushima and the town closest to Tjernobyl e.g. (at least for the forseable future), there's a French town that was massacred in WW2 by the Germans and was left empty as a monument sort of. Some cities that relied on a major antural resource that has been used up do go this way but social inertia means we fight to keep them around too often.

An interesting example, sort of, is the Swedish town of Kiruna which is currently in the process of being relocated a couple of miles because the iron ore seam being mined is taking it towards the current city centre making buildings unstable and dangerous due to the work underground.

Basically it's more common for ancient cities to be abandoned because one was largely dependant on argiculture. Today cities are more about the people really and as such rebuilding them for the economci and social factors why they existed made sense.
Interestingly enough, it's also difficult to build a large city from scratch, most such modern examples haven't been the biggest successes.

I think there are plenty of examples of cities that due to a catastrophe were severely reduced though. IIRC Magdeburg before the sack was a large, wealthy city, but it never really recovered from it's destruction in the 30YW.

The "viking" town of Birka in what's now Swede was razed by the local king when he lost control of it and a new town of Sigtuna was built further north essentially moving the there.

I'll have to see if I can think of more. Usually a more radical change in circumstances has to happen for a city to not be rebuilt, it's totally destroyed is funnily enough not quite enough.

Haighus
2018-01-25, 07:54 AM
Generally speaking that's probably true. But consider what are the options? You had a city in that place because it made sense for various reasons. These tend to remain so you would want the town to remain and you've got to do somethign with the populace, or what's left of it, usually the simplest path is to rebuild their own home. Even execessively ruined cities can have some semblance of basic infrastrucutre, like roads and underground infrastructures that can be valuable in rebuilding. In Europe at least resettling a major city just can't be done very easily because there were reasons the cities were there in the first place.

Some exceptions exists. Fukushima and the town closest to Tjernobyl e.g. (at least for the forseable future), there's a French town that was massacred in WW2 by the Germans and was left empty as a monument sort of. Some cities that relied on a major antural resource that has been used up do go this way but social inertia means we fight to keep them around too often.

An interesting example, sort of, is the Swedish town of Kiruna which is currently in the process of being relocated a couple of miles because the iron ore seam being mined is taking it towards the current city centre making buildings unstable and dangerous due to the work underground.

Basically it's more common for ancient cities to be abandoned because one was largely dependant on argiculture. Today cities are more about the people really and as such rebuilding them for the economci and social factors why they existed made sense.
Interestingly enough, it's also difficult to build a large city from scratch, most such modern examples haven't been the biggest successes.

I think there are plenty of examples of cities that due to a catastrophe were severely reduced though. IIRC Magdeburg before the sack was a large, wealthy city, but it never really recovered from it's destruction in the 30YW.

The "viking" town of Birka in what's now Swede was razed by the local king when he lost control of it and a new town of Sigtuna was built further north essentially moving the there.

I'll have to see if I can think of more. Usually a more radical change in circumstances has to happen for a city to not be rebuilt, it's totally destroyed is funnily enough not quite enough.
Local climate and environmental changes (including natural disasters) seem to be a common cause of this. There was that Egyptian city posted in the last thread, which just sunk into the ocean, and no one bothered rebuilding it because the Nile distributary shifted and left the port with no river access. There are also well known examples like Pompeii being buried in volcanic ash.

gkathellar
2018-01-25, 10:29 AM
I more-or-less precisely defined what sort of magic I mean in my post (http://www.giantitp.com/forums/showsinglepost.php?p=22779099&postcount=70). In the simplest form, it works out to the ability to counteract some percentage of gravity (less than 100%, of course).

Then it becomes progressively easier based on how much gravity they can ignore, and also ... honestly, the selective application of gravity gets pretty weird. How do two bodies operating under different gravities interact? How far does this ability to reduce gravity extend beyond the flying dude? Gravity has a crucial role in nearly all aspects of physiology, from the immune and cardiovascular systems to bone and muscle formation. I think for the sake of simplicity you may just want to say that these guys weigh far, far less than is plausible - i.e. they're made of SmalleculesTM, and that's all we have to say about that.

The big limitation on a flyer's size is the ratio of (muscle weight + bone weight) : (pectoral muscle output + wing size advantage). To maximize the back end of the ratio, flying animals tend to have minimized lower limbs, shortened necks, light bones, and tremendously emphasized pectoral muscles. I can't stress this last one enough: the breast is to a flying bird as fuel is to a rocket, in that it has to lift its own weight and the weight of absolutely everything else. Even if their arms were wings, a human could not fly within normal weight constraints because our chest muscles cannot manage that kind of output. Once you start redesigning humanoid body plan for flight, the compromises you have to make pretty much mean you'll end up with a bat.

As an alternative or supplement to weird weight, hat could make this all easier is if you take inspiration from wingsuits and hang-gliders by ruling out takeoff from a stationary position, and specifically say that your "flying" humanoids require strong headwinds to get airborne (which can be managed by leaping off of a high point). This is much more plausible, because it mainly requires changes to the arm structure in keeping with what raptors and long-term fliers like the albatross have (including the ability to lock their forelimbs in position) along with modest pectoral strength for the occasional flap. The big droopy legs a humanoid would be dragging around in the air could be kept out of the way by giving them their own "sails" or somehow incorporating them into those of the arms.

Yora
2018-01-25, 01:50 PM
A better example would probably be the battle of Sarhu in 1618, when Manchu forces armed with essentially no firearms whatsoever defeated a much larger Ming army including at least 10,000 korean musketeers. Even then though I think it more showcases the advantage Manchu horses provided moreso than their bows.

You just can't reload a musket on the back of a running horse. There was a period where you got light cavalry armed with half a dozen pistols each, but except for that, firearms as the main weapon for cavalry really only become viable with the primed cartridge.

Spiryt
2018-01-25, 02:59 PM
You just can't reload a musket on the back of a running horse. There was a period where you got light cavalry armed with half a dozen pistols each, but except for that, firearms as the main weapon for cavalry really only become viable with the primed cartridge.

You can't use a musket, that 's why you had things called carbines, musketoons, baundelieurs.

There aren't that much detail about their use available, they often could have been reloaded dismounted anyway, but they point was that the horse provided mobility to do so safely, very good for skirmishing.

rrgg
2018-01-25, 05:07 PM
You just can't reload a musket on the back of a running horse. There was a period where you got light cavalry armed with half a dozen pistols each, but except for that, firearms as the main weapon for cavalry really only become viable with the primed cartridge.

Oddly enough, it seems that shooting and reloading muzzle loaders from the back of a moving horse was far more common in Europe
(https://upload.wikimedia.org/wikipedia/commons/5/5c/Slag_bij_Borgerhout%2C_1579.JPG), india, North America, and even China was much more common than you might think. From a description in 1860:

"In the use of their weapons such as they are these Tartars and Mongolians displayed considerable dexterity in the action of yesterday. The matchlock men were frequently seen, when retreating at full speed, to turn round, fire off their pieces, and reload as they galloped away. The bowmen also discharged their arrows when at full gallop."

https://books.google.com/books?id=diwLAAAAYAAJ&lpg=PA94&dq=turn%20around%2C%20fire%20off%20their%20pieces% 2C%20and%20reload%20as%20the%20galloped%20away&pg=PA94#v=onepage&q&f=false

in Europe this was made much easier in the second half of the 16th century with the introduction of wheelocks, snaphaunces, and prepared paper cartridges for horsemen. But what I think it mainly comes down to is that both the bow and the arquebus/carbine were extremely difficult weapons to learn how to use from the back of a moving horse, so which was "best" was typically whichever one you were most familiar with. In some cases like with Turkish cavalry during the 16th-17th century, many horsemen would carry both an arquebus and a bow into battle just in case.

snowblizz
2018-01-26, 04:41 AM
Local climate and environmental changes (including natural disasters) seem to be a common cause of this. There was that Egyptian city posted in the last thread, which just sunk into the ocean, and no one bothered rebuilding it because the Nile distributary shifted and left the port with no river access. There are also well known examples like Pompeii being buried in volcanic ash.

That's usually what does a cities in. The economical factors that make them possible change dramatically. Droughts/stuff killing arable land, harbours or rivers silting up/changing course, upheavals shifting trade patterns and so on. The closer to modern times we get the more we are able to mitigate such factors. And the more important the people and economic interests they represent are.
Port Royal on Jamaica was lost in an earthquake, the administration moved to Spanish Town and changes in trading, the city was essentially gone, killed it the rest of the way.

Today cities often justify their existence by their existence, it's where we collected all the stuff. Mainly they are competed out by other such entities. So today Detroit is a shadow of it's former self due to realignment of manufacturing and one could say economic self-interests in the econo-socio-political fabric.

Must admit I find these things very interesting, ie how and why cities form, develop and decline. If anyone else has interesting examples of urban entities that are essentially abadoned as unrecoverable in more modern times I'd be curious to hear about it too.

I'm positing the statement that a city is unlikely to remain "lost" unless factors which supported it's existence are also lost in conjunction with or close by the natural or man-made disaster.

Haighus
2018-01-26, 06:02 AM
The closer to modern times we get the more we are able to mitigate such factors. And the more important the people and economic interests they represent are.
It is true- New Orleans would likely have already have been largely abandoned by now if it was in the situation it is in now with pre-industrial technology. The Old River control structure is the only thing stopping the Mississippi river from diverting down the Atchafalaya river and entirely missing New Orleans. If this had not been built, New Orleans would be economically crippled by now, and would likely have been mostly abandoned if the event had happened in earlier times.


I'm positing the statement that a city is unlikely to remain "lost" unless factors which supported it's existence are also lost in conjunction with or close by the natural or man-made disaster.

I agree, which makes those few examples with no clear change in factors all the more fascinating. Angkor could be an example of this. It was destroyed in a sacking, but that usually isn't enough to wipe out a city.

snowblizz
2018-01-26, 07:25 AM
I agree, which makes those few examples with no clear change in factors all the more fascinating. Angkor could be an example of this. It was destroyed in a sacking, but that usually isn't enough to wipe out a city.
Oh Angkor is simple really. It depended on vast amounts of aritificial irrigation, this was damaged and not kept in as good repair as needed, IIRC in part due to a relative weakening of the state which meant that the agricultural base was partially disrupted and a sacking would have been the deathblow.

I'm not sure if there were additional environmental pressures at the time like disruped monsoons or anything but I remember watching a documentary that found many of the irrigation works had degraded.

The city/temple complex apparently survived the sacking by the Chems though, in some sense, and even transferred from Hinduism to Buddhism, and apparently was never entirely abandoned, serving as a Buddhist place of worship in its later years. However the kings seem to have relocated their powerbase. Which I guess makes sense as you put your own stamp of glory on what you build.

I can't find anything to refresh my memory on the "fall" of all of these temple-cities, there have been several built by the Khmers, jungles by and large aren't good places to build permanent cities in though.

Haighus
2018-01-26, 01:05 PM
Oh Angkor is simple really. It depended on vast amounts of aritificial irrigation, this was damaged and not kept in as good repair as needed, IIRC in part due to a relative weakening of the state which meant that the agricultural base was partially disrupted and a sacking would have been the deathblow.

I'm not sure if there were additional environmental pressures at the time like disruped monsoons or anything but I remember watching a documentary that found many of the irrigation works had degraded.

The city/temple complex apparently survived the sacking by the Chems though, in some sense, and even transferred from Hinduism to Buddhism, and apparently was never entirely abandoned, serving as a Buddhist place of worship in its later years. However the kings seem to have relocated their powerbase. Which I guess makes sense as you put your own stamp of glory on what you build.

I can't find anything to refresh my memory on the "fall" of all of these temple-cities, there have been several built by the Khmers, jungles by and large aren't good places to build permanent cities in though.

Ok, this makes a lot of sense. From what I've read, the city itself was abandoned, but the Angkor Wat temple complex in the centre was the only part which remained inhabited in the following centuries as a Buddhist place of worship (was built as a Hindu temple, but had already been converted prior to the collapse). It seems to have been abandoned at some time between the 17th and 19th centuries? Quite the remarkable piece of achitecture.

Haighus
2018-01-26, 01:39 PM
So I was looking at some images of Angkor as a result of this discussion, and I noticed something interesting on one of the friezes.

https://thumbs.dreamstime.com/z/ancient-khmer-military-commander-frieze-23786073.jpg

This shows a commander leading some troops. Unlike most of the Khmer carvings showing warriors, this clearly shows the soldiers to be wearing some kind of clothing, quite possibly armour. I think the design on the carving looks remarkably similar to a lot of depictions of Chinese lamellar armour, but the way it is hanging on the designs perhaps is more suggestive of cloth armour. Quite interesting.

Edit: done some digging, apparently those are meant to be Chinese soldiers in a Khmer army. Doh!

VoxRationis
2018-01-27, 10:55 AM
At what point would sailing and logistical technology of Europe have been sufficient to make the full Spain-India westward journey in a world sans Americas? It's my understanding that Columbus thought he could do it because he mistakenly thought the Earth's circumference was smaller than it was, and all the people who had the right figure thought it couldn't be done. But at some point, sailing ships got faster, understanding of astronomy and navigation improved, and if we go far enough, we eventually get to steamships, so we would expect that a nonstop westward Europe-Asia trip would become doable. About when would that be?

Carl
2018-01-27, 12:56 PM
Not something i'm an expert on but changes in wind patterns from the lack of that landmass probably make any answer uncertain. Not to mention no Americas means no North Atlantic Current, which means europe is colder much more so for the northern regions but even spain would feel the effects. older europe means likely different societies tech pressures, e.t.c. All of which will result in changes in timeline on when different bits would become available both in absolute terms and relative to each other.

VoxRationis
2018-01-27, 01:12 PM
Right, right. Sorry. I meant "if we pretend that the Americas don't exist for the purposes of anticipating the actions of explorers and rulers." I've been thinking about an alternate history scenario where Columbus fails to return from his voyage, leading Europe to conclude that he was, unsurprisingly, wrong about the circumference of the Earth and that sailing due westward to India is possible, but not with the effective range of their ships. Eventually, however, in such a scenario, people will likely develop ships that can make the journey, leading someone else to be surprised by the existence of the Americas.

Galloglaich
2018-01-27, 02:19 PM
At what point would sailing and logistical technology of Europe have been sufficient to make the full Spain-India westward journey in a world sans Americas? It's my understanding that Columbus thought he could do it because he mistakenly thought the Earth's circumference was smaller than it was, and all the people who had the right figure thought it couldn't be done. But at some point, sailing ships got faster, understanding of astronomy and navigation improved, and if we go far enough, we eventually get to steamships, so we would expect that a nonstop westward Europe-Asia trip would become doable. About when would that be?

Good question.

The thing to keep in mind was their goal - they were trying to get to China and there were actually two ways to get there, both very difficult:


Cross the Atlantic, go around the Americas somehow and then cross the Pacific or
Sail down the West coast of Africa, round the Cape of Good Hope, sail back up the East Coast of Africa, and cross the Indian Ocean and sail around India to China.



Those of us in the US tend to focus heavily on Columbus and the discovery of the Americas, but we have to remember that the Portuguese (and others) were already well underway in trying to get to India this way throughout the 15th Century. They already had a map showing the cape and that you could sail around South Africa by at least 1450.

https://en.wikipedia.org/wiki/Fra_Mauro_map

This is the map shown next to a NASA photograph, where you can see it was quite accurate.

https://upload.wikimedia.org/wikipedia/commons/thumb/b/b1/Fra_Mauro_World_Map_Satellite_Comparison.jpg/640px-Fra_Mauro_World_Map_Satellite_Comparison.jpg

The Portuguese explorer Bartolomeu Dias reached the Cape in 1488, and Vasco Da Gama reached India for the first time in 1497.

So point #2 they had another way to get to the Far East.

Simultaneously, also through the 15th Century, Portuguese, Spanish, Italian, Flemish and Dutch explorers found many of the islands in the Atlantic. Madeira was discovered in 1419 and was already an important sugar and wine exporter by the 1430's, the Azores were discovered by a Flemish ship in 1427 already colonized in the 1430's. The Canaries, Capo Verde, Principes and most of the other East to mid-Atlantic Islands chains had all been discovered by European explorers before Columbus found America, and they were all important dropping-off points.

So in your hypothetical world with no American continents, would you have Islands? If so I suspect they would have made their way across the oceans by the end of the 16th Century regardless. The African route was available but it was fairly perilous and passed by Muslim zones which may or may not always play ball.

G

Ironsmith
2018-01-28, 10:00 AM
This one's a little strange... how usable would solid corundum (the stuff rubies and sapphires are made from) be for armor? Ignoring the logistics of how solid corundum plate would exist in the first place, due to the high melting point, would it be any good for actually preventing someone from being impaled/cut apart/beaten to death? If so, how good?

(Redirected from here (http://www.giantitp.com/forums/showthread.php?549490-New-Armor-Material))

Knaight
2018-01-28, 11:11 AM
This one's a little strange... how usable would solid corundum (the stuff rubies and sapphires are made from) be for armor? Ignoring the logistics of how solid corundum plate would exist in the first place, due to the high melting point, would it be any good for actually preventing someone from being impaled/cut apart/beaten to death? If so, how good?

(Redirected from here (http://www.giantitp.com/forums/showthread.php?549490-New-Armor-Material))

Armor isn't brittle for a reason; corundum would be terrible.

Ironsmith
2018-01-28, 12:07 PM
Armor isn't brittle for a reason; corundum would be terrible.

Somehow, I don't think that's due consideration to the question I asked. Corundum's only brittle in the sense that it breaks apart instead of bending, but actually breaking it isn't easy to do. Unlike diamond, and a lot of other gemstones for that matter, corundum has no cleavage planes (though it does exhibit parting), meaning there aren't a lot of low-resistance ways of cutting through it. Plus, it has a very high resistance to deformation (though also in fairness, if it deforms and you're counting on it to stop a projectile, you're probably screwed) and a compressive strength about twice that of your average sample of stainless steel.

So basically, what I'm saying is I was kind of hoping for a better answer than "no, it's brittle".

Galloglaich
2018-01-28, 12:29 PM
Somehow, I don't think that's due consideration to the question I asked. Corundum's only brittle in the sense that it breaks apart instead of bending, but actually breaking it isn't easy to do. Unlike diamond, and a lot of other gemstones for that matter, corundum has no cleavage planes (though it does exhibit parting), meaning there aren't a lot of low-resistance ways of cutting through it. Plus, it has a very high resistance to deformation (though also in fairness, if it deforms and you're counting on it to stop a projectile, you're probably screwed) and a compressive strength about twice that of your average sample of stainless steel.

So basically, what I'm saying is I was kind of hoping for a better answer than "no, it's brittle".

"no it's brittle"

I think in general, people fail to grasp how important flexibility is for both armor and weapons. More so for things like swords, obviously but for armor too. Bronze, copper, iron, and brass will all deform substantially before fracturing. Steel, if properly heat treated, will bend back so it's even better. That is what allows it to be so thin.

I am not an engineer or a materials scientist, but I think you'll find that brittle material makes poor armor - even if it's very hard. Your best bet would probably be to make lamellar but even then, I think you'd have problems. You have to compete with metals that can protect when only 1 -2mm thick. Of course, it is also possible to temper glass and make it much stronger. My guess is that you'd end up with tiny cracks pretty quickly which would only get worse over time.

Incidentally, by the Late Medieval period I believe they could have made at least lamellar out of beryl though I don't know how expensive it would be. Beryl was used to make lenses for eyeglasses fairly routinely so presumably you could make scales or lames out of it, though not sure about something the size of a breast plate. Beryl eyeglasses were common enough that burghers and even more well off peasants could afford them by the 15th Century.

PhoenixPhyre
2018-01-28, 12:56 PM
Another issue is workability. You can't easily join plates of corundum together with bolts or welds, and you can't easily fasten straps to it.

Ironsmith
2018-01-28, 01:10 PM
"no it's brittle"

I think in general, people fail to grasp how important flexibility is for both armor and weapons. More so for things like swords, obviously but for armor too. Bronze, copper, iron, and brass will all deform substantially before fracturing. Steel, if properly heat treated, will bend back so it's even better. That is what allows it to be so thin.

I am not an engineer or a materials scientist, but I think you'll find that brittle material makes poor armor - even if it's very hard. Your best bet would probably be to make lamellar but even then, I think you'd have problems. You have to compete with metals that can protect when only 1 -2mm thick. Of course, it is also possible to temper glass and make it much stronger. My guess is that you'd end up with tiny cracks pretty quickly which would only get worse over time.

Incidentally, by the Late Medieval period I believe they could have made at least lamellar out of beryl though I don't know how expensive it would be. Beryl was used to make lenses for eyeglasses fairly routinely so presumably you could make scales or lames out of it, though not sure about something the size of a breast plate. Beryl eyeglasses were common enough that burghers and even more well off peasants could afford them by the 15th Century.

(First off, much better; my frustration was more with the lack of effort in response than the answer itself. Thanks for taking the time out to write this.)

You're not the first person to suggest lamellar armor, and I'm starting to think that while it might not be as flashy as a full plate set of corundum would be, it'd probably be more practical (I imagine replacing cracked lamellar plate would be much easier than, say, repairing a shattered chestplate). Somehow working more elasticity into the design might be beneficial, as well.

Incidentally also, most samples of corundum that we have in ruby or sapphire form are only a few centimeters thick to begin with, so that kind of scale isn't exactly news.


Another issue is workability. You can't easily join plates of corundum together with bolts or welds, and you can't easily fasten straps to it.

Presumably, that's the kind of thing that'd be worked out in making corundum crystals that are big enough to be considered for deflecting sword strokes, crossbow bolts and whatnot. In a more fantastic setting, it wouldn't be inconceivable that the crystals themselves would be grown onto the belts themselves, or somehow set up so they'd have holes for bolts and rivets in them without disturbing/potentially breaking the material trying to put them there.

Haighus
2018-01-28, 01:43 PM
The material could also be used in a composite manner in the same way as ceramic plates in modern armour. If it was possible to create, for example, two thin plates of tempered steel with a thin layer of corundum sandwiched between, over typical padding, this would give many of the benefits of just corundum or just steel. To question is whether this would be more effective than the same thickness of just tempered steel. Also agree that lamellar armour would be the best way to field it, as such damaged lames would need to be replaced fairly frequently.

Carl
2018-01-28, 04:39 PM
It took a lot of digging to find any values i could compare with data i found for Corundum but the TLDR is it's MUCH easier to fracture it than even aluminium alloy which generally hold up less well than steel, (most tests are done using fixed sample sizes, and shapes, Aluminum is better than steel by weight but not normally in most respects by volume). I'm having real issues with the steel properties as they typically seem to measure toughness using a different test to that used with Corundum so i can't directly compare those two, the aluminum ones where an accidental find. But overall i'd say your looking at a very significant increase in ease of causing it to break into multiple pieces, the ratio for aluminium is over 4 times harder to shatter. Steel as noted is likely higher still.

Carl
2018-01-28, 04:43 PM
The material could also be used in a composite manner in the same way as ceramic plates in modern armour. If it was possible to create, for example, two thin plates of tempered steel with a thin layer of corundum sandwiched between, over typical padding, this would give many of the benefits of just corundum or just steel. To question is whether this would be more effective than the same thickness of just tempered steel. Also agree that lamellar armour would be the best way to field it, as such damaged lames would need to be replaced fairly frequently.

I did run across some indications it had at least been investigated for use in modern armour systems and we can actually manufacture synthetic stuff at larger sizes. Thee ws a lot of stuff about using it in vapor deposition based composite coatings for unclear applications.

Mr Beer
2018-01-28, 05:13 PM
The reason that arms and armour used steel instead of some exotic wonder material is that steel basically is the wonder material. I think if we had to make swords, crossbows, gothic plate, halberds etc. now, we could do better than the wood, leather and cloth components but we'd probably end up keeping most of the steel.

EDIT

Armour we would presumably use layered composites. I don't know if we'd keep the steel shell or not. Swords would likely remain all steel. Beaked weapons might employ spike made of tungsten or some other hard, high density material as a penetrator. Modern crossbows still use steel I believe.

Ironsmith
2018-01-28, 05:45 PM
It took a lot of digging to find any values i could compare with data i found for Corundum but the TLDR is it's MUCH easier to fracture it than even aluminium alloy which generally hold up less well than steel, (most tests are done using fixed sample sizes, and shapes, Aluminum is better than steel by weight but not normally in most respects by volume). I'm having real issues with the steel properties as they typically seem to measure toughness using a different test to that used with Corundum so i can't directly compare those two, the aluminum ones where an accidental find. But overall i'd say your looking at a very significant increase in ease of causing it to break into multiple pieces, the ratio for aluminium is over 4 times harder to shatter. Steel as noted is likely higher still.

Don't suppose you have a source? I could really use something like that...


The reason that arms and armour used steel instead of some exotic wonder material is that steel basically is the wonder material. I think if we had to make swords, crossbows, gothic plate, halberds etc. now, we could do better than the wood, leather and cloth components but we'd probably end up keeping most of the steel.

EDIT

Armour we would presumably use layered composites. I don't know if we'd keep the steel shell or not. Swords would likely remain all steel. Beaked weapons might employ spike made of tungsten or some other hard, high density material as a penetrator. Modern crossbows still use steel I believe.

We've actually picked up a few more wonder materials since then (ironically, including alumina, chemically the same stuff as corundum but structurally less specific), but yeah, I see your point. It also probably doesn't help that, for any given material we'd want to shape, we'd have to find a way to break it first...

Haighus
2018-01-28, 05:56 PM
It took a lot of digging to find any values i could compare with data i found for Corundum but the TLDR is it's MUCH easier to fracture it than even aluminium alloy which generally hold up less well than steel, (most tests are done using fixed sample sizes, and shapes, Aluminum is better than steel by weight but not normally in most respects by volume). I'm having real issues with the steel properties as they typically seem to measure toughness using a different test to that used with Corundum so i can't directly compare those two, the aluminum ones where an accidental find. But overall i'd say your looking at a very significant increase in ease of causing it to break into multiple pieces, the ratio for aluminium is over 4 times harder to shatter. Steel as noted is likely higher still.
Interesting, thanks. Presumably ceramic won out for composites, at least with current tech. I suspect the thing about steel could be due to steel having so many potential properties depending on the particular alloy in question, from hard and brittle to soft and malleable to soft and springy to hard and springy. I think this is why steel is really unusual and great for swords?

The reason that arms and armour used steel instead of some exotic wonder material is that steel basically is the wonder material. I think if we had to make swords, crossbows, gothic plate, halberds etc. now, we could do better than the wood, leather and cloth components but we'd probably end up keeping most of the steel.

EDIT

Armour we would presumably use layered composites. I don't know if we'd keep the steel shell or not. Swords would likely remain all steel. Beaked weapons might employ spike made of tungsten or some other hard, high density material as a penetrator. Modern crossbows still use steel I believe.I reckon the tungsten would be best in the core, like how bronze was used in the core of mace heads. Steel would still be better for the flanges/points, as it would be more durable, but you still have the density of the tungsten in the impact.

I reckon the only (currently known) material that has the potential to supplant steel in many of these roles is carbon materials specifically crafted with the optimum nano-structure? They may make better armour once people can produce them easily in the ways they want to, with accurate nano structures.

rrgg
2018-01-28, 07:24 PM
You could try covering medieval armor with ERA panels so that when it gets hit by an arrow it explodes and sends the arrow flying back at whoever shot it.

Mr Beer
2018-01-28, 07:31 PM
I reckon the tungsten would be best in the core, like how bronze was used in the core of mace heads. Steel would still be better for the flanges/points, as it would be more durable, but you still have the density of the tungsten in the impact.

I thought tungsten would be a good penetrating tip due to its extreme density and hardness. But maybe it would be too brittle for battlefield use.

I know we use it as a kinetic penetrator in modern weapons but I believe very high speed collisions work more like an explosive than a bullet and by increasing density, you get a bigger explosion for your projectile size.

So it may not be particularly great at punching through steel at the kind of speeds you get by swinging a polearm.



I reckon the only (currently known) material that has the potential to supplant steel in many of these roles is carbon materials specifically crafted with the optimum nano-structure? They may make better armour once people can produce them easily in the ways they want to, with accurate nano structures.

Yeah probably. I would like to see some of the structures that cheap nano-carbon could build. Mainly space elevators.

Galloglaich
2018-01-28, 07:50 PM
I thought tungsten would be a good penetrating tip due to its extreme density and hardness. But maybe it would be too brittle for battlefield use.


Nothing wrong with tungsten for military purposes, it's been used as armor-piercing ammunition since WW2, both for AP and APDS (armor piercing discarding sabot) and various similar types. I think the US switched to depleted uranium for a while in Iraq, not sure what they are using now.

Haighus
2018-01-28, 08:30 PM
The requirements are different though. The tungsten used in kinetic penetrators is for one use only, and the tungsten is destroyed in the process. A mace needs to be able to collide with targets all day and not shatter or deform. I still think a tungsten core with steel flanges would be more useful than a pure tungsten mace.

Max_Killjoy
2018-01-28, 08:40 PM
Nothing wrong with tungsten for military purposes, it's been used as armor-piercing ammunition since WW2, both for AP and APDS (armor piercing discarding sabot) and various similar types. I think the US switched to depleted uranium for a while in Iraq, not sure what they are using now.

Still DU, it has some rather different properties (pyrophoric, and self-sharpens by fracture instead of deforming during impact, for example).

Clistenes
2018-01-29, 03:14 AM
The requirements are different though. The tungsten used in kinetic penetrators is for one use only, and the tungsten is destroyed in the process. A mace needs to be able to collide with targets all day and not shatter or deform. I still think a tungsten core with steel flanges would be more useful than a pure tungsten mace.

I think depth of penetration of a projectile depends on density only (Isaac Newton made an algorithm to calculate that...); the projectile itself may disintegrate or not depending on its resilience, and excess energy from high speed is dispersed as heat and kinetic waves that damage the target...

So a high density projectile has more chances to punch through armor, even is it disintegrates into tiny chunks while doing so...

Haighus
2018-01-29, 03:32 AM
I think depth of penetration of a projectile depends on density only (Isaac Newton made an algorithm to calculate that...); the projectile itself may disintegrate or not depending on its resilience, and excess energy from high speed is dispersed as heat and kinetic waves that damage the target...

So a high density projectile has more chances to punch through armor, even is it disintegrates into tiny chunks while doing so...

Yes, but this is only useful for a one shot projectile. Anyone whose mace becomes useless after a couple of bashes has no mace anymore. It is the same as having a very hard sword- it is able to cut really well, but it is too fragile, so people used ones that are more springy.

Carl
2018-01-29, 06:42 AM
Corundum data source: http://www.matweb.com/search/datasheet.aspx?matguid=c8c56ad547ae4cfabad15977bfb 537f1&ckck=1

The stuff i found on aluminium fracture stress: http://beyond-steel.com/product/

S355 steel, (the supposed closest to medieval quality stuff): http://www.matweb.com/search/DataSheet.aspx?MatGUID=4f4caff896374b718d02f641ff2 ae644

Brother Oni
2018-01-29, 07:53 AM
I think corundum could be potentially useful as a limited use armour, much like modern ceramic armours on AFVs or the ceramic plates on body armour. It would be very effective at dispersing the energy from a single hit that defeats it as all the energy would be re-directed into shattering it, rather than it carrying through into the person.

The only issues are that after that single hit, you now have a gap in your armour where that corundum plate was, or worse, a hit that causes the plate to fracture but not shatter, so a second hit on the damaged plate ploughs straight through.

In my opinion, it would be great for very short duration encounters (assassinations/ambushes and the like), not so good if you're spending all day on the battlefield slugging it out on the front lines.

Clistenes
2018-01-29, 08:43 AM
Yes, but this is only useful for a one shot projectile. Anyone whose mace becomes useless after a couple of bashes has no mace anymore. It is the same as having a very hard sword- it is able to cut really well, but it is too fragile, so people used ones that are more springy.

I was speaking of why tungsten was used for anti-tank projectiles, not of a theorical mace made of tungsten...

Ironsmith
2018-01-29, 08:48 AM
Corundum data source: http://www.matweb.com/search/datasheet.aspx?matguid=c8c56ad547ae4cfabad15977bfb 537f1&ckck=1

The stuff i found on aluminium fracture stress: http://beyond-steel.com/product/

S355 steel, (the supposed closest to medieval quality stuff): http://www.matweb.com/search/DataSheet.aspx?MatGUID=4f4caff896374b718d02f641ff2 ae644

Huh. I tried matweb, but I didn't see the data you mentioned... guess we know who failed his spot check, huh? :smalltongue:


I think corundum could be potentially useful as a limited use armour, much like modern ceramic armours on AFVs or the ceramic plates on body armour. It would be very effective at dispersing the energy from a single hit that defeats it as all the energy would be re-directed into shattering it, rather than it carrying through into the person.

The only issues are that after that single hit, you now have a gap in your armour where that corundum plate was, or worse, a hit that causes the plate to fracture but not shatter, so a second hit on the damaged plate ploughs straight through.

In my opinion, it would be great for very short duration encounters (assassinations/ambushes and the like), not so good if you're spending all day on the battlefield slugging it out on the front lines.

Combined with the light weight of the armor itself (half the density of steel), that might make it good-ish armor for skirmishers and such, yeah? Mechanically speaking, I mean... I'm sure it's pretty hard to sneak up on someone while wearing a highly visible ruby.

Lapak
2018-01-29, 10:03 AM
Combined with the light weight of the armor itself (half the density of steel), that might make it good-ish armor for skirmishers and such, yeah? Mechanically speaking, I mean... I'm sure it's pretty hard to sneak up on someone while wearing a highly visible ruby.It’s actually sounding more and more like a specialty armor marketed to an obscenely decadent upper class as anti-assassination chic.

“Yes, I was just hearing about it. It’s a new technique, they call it ‘ablative armor.’ It’s the latest fashion.”

“That sounds perfect, my darling. Jenkins! Send my sizes to the armorer.”

“It gets better, dearest.”

“Oh?”

“The armor is made entirely from precious gems. And it works perfectly just once, then you have to throw it away.”

“*swoon*”

gkathellar
2018-01-29, 10:21 AM
All of these materials with a mix of good and bad properties really sound like they could be effective if we alloyed them with iron, and maybe each other, in small quantities. Hm, I wonder why no one ever thought of that before ...

Seriously, though, steel is the wonder metal at least in part because the word is ambiguous by definition. "Iron and other stuff" is a pretty good response to this kind of materials science question.

The other thing to bear in mind is that a lot of your tungstens and iridiums are phenomenally expensive and/or heavy. It's generally more efficient to alloy small quantities of these and derive significant benefits at minimal cost. Others, like molybdenum or titanium, really don't show their utility until they're alloyed in the first place.

The answer is steel, whatever that even means.

Haighus
2018-01-29, 10:45 AM
I was speaking of why tungsten was used for anti-tank projectiles, not of a theorical mace made of tungsten...

Ah, apologies, I thought you were referring to tungsten in the context of modern materials for medieval weaponry, which is what spawned the tungsten discussion.

Galloglaich
2018-01-29, 12:02 PM
I wonder if anyone can tell me anything about the Mangual, or Iberian flail, which looks similar to the Czech Flegel. Who used it (what kind of troops or estates), how common was it, was it known in any battles?

http://www.flickr.com/photos/98015679@N04/9261024278/in/set-72157634584454532

http://www.flickr.com/photos/98015679@N04/9257840832/in/set-72157634584454532

G

Brother Oni
2018-01-29, 12:04 PM
Combined with the light weight of the armor itself (half the density of steel), that might make it good-ish armor for skirmishers and such, yeah? Mechanically speaking, I mean... I'm sure it's pretty hard to sneak up on someone while wearing a highly visible ruby.

I wouldn't use density as the sole measurement for comparison as armour durability is highly dependent on the material's molecular structure and properties. An armour plate of good quality steel ranges from 1-2mm thickness; I doubt a similar thickness ruby sheet would be as effective, so would have to be thicker to obtain the same protective capabilities.

Unfortunately, I'm not that well versed in materials science, so I couldn't tell you how thick a ruby plate you'd need to have the same energy dispersing capability as a similar area sheet of 2 mm good quality steel.

I do like Lapak's suggestion of it being used as 'bling' armour, particularly when there's historical precedent for the concept:

https://upload.wikimedia.org/wikipedia/commons/a/a9/Henry_VIII_armor_by_Matthew_Bisanz.JPG
https://farm3.static.flickr.com/2405/5707804254_1884eee998_b.jpg

DrewID
2018-01-29, 02:40 PM
Yes, but this is only useful for a one shot projectile. Anyone whose mace becomes useless after a couple of bashes has no mace anymore. It is the same as having a very hard sword- it is able to cut really well, but it is too fragile, so people used ones that are more springy.

Oh, come on! Are you seriously telling me that if you put "Exploding Mace" on the price list for your RPG, the PCs wouldn't be lining up to buy out the local stockpiles?

DrewID

Carl
2018-01-29, 02:57 PM
Huh. I tried matweb, but I didn't see the data you mentioned... guess we know who failed his spot check, huh? :smalltongue:



Combined with the light weight of the armor itself (half the density of steel), that might make it good-ish armor for skirmishers and such, yeah? Mechanically speaking, I mean... I'm sure it's pretty hard to sneak up on someone while wearing a highly visible ruby.


I got it via a direct link to the page from google search, the built in search function is terrible.

Galloglaich
2018-01-29, 03:04 PM
In the middle ages they used to use (I think cast) iron bullets for armor-piercing ammunition in firearms going back to the 1300's.

So it seems to me that tungsten ammunition might be appreciated. Both for firearms bullets and for 'cannister' type shot for cannons.

Conversely, ceramic or even beryl armor as a kind of one-off defense against powerful firearms, muskets etc., maybe for some kind of elite one-time use like a hit-squad is a kind of interesting idea. I mean, it's used today. Could find a niche. Seems like it would be extra vulnerable to a lot of hand weapons for example, hammers and maces. Probably axes, maybe swords and spears too.

And I think in thicknesses being worth use- it would be heavier and bulkier than armor people were used to. However anywhere that high energy muskets are a threat anything that can save you might be appreciated.

G

Haighus
2018-01-29, 03:15 PM
Oh, come on! Are you seriously telling me that if you put "Exploding Mace" on the price list for your RPG, the PCs wouldn't be lining up to buy out the local stockpiles?

DrewID
:smallbiggrin:


In the middle ages they used to use (I think cast) iron bullets for armor-piercing ammunition in firearms going back to the 1300's.

So it seems to me that tungsten ammunition might be appreciated. Both for firearms bullets and for 'cannister' type shot for cannons.

Conversely, ceramic or even beryl armor as a kind of one-off defense against powerful firearms, muskets etc., maybe for some kind of elite one-time use like a hit-squad is a kind of interesting idea. I mean, it's used today. Could find a niche. Seems like it would be extra vulnerable to a lot of hand weapons for example, hammers and maces. Probably axes, maybe swords and spears too.

And I think in thicknesses being worth use- it would be heavier and bulkier than armor people were used to. However anywhere that high energy muskets are a threat anything that can save you might be appreciated.

G
This all makes sense. Could be useful for generals who don't engage in combat? They are unlikely to be engaged in melee, but could need protection against ooportune shots. Especially in sieges. I believe quite a few Maltese commanders were sniped in the Siege of Malta?

Galloglaich
2018-01-29, 03:47 PM
:smallbiggrin:


This all makes sense. Could be useful for generals who don't engage in combat? They are unlikely to be engaged in melee, but could need protection against ooportune shots. Especially in sieges. I believe quite a few Maltese commanders were sniped in the Siege of Malta?

Yes indeed, there is this curious thing in the Late Medieval and Early Modern era that in spite of the general rule that early muzzle loading firearms were fairly inaccurate and had fairly short effective range, there were clearly exceptions to this. We can see this in records of shooting contests sponsored by the towns in Italy and Central Europe, and on the battlefield in places like Malta and also famously in the siege of Rome where the artist Buenvenutto Cellini killed at least one, possibly two senior commanders of the besieging army (ironically perhaps contributing to the brutal sack of Rome).

IIRC at Malta you had about 50 'marksmen' on both sides who could hit individual targets at 200 or 300 meters or maybe even more, whereas the typical handgunner could only hit targets ~50 -100 meters away.

I think Cellini actually got one of his victims with a long-barreled cannon.

G

Carl
2018-01-29, 04:07 PM
I think Cellini actually got one of his victims with a long-barreled cannon.


Sounds like an earlier version of Hathcock and his scoped M2 machine gun.

Ironsmith
2018-01-29, 05:39 PM
One more permutation for this... how effective would corundum be as a sort of glaze? Regardless of its use, it'd almost certainly be a vanity thing, as a glaze-thin layer of corundum probably won't affect the overall durability of the armor itself that much...

Haighus
2018-01-29, 05:44 PM
Yes indeed, there is this curious thing in the Late Medieval and Early Modern era that in spite of the general rule that early muzzle loading firearms were fairly inaccurate and had fairly short effective range, there were clearly exceptions to this. We can see this in records of shooting contests sponsored by the towns in Italy and Central Europe, and on the battlefield in places like Malta and also famously in the siege of Rome where the artist Buenvenutto Cellini killed at least one, possibly two senior commanders of the besieging army (ironically perhaps contributing to the brutal sack of Rome).

IIRC at Malta you had about 50 'marksmen' on both sides who could hit individual targets at 200 or 300 meters or maybe even more, whereas the typical handgunner could only hit targets ~50 -100 meters away.

I think Cellini actually got one of his victims with a long-barreled cannon.

G

On the subject of old firearms, I've just come across this fabulous example of a 17th century smoothbore. Simply breathtaking. I am really falling in love with these old guns.


https://www.youtube.com/watch?v=beOgmCxeh7A

Carl
2018-01-30, 07:10 AM
One more permutation for this... how effective would corundum be as a sort of glaze? Regardless of its use, it'd almost certainly be a vanity thing, as a glaze-thin layer of corundum probably won't affect the overall durability of the armor itself that much...

It might actually help, a thin glaze assuming it was thin enough probably wouldn't break over a large area on any hit and the extra surface hardness would tend to inhibit a lot of things. However it would likely be very rough so mishandling could result in nasty injuries.

Brother Oni
2018-01-30, 07:36 AM
It might actually help, a thin glaze assuming it was thin enough probably wouldn't break over a large area on any hit and the extra surface hardness would tend to inhibit a lot of things. However it would likely be very rough so mishandling could result in nasty injuries.

You would also have to be very careful that the shattering crystal not send razor sharp shards everywhere, including into you.

It's one of the issue the conquistadors had against the Incan obsidian arrow heads, as they had a tendency to shatter on their metal armour, with the glass fragments potentially causing more grievous wounds than an intact arrow itself.

wolflance
2018-01-31, 04:25 AM
The Chinese had an advantage of 17 to 1, and the Dutch stood their ground until their rear was taken by surprise... if that battle proves something, it is the superiority of guns over bows...

I mean, if a sword-wielding guy keeps at bay 17 other guys armed with bricks in socks until one of them manages to sneak from behind and hit the back of his head, that doesn't mean the brick and sock are more powerful than the sword, it just means that he lost to an overwhelming numerical advantage and a surprise attack...
Actually the Chinese side of the story clearly shows that it was a gun vs gun battle. And Koxinga only had like 1,000 ~ 1,200 troops in that battle (500 gunners, 200 small cannons, 500 ambushers. Uncertain if the 200 small cannons count towards the initial 500 gunners, or count as separate unit). Bow wasn't even mentioned as far as I know (sure they had bows, but bows probably didn't factor much in their tactical plannin).

rrgg
2018-01-31, 04:38 AM
Yes indeed, there is this curious thing in the Late Medieval and Early Modern era that in spite of the general rule that early muzzle loading firearms were fairly inaccurate and had fairly short effective range, there were clearly exceptions to this. We can see this in records of shooting contests sponsored by the towns in Italy and Central Europe, and on the battlefield in places like Malta and also famously in the siege of Rome where the artist Buenvenutto Cellini killed at least one, possibly two senior commanders of the besieging army (ironically perhaps contributing to the brutal sack of Rome).

IIRC at Malta you had about 50 'marksmen' on both sides who could hit individual targets at 200 or 300 meters or maybe even more, whereas the typical handgunner could only hit targets ~50 -100 meters away.

I think Cellini actually got one of his victims with a long-barreled cannon.

G

If Barnabe Rich is to be believed, then according to those who fought in the low countries, the single greatest advantage of the full sized musket was that it could be used almost like artillery, to create a beaten ground against formations of horsemen and footmen 480-600 yards away.

" in truth one of the most especiall causes that muskets are so much regarded, is because they may be brought 24 and 30 scores off to beate upon squadrons either of horsemen or footmen, to breake and dismember them: and in like maner to beate passages or grounds of advantage taken by the enemy. . . but for those that do no better valew of the musket, but to give their volies at tenne, twenty, or thirtie paces: it should seeme they knew of no other service in the field. . . captains that be of experience are accustomed to place the stand of pikes (wherein consisteth their strength) upon some ground of advantage, and as neere as they can will bring some hedge, some ditch, some shrubbes or bushes, or some other like helpes betweene them and the enemy, because they would not lie open to the musket shot"

"

Clistenes
2018-01-31, 06:36 AM
Actually the Chinese side of the story clearly shows that it was a gun vs gun battle. And Koxinga only had like 1,000 ~ 1,200 troops in that battle (500 gunners, 200 small cannons, 500 ambushers. Uncertain if the 200 small cannons count towards the initial 500 gunners, or count as separate unit). Bow wasn't even mentioned as far as I know (sure they had bows, but bows probably didn't factor much in their tactical plannin).

The text I read said it were more than 4,000 chinese troops vs 240 or less dutchmen...

gkathellar
2018-01-31, 10:16 AM
The text I read said it were more than 4,000 chinese troops vs 240 or less dutchmen...

Was it a Dutch text?

(If you gave me those numbers measuring astronauts modern infantry fighting against cavemen, I'd still raise my eyebrows.)

Brother Oni
2018-01-31, 12:06 PM
(If you gave me those numbers measuring astronauts modern infantry fighting against cavemen, I'd still raise my eyebrows.)

There's the Siege of the Internation Legations during the Boxer War (https://en.wikipedia.org/wiki/Siege_of_the_International_Legations), which pitted 409 soldiers and 150 civilian volunteers versus somewhere in the region of 20,000 Chinese rebels.

Even taking into account that the unit 10,000 is often a poetic Chinese term meaning 'lots and lots' rather than an actual figure, that's still a lot of troops, showcasing the power of modern weaponry, better trained troops and fortified positions (the Western powers, Russian and Japanese troops were equipped with a variety of bolt action repeating rifles, while the Chinese rebels had single shot Martini Henrys or Martini Metfords, or just armed with melee weapons).

While searching, I found this (most likely apocryphal) anecdote regarding how the various relief forces managed to get inside Peking to relive the siege:

- The Russians love artillery so they wheeled up their fieldpieces and blew it open.
- The Japanese love insane bravery: one man rushed forward with a satchel charge and was shot down. Another man rushed forward, picked it up, and was shot down. Repeat until the charge was finally placed to blow open the gate.
- The Americans love individual guts: two men snuck up to the wall by night, scaled it and opened the gate.
- The British are just stupid-lucky: they found their gate unguarded and marched through with bands and bagpipes playing.

rrgg
2018-01-31, 01:26 PM
In the middle ages they used to use (I think cast) iron bullets for armor-piercing ammunition in firearms going back to the 1300's.

So it seems to me that tungsten ammunition might be appreciated. Both for firearms bullets and for 'cannister' type shot for cannons.

Conversely, ceramic or even beryl armor as a kind of one-off defense against powerful firearms, muskets etc., maybe for some kind of elite one-time use like a hit-squad is a kind of interesting idea. I mean, it's used today. Could find a niche. Seems like it would be extra vulnerable to a lot of hand weapons for example, hammers and maces. Probably axes, maybe swords and spears too.

And I think in thicknesses being worth use- it would be heavier and bulkier than armor people were used to. However anywhere that high energy muskets are a threat anything that can save you might be appreciated.

G

You know, as long as we're on the subject of exotic materials used for bullets it seems that gold and silver actually were considered for either small arms or cannons, if not really taken seriously. From Turner's Pallas Armata, page 192:

"Bullets for any kind of Ordnance or Fire-Guns, may be of any metal you please, yea of Gold or Silver; the first is too costly; the second some fancy to be able to pierce such as are (by some black art or other) hard, or Bullet-proof. But to charge a person that is Bullet-proof, with a Silver-ball, to me seems to be like the Assaulting an Inchanted Castle."

gkathellar
2018-01-31, 01:27 PM
Oh, I'm not saying the defeat of a vastly numerically superior force never happens, just that it's cause to look for corroborating sources.

Haighus
2018-01-31, 02:44 PM
The smaller Dutch force did also get promptly routed by the superior Chinese forces after they underestimated Chinese resolve...

Clistenes
2018-01-31, 02:45 PM
Was it a Dutch text?

(If you gave me those numbers measuring astronauts modern infantry fighting against cavemen, I'd still raise my eyebrows.)

Well, the Dutch claimed that a well-armed company could face off a Chinese troop 25 times larger, if the latter were armed just with bows, swords, polearms and paper armor. It seems they were very confidend their technological advantage could beat the numbers of the Chinese...


There's the Siege of the Internation Legations during the Boxer War (https://en.wikipedia.org/wiki/Siege_of_the_International_Legations), which pitted 409 soldiers and 150 civilian volunteers versus somewhere in the region of 20,000 Chinese rebels.

Even taking into account that the unit 10,000 is often a poetic Chinese term meaning 'lots and lots' rather than an actual figure, that's still a lot of troops, showcasing the power of modern weaponry, better trained troops and fortified positions (the Western powers, Russian and Japanese troops were equipped with a variety of bolt action repeating rifles, while the Chinese rebels had single shot Martini Henrys or Martini Metfords, or just armed with melee weapons).

While searching, I found this (most likely apocryphal) anecdote regarding how the various relief forces managed to get inside Peking to relive the siege:

- The Russians love artillery so they wheeled up their fieldpieces and blew it open.
- The Japanese love insane bravery: one man rushed forward with a satchel charge and was shot down. Another man rushed forward, picked it up, and was shot down. Repeat until the charge was finally placed to blow open the gate.
- The Americans love individual guts: two men snuck up to the wall by night, scaled it and opened the gate.
- The British are just stupid-lucky: they found their gate unguarded and marched through with bands and bagpipes playing.

The Spaniards were in good terms with the Chinese government, so they spent most of the war acting go-betweens for the Chinese and Europeans... the spanish ambassador, Bernardo de Cólogan y Cólogan was the last foreign diplomat to be allowed access to the Forbidden City, had a key role in the redaction of the Boxer Protocol, which was finally signed in the Spanish Embassy...

Benardo de Cólogan y Cologán was awarded the Great Cross of the Red Eagle from Prussia, the Great Cross of the Order of Saint Anna from Russia, the Great Cross of the Order of the Polar Star from Sweden, the National Order of the Legion of Honour from France and the Great Cross of Military Merit from Spain to show their gratitude and as recognition for his role...

Great Britain, on the other hand, utterly ignored him, and tried to erase him from history and pretend that their own diplomats had done all the work and that the Boxer Protocol had been signed in their own embassy... Oh, and all the troops from the other countries were there mostly to watch the British troops fight and cheer them from afar! :smalltongue:

wolflance
2018-01-31, 11:27 PM
The text I read said it were more than 4,000 chinese troops vs 240 or less dutchmen...
Yes. That's from the Dutch side of the record. There are numerous accounts for that battle, including at least two Ming ones as well as several European (three from the Dutch and one I believe from a Swiss working for the VOC, written in German).

Detail comparison:
1) (Dutch, with English translation) Neglected Formosa: 4,000 Chinese vs 240 Dutch, Chinese using bows & arrows.

2) (Dutch) De Dagregisters van het Kasteel Zeelandia: "unspecified number of Chinese troops from three junks + unspecified reinforcement" vs 250 Dutch + several Dutch ships and sampans (using swivel guns on the boats for fire support). Chinese using small cannons from breastwork.

3) (Swiss) Reise nach Java, Formosa, Vorder-Indien und Ceylon: Numerous Chinese junks + unspecified Chinese troops from the junks, 200 Dutch and three Dutch ships + several small Dutch boats and sampans (fire support). Chinese had 50 or more "Passen" or "Doppelhaggen" (swivel gun and wall gun) deployed on breastwork, as well as rockets (feurpfejlen).

4) (Chinese) 臺灣外記: 500 Chinese gunners, 200 small cannons, 500 ambushers (rattan shieldmen), twenty small junks (that ignored the battle and attacked Dutch fortress directly as diversion to distract Dutch troops on the field) vs unspecified number of Dutch troops.

5) (Chinese) 從征實錄: Unspecified number of Chinese troops vs "several hundred" Dutch. Details of the battle not recorded.

6) (Dutch) Diary of Philippus Daniel Meij van Meijensteen, a Dutch geographer captured by Koxinga for nearly the entire duration of the war, and acted as Koxinga's translator/land surveyor: "Numerous Chinese junks" vs three Dutch ships. Account of land battle not given.


Only one account mention Chinese using arrow at all (which unfortunately is the most well-known one, as it is translated to English), while others describe the Chinese using firearms. In any case, Chinese couldn't have fielded 4,000 troops using only small junks as transport, but all accounts otherwise mostly agree with each others on the more important events (Thomas Pedel as the Dutch leader, Chinese ambush with swords, a Dutch ship named Hector got blown to pieces etc).

Incanur
2018-02-01, 10:23 PM
Well, the Dutch claimed that a well-armed company could face off a Chinese troop 25 times larger, if the latter were armed just with bows, swords, polearms and paper armor.

One Dutch source (https://books.google.com/books?id=cRXAcZGcpa8C&pg=PA218&lpg=PA218&dq=koxinga+archers+%2B+riflemen&source=bl&ots=dRdWD42b7s&sig=WX5eQWwRlCKAUE7oay_1JVsgYuY&hl=en&sa=X&ved=0ahUKEwjrr-u8oIbZAhVH1IMKHRgZCm8Q6AEISDAM#v=onepage&q=koxinga%20archers%20%2B%20riflemen&f=false) claims Koxinga's archers were comparable to Dutch riflemen. What's the source you're referring to?

A Chinese source from the same book describes how Ming troops who only had firearms tended to lose against to Manchu cavalry and their bows and arrows, which were deadly within thirty paces. The Chinese troops that did the best had thick sleeping quilts to defend them from arrows.

Taken as a whole, there's a mountain of evidence that bows were martially effective in China and surrounding regions at least through the 17th century. The Qing military used a combination of bows and firearms, in some cases equipping soldiers with both a bow and a firearm.

It's possible the Dutch source above is inaccurate, but I doubt it given the bow's success across the Chinese region in the 17th century.

wolflance
2018-02-01, 11:35 PM
One Dutch source (https://books.google.com/books?id=cRXAcZGcpa8C&pg=PA218&lpg=PA218&dq=koxinga+archers+%2B+riflemen&source=bl&ots=dRdWD42b7s&sig=WX5eQWwRlCKAUE7oay_1JVsgYuY&hl=en&sa=X&ved=0ahUKEwjrr-u8oIbZAhVH1IMKHRgZCm8Q6AEISDAM#v=onepage&q=koxinga%20archers%20%2B%20riflemen&f=false) claims Koxinga's archers were comparable to Dutch riflemen. What's the source you're referring to?
I believe the one cited by Clistenes and the one given by you are actually from the same source - the Neglected Formosa.

rrgg
2018-02-02, 01:05 AM
Taken as a whole, there's a mountain of evidence that bows were martially effective in China and surrounding regions at least through the 17th century. The Qing military used a combination of bows and firearms, in some cases equipping soldiers with both a bow and a firearm.

The arrows could still wound or kill, especially when combined with a fast horse to quickly get into range. It was also a weapon that they were already very familiar with and it didn't requite spending a lot of money on powder during training.

Incanur
2018-02-02, 01:41 AM
The narrative I recall, from reading a translation and secondary sources, is that the Dutch (particularly Captain Thomas Pedel) thought they could defeat much larger numbers of Chinese troops with ease until they faced Koxinga's (Zheng Chenggong's) hardened veterans. Then they lost.

Edit: Koxinga certainly had a massive numerical advantage & that was why his forces won so convincingly in open confrontations. The Dutch fortress held out for some time, because Dutch artillery was definitely better and it was a quality defensive position. There was at least one (I think nighttime?) skirmish were Dutch musketeers/riflemen did at least decently against opposing archers. But if you look at all the factors combined, it's a solid showing for the bow. Bows aren't optimized for sieges by any stretch of the imagination, but Koxinga's archers nonetheless served well enough to impress. Manchu/Qing armies used muskets and artillery, but archery remained a prized military skill even through the 19th century. It was laughable at the end, and presumably a matter of cultural inertia, but Manchu/Qing armies tended to win in the 17th and 18th centuries.

wolflance
2018-02-02, 10:16 AM
The narrative I recall, from reading a translation and secondary sources, is that the Dutch (particularly Captain Thomas Pedel) thought they could defeat much larger numbers of Chinese troops with ease until they faced Koxinga's (Zheng Chenggong's) hardened veterans. Then they lost.
Yes, that's what I think as well.

As a trivia, it is quite well-known that Koxinga was eventually deified by the Chinese, but lesser known is that several Hollanders are also deified (usually as lesser gate guardians that work for deified Koxinga). Among them also include a Dutch princess (because why not?) named Magriet.

Archpaladin Zousha
2018-02-02, 04:45 PM
The 19th Century adventurer, Sir Richard Burton, stated in his 1884 work The Book of the Sword that the yataghan was the best-designed sword out there. Was he accurate, or was that merely his opinion?

Max_Killjoy
2018-02-02, 04:51 PM
The 19th Century adventurer, Sir Richard Burton, stated in his 1884 work The Book of the Sword that the yataghan was the best-designed sword out there. Was he accurate, or was that merely his opinion?


Eh, it's a sword, it has tradeoffs with other sword designs.

The same basic forward-curve shape can be seen in the ancient xiphos or falcata, although the yataghan is a bit leaner.

If its design was good for the techniques it was used with, and the quality of construction was good, then it was a good sword. If the design was mismatched or the quality was poor, it was not so good.

Vinyadan
2018-02-03, 04:06 PM
Has anyone heard about the recent Status-6 buzz? https://en.wikipedia.org/wiki/Status-6_Oceanic_Multipurpose_System I don't know if it's actually doable, but it sounds like the subject of a cool storyline.

gkathellar
2018-02-03, 05:13 PM
The 19th Century adventurer, Sir Richard Burton, stated in his 1884 work The Book of the Sword that the yataghan was the best-designed sword out there. Was he accurate, or was that merely his opinion?

"This weapon is the best" assertions virtually always reflect the biases and experiences of the speaker, rather than any verifiable truth - at least in a dueling/skirmishing setting. Weapons exist in the context of the techniques employed with them, what they have to compete with, social factors surrounding their possession and use, and the defensive measures that can be taken against them.

Haighus
2018-02-03, 06:25 PM
Yeah, I think that at best, a weapon can be the most ideal form for a specific role, but that always comes with trade offs that make it perform worse in other situations.

Within this context, the yataghan could perhaps be considered the best form of cavalry sabre in the role of striking lightly armoured infantry from horseback, but I'm sure many would disagree with that (and did, considering how most sabers were shaped without the forward curve, and many had different hilts and guards). It is also far from ideal for fighting heavily armoured opponents, and poorly suited for thrusting.

It is not even the best pure cutting design, but it is a good design for cutting. Early falchions have blades even more specialised for raw slicing. I suspect the need for decent reach, and a blade that isn't too fragile, is the reason cavalry sabres don't have very wide blades like early falchions. Perhaps drawing the weapon from a scabbard and carrying it at the hip is also much more difficult with early falchions?

Gnoman
2018-02-03, 07:44 PM
Unless I'm mixing up my swords again, the yataghan was notable for being extremely light compared to most swords of the era. Provided there is enough structural strength to hold up to use, a light blade can be extremely powerful.

Brother Oni
2018-02-04, 08:14 AM
Has anyone heard about the recent Status-6 buzz? https://en.wikipedia.org/wiki/Status-6_Oceanic_Multipurpose_System I don't know if it's actually doable, but it sounds like the subject of a cool storyline.

I haven't, but I've just heard about the proposed new 'Sea Hunter' class unmanned ASW drone ship: link (https://www.digitaltrends.com/cool-tech/darpa-sea-hunter-joins-navy-fleet/).

I always found it interesting how weapons development mimicked the Red Queen hypothesis of evolutionary biology, where organisms are constantly adapting and evolving just to simply survive against other ever-evolving predators and competitor species, let alone gain an evolutionary advantage.

Carl
2018-02-04, 10:13 AM
I haven't, but I've just heard about the proposed new 'Sea Hunter' class unmanned ASW drone ship: link (https://www.digitaltrends.com/cool-tech/darpa-sea-hunter-joins-navy-fleet/).

I always found it interesting how weapons development mimicked the Red Queen hypothesis of evolutionary biology, where organisms are constantly adapting and evolving just to simply survive against other ever-evolving predators and competitor species, let alone gain an evolutionary advantage.

I'd be cautious of reading too much into that however. WHilst using a large number of smaller unmanned platforms is likely to seriously reduce manpower costs, it's nearly guarantee inflated electronics equipment costs as each platform needs certain things once over. So whilst it may save on operating costs it's likely to produce inflated build costs and potentially serious production rate bottlenecks.

Lapak
2018-02-04, 10:30 AM
I'd be cautious of reading too much into that however. WHilst using a large number of smaller unmanned platforms is likely to seriously reduce manpower costs, it's nearly guarantee inflated electronics equipment costs as each platform needs certain things once over. So whilst it may save on operating costs it's likely to produce inflated build costs and potentially serious production rate bottlenecks.
Feels like a nod to Superiority (https://www.goodreads.com/book/show/18038608-superiority) is obligatory here.

Carl
2018-02-04, 10:51 AM
It was a big part, (on the personel side), of that which did in the Japanese air power during WW2. They just couldn't train new pilots well enough, (unlike the allies they didn't pull veterans back to train new pilots, so the new recruits got very little first hand experiance), or fast enough.

And it's somthing that concerns me TBH about modern western militaries. Whilst most of what makes them tick strikes me as good practise, the sheer amount of time it takes to train new people and the expense of some pieces of kit makes me wonder if the system can really keep up with a sustained high intensity conflict. Most planning now seems to assume either short duration or low intensity with comparatively speaking low loss rates of men and material.

Haighus
2018-02-04, 11:14 AM
A lot of the equipment used by Western militaries is also designed with crew safety in mind though, so that the personnel loss rates are hopefully minimised.

Even if we look at WWII, most crew survived vehicles being knocked out. I think that has only gotten better since, especially with the widespread adoption of equipment like the BV reducing the risk to crew when on downtime.

Mike_G
2018-02-04, 01:40 PM
And it's somthing that concerns me TBH about modern western militaries. Whilst most of what makes them tick strikes me as good practise, the sheer amount of time it takes to train new people and the expense of some pieces of kit makes me wonder if the system can really keep up with a sustained high intensity conflict. Most planning now seems to assume either short duration or low intensity with comparatively speaking low loss rates of men and material.

For over sixty years, all our fights have been low intensity, so it makes sense to build better, safer, more expensive stuff in lower numbers so our troops better survive low intensity fights that we get in than build for hypothetical fights we haven't had in a long time.

The gear adapts to the mission we do. More defense against IEDs than MBTs makes a lot of sense for what we're doing right now. And fewer body bags coming home next year is a big deal.

If we fought another WWI or WWII, we couldn't sustain our current tech or training in massive numbers. But you see that even in the World Wars. Basic training was shortened, weapons were modified to be made faster and cheaper, like the stamped parts of the M3 SMG versus the expensive Thompson. Later Thompsons had simplified sights and so on.

But the big scary tech we do have make it unlikely that we'd fight a major war for every long. I can see a NATO/Russia conflict on th scale of WWII that doesn't go nuclear.

If the USSR has nukes in 1940, I think they'd have pushed the button waaaay before Stalingrad.

Carl
2018-02-04, 08:34 PM
For over sixty years, all our fights have been low intensity, so it makes sense to build better, safer, more expensive stuff in lower numbers so our troops better survive low intensity fights that we get in than build for hypothetical fights we haven't had in a long time.

The gear adapts to the mission we do. More defense against IEDs than MBTs makes a lot of sense for what we're doing right now. And fewer body bags coming home next year is a big deal.

If we fought another WWI or WWII, we couldn't sustain our current tech or training in massive numbers. But you see that even in the World Wars. Basic training was shortened, weapons were modified to be made faster and cheaper, like the stamped parts of the M3 SMG versus the expensive Thompson. Later Thompsons had simplified sights and so on.

But the big scary tech we do have make it unlikely that we'd fight a major war for every long. I can see a NATO/Russia conflict on th scale of WWII that doesn't go nuclear.

If the USSR has nukes in 1940, I think they'd have pushed the button waaaay before Stalingrad.

Don't get me wrong i'm not saying we couldn't adapt to such a fight, (or that i think it's likely, it's more the don;t get caught out mentality on that part), but that i worry there will be so much institutional inertia that much like Germany in WW2 too much of the people who mater won't be able to make the leap to realising that trying to do things to the utmost isn't necessarily as important as getting the most cost effective solution. Which is not a very nice thing to talk about when most cost effective probably means more losses for the troops using them i admit.

At the same time i agree there are good reasons to use such a mentality right now for the most part.

Gnoman
2018-02-04, 08:43 PM
Based on your choice of analogy, you're operating under some flawed assumptions. WWII Germany didn't lose because they were "trying to do things to the utmost" instead of looking for the most cost-effective solution. WWII Germany lost because they were massively outweighed, and because their equipment was on par at best with what the Allies were using.

Mr Beer
2018-02-05, 12:44 AM
Based on your choice of analogy, you're operating under some flawed assumptions. WWII Germany didn't lose because they were "trying to do things to the utmost" instead of looking for the most cost-effective solution. WWII Germany lost because they were massively outweighed, and because their equipment was on par at best with what the Allies were using.

Agree, they lost because Hitler bit off more than he could chew with Barbarossa and then resolved every strategic crisis that followed by doubling down. Two fronts giving us trouble? No worries, lets declare war on the US!

Haighus
2018-02-05, 07:09 AM
Based on your choice of analogy, you're operating under some flawed assumptions. WWII Germany didn't lose because they were "trying to do things to the utmost" instead of looking for the most cost-effective solution. WWII Germany lost because they were massively outweighed, and because their equipment was on par at best with what the Allies were using.
There are several examples with the German military having equipment that was more than on par with the Allies' gear, it just isn't typically in the areas people think of (tanks). For example, the German small arms had some examples much more modern and practical than the Allies- in particular the sturmgewehr and Mg42 being concepts that changed the way modern infantry were armed. If the stg44 was not delayed in deployment, it probably could've had a bigger impact on infantry engagements. Most of the other small arms is good but not better than the Allies' equivalents, and often more expensive (such as the Mp40 being fairly equivalent to a Thompson in being good but expensive), and the K98 being a very good bolt action which is probably a little better than a Mosin-Nagant, but a little worse as a battlefield weapon than a Lee-Enfield or M1 Garand (although the K98 was good for marksman duties).

Their man-portable anti-tank weapons were also very good, and the panzerfaust was an unusual German example of a cost-effective, cheap but individually not so great weapon.

This doesn't change that Germany was outproduced industrially, and made some huge strategic blunders that cost them a lot, but they definitely could've optimised their industrial output to produce the most cost-effective military gear, and wasted less on fancy tanks.

Carl
2018-02-05, 08:14 AM
Based on your choice of analogy, you're operating under some flawed assumptions. WWII Germany didn't lose because they were "trying to do things to the utmost" instead of looking for the most cost-effective solution. WWII Germany lost because they were massively outweighed, and because their equipment was on par at best with what the Allies were using.

I think you need to check some of my prior posts, i'm well aware of the strategic reasons Germany lost WW2. But it's also widely agreed they lost it much faster than they otherwise would because their production pipeline was a mess because of heir focus on doing everything "just so", and on allways pushing the tech curve. (It also caught them out in other ways with overengineered items but thats a whole other ballgame to what i'm discussing).

Haighus
2018-02-05, 09:09 AM
The 19th Century adventurer, Sir Richard Burton, stated in his 1884 work The Book of the Sword that the yataghan was the best-designed sword out there. Was he accurate, or was that merely his opinion?

Coincidentally I have just come across this video on Burton, which gives me much less faith in his opinion on swords. I think it is merely his opinion.

https://www.youtube.com/watch?v=CRZJxGi8Z44

Vinyadan
2018-02-05, 09:32 AM
This doesn't change that Germany was outproduced industrially, and made some huge strategic blunders that cost them a lot, but they definitely could've optimised their industrial output to produce the most cost-effective military gear, and wasted less on fancy tanks.

That's a big point, Germany entered war economy very late. In 1942, weapon factories were working on one shift, and very few women were employed. There also were power struggles within Hitler's court, which meant that Speer, who more or less doubled weapon production after his appointment, had to be very watchful not to be ousted and his special powers ransacked by rivals.

In general, I think that there simply aren't the conditions for a WW2 kind of war for Western countries. W-European powers are peaceful concerning each other, and are way too large for any other territorially close enemy that isn't Russia, and a war with Russia isn't going to revolve around conventional weapons anyway. To be on par with the USA, it would take Russia, China and India to somehow become allied and go to war together. Beside the obvious question (why should India go to war against the USA?), there also is a less obvious problem with this: Russia and China are rival powers, too, and this makes for bad allies. They really don't have much in common, beside wanting to be an alternative to US dominance, but they also want to be an alternative to each other. And then we again have the atomic deal, which would make such a war look very different from previous wars.

Probably the most likely war of this kind would be in Korea, if there were a US invasion, with China joining the other side, like they did in the Fifties. However, even China (and Russia) are going for smaller, more professional, more technological armed forces.

VoxRationis
2018-02-08, 12:46 PM
In the games I play, I typically see scale armor presented as out-and-out inferior to mail. Is this so? Does mail provide better protection than scale, either outright or on a per weight basis? Are there any sorts of things against which scale is a superior defense?

PersonMan
2018-02-08, 02:10 PM
Agree, they lost because Hitler bit off more than he could chew with Barbarossa and then resolved every strategic crisis that followed by doubling down. Two fronts giving us trouble? No worries, lets declare war on the US!

Barbarossa was a mess itself, as a campaign. Interestingly enough, the Wehrmacht put more thought into logistical planning than they get credit for. It's just that the planning process went something like this:

1. We won't be able to send winter supplies to our troops while also sending combat supplies.
2. We will only need combat supplies in large quantities during the actual campaign.
3. Solution: Defeat Soviet Union before winter supplies are needed.

Mendicant
2018-02-08, 03:58 PM
Are there any well attested instances of classical-medieval heavy weaponry being incorporated into fortresses? I'm thinking of scorpions/catapults set up on towers, that sort of thing. The architecture doesn't seem designed around them the way later fortifications were built not just to resist cannons but to incorporate them.

If not, why not? A scorpion's range and accuracy seems like it'd be especially helpful for harrassing enemy siege weapon crews.

Haighus
2018-02-08, 04:15 PM
Chateau Gaillard had a mangonel on the most exposed tower. It was positioned to counter enemy siege weapons.

Vinyadan
2018-02-08, 05:18 PM
Are there any well attested instances of classical-medieval heavy weaponry being incorporated into fortresses? I'm thinking of scorpions/catapults set up on towers, that sort of thing. The architecture doesn't seem designed around them the way later fortifications were built not just to resist cannons but to incorporate them.

If not, why not? A scorpion's range and accuracy seems like it'd be especially helpful for harrassing enemy siege weapon crews.

There were the ballistae on the towers around Londinium (built after 350 AD). https://books.google.it/books?id=MCQzDAAAQBAJ&pg=PT102&lpg=PT102&dq=ballista+londinium&source=bl&ots=KHu7y3iMuq&sig=M6tdMIJ89_sSIb5gX9DUgJy9Dcs&hl=it&sa=X&ved=0ahUKEwjA9eLFq5fZAhXR2qQKHas-Da8Q6AEITzAI#v=onepage&q=ballista%20londinium&f=false

wolflance
2018-02-08, 08:42 PM
In the games I play, I typically see scale armor presented as out-and-out inferior to mail. Is this so? Does mail provide better protection than scale, either outright or on a per weight basis? Are there any sorts of things against which scale is a superior defense?
Yes, scale armor is generally inferior to mail - not in term of protective quality (it can resist sword, axe, spear and arrow just fine), but in term of durability. A powerful axe chop against scale armor will likely knock some scales off even if it fails to penetrate and harm the wearer, so repeated blows will eventually damage the armor enough to render it useless. Mail isn't immune to this either, but it generally last way longer.

(Some European scale armors are riveted to the cloth backing. I don't know how durable those are but likely much better than the laced version)

And scale also has the vulnerability to upward thrust...

Scale's advantages over mail are that it's less picky about what material you use, requires less expertise to manufacture, and probably cheaper (although in this case lamellar or brigandine are much better choices compared to scale).

It's also better against blunt trauma than mail, presumably.

Mendicant
2018-02-08, 11:11 PM
It isn't really that much better against blunt trauma. In order to keep the weight reasonable the thickness of the scales needs to be pretty thin, so in the end you get something that isn't terribly good at absorbing those sorts of shocks.

Tobtor
2018-02-09, 06:31 AM
In the games I play, I typically see scale armor presented as out-and-out inferior to mail. Is this so? Does mail provide better protection than scale, either outright or on a per weight basis? Are there any sorts of things against which scale is a superior defense?

I agree with wolflance: mail is (slightly) better. Especially in prolonged melee battles. In Europe mail was (genreally) prefered, in spite of scale was known. Scale takes less time to produce, so it will be cheaper. Scale is extensively used among horse-cultures, and offer the same protection as mail against arrows, so if arrows are you main concern it is a good choice.

wolflance
2018-02-09, 10:45 AM
I agree with wolflance: mail is (slightly) better. Especially in prolonged melee battles. In Europe mail was (genreally) prefered, in spite of scale was known. Scale takes less time to produce, so it will be cheaper. Scale is extensively used among horse-cultures, and offer the same protection as mail against arrows, so if arrows are you main concern it is a good choice.
Actually depending on where you look. Even horse cultures rarely used scale armor.

Lamellar armor is all around superior to scale, having all of its advantages and much less downsides. It is also superior against blunt and projectile weaponry against both scale and mail.

Vinyadan
2018-02-09, 11:27 AM
Scales is superior when you want to draw a gritty fight with swords as wide as your hand, scales exploding out as the coat is torn open by the unavoidable swing!

Haighus
2018-02-10, 10:03 PM
Can anyone give me any info or sources about firearms in medieval India? I am mainly looking at pre-Mughal India.

From what I understand, the earliest gunpowder weapons were introduced around the time of the Mongol invasions of north India in the 13th century, like most of Eurasia, but then there doesn't seem to be much for the next few centuries until the Mughals. Mainly just bombs and other incendiaries, with firearms being uncommon. This is based mainly on Wikipedia to be honest, but I don't know where else to look for info on this in English. Also Wikipedia had much more info for other world regions. I have had someone tell me firearms were considered unpopular and poor military weapons in India.

This seems pretty odd to me, when firearms were generally taken up with great enthusiasm by just about every other region of the world, despite their drawbacks. The culturally close South East Asia used them, as did Central Asia, China and the Middle East. If all these peripheral regions made extensive use of firearms, why would highly developed India not?

Galloglaich
2018-02-11, 07:25 AM
Can anyone give me any info or sources about firearms in medieval India? I am mainly looking at pre-Mughal India.

From what I understand, the earliest gunpowder weapons were introduced around the time of the Mongol invasions of north India in the 13th century, like most of Eurasia, but then there doesn't seem to be much for the next few centuries until the Mughals. Mainly just bombs and other incendiaries, with firearms being uncommon. This is based mainly on Wikipedia to be honest, but I don't know where else to look for info on this in English. Also Wikipedia had much more info for other world regions. I have had someone tell me firearms were considered unpopular and poor military weapons in India.

This seems pretty odd to me, when firearms were generally taken up with great enthusiasm by just about every other region of the world, despite their drawbacks. The culturally close South East Asia used them, as did Central Asia, China and the Middle East. If all these peripheral regions made extensive use of firearms, why would highly developed India not?

It's an interesting question and I don't have a direct answer (looking forward to learning from someone who does). As a wild guess though if it's true that there was less ready adoption of firearms in India it might be due to moisture. Humidity, monsoon rains, jungle conditions. Early gunpowder (etc.) was notoriously vulnerable to moisture, especially before the 14th Century when they began to learn to process potassium nitrate better (to exclude the calcium nitrate).

I would be really surprised if they didn't use some gunpowder weapons though at least in forts.

G

Galloglaich
2018-02-11, 07:26 AM
I agree with wolflance: mail is (slightly) better. Especially in prolonged melee battles. In Europe mail was (genreally) prefered, in spite of scale was known. Scale takes less time to produce, so it will be cheaper. Scale is extensively used among horse-cultures, and offer the same protection as mail against arrows, so if arrows are you main concern it is a good choice.

I suspect scale, and lamellar might both be a bit better against missiles, while mail is better against sustained close range attacks (cuts and thrusts) and more durable as people have mentioned.

Both scale and lamellar seem to have been more widely used in the Middle East and in Asia whereas mail, especially the stronger types of riveted mail, seems to have originated in Europe and was mostly widely used there (although also in the Middle East)

G

Haighus
2018-02-11, 07:57 AM
It's an interesting question and I don't have a direct answer (looking forward to learning from someone who does). As a wild guess though if it's true that there was less ready adoption of firearms in India it might be due to moisture. Humidity, monsoon rains, jungle conditions. Early gunpowder (etc.) was notoriously vulnerable to moisture, especially before the 14th Century when they began to learn to process potassium nitrate better (to exclude the calcium nitrate).

I would be really surprised if they didn't use some gunpowder weapons though at least in forts.

G
Humidity could well be a factor, which perhaps makes it all the more confusing that the armies of South East Asia used them in the rainforests there. Forts and sieges would be the logical place to find gunpowder weapons I agree.

I think you are right, that there were at least some gunpowder weapons. I likewise would be interested to hear any other ideas for why they may be uncommon.

Warfare in the Indian-influenced regions of South East Asia and India itself is something I know little about, but would like to learn more.

Kiero
2018-02-11, 08:56 AM
I'd also suspect entrenched vested interests in fighting in particular ways and good, old-fashioned cultural inertia as well. You had guilds, warrior societies and tribal groupings who were all invested in fighting with bow, spear and sword (along with a plethora of unarmed styles), which create exclusivity given the time taken to train with them.

By contrast any peasant can learn to stand in a firing line in a matter of weeks. When you have large, complex societies, as you had in India, cultural factors can overwhelm everything else.

Haighus
2018-02-11, 10:53 AM
I'd also suspect entrenched vested interests in fighting in particular ways and good, old-fashioned cultural inertia as well. You had guilds, warrior societies and tribal groupings who were all invested in fighting with bow, spear and sword (along with a plethora of unarmed styles), which create exclusivity given the time taken to train with them.
This seems reasonable.

By contrast any peasant can learn to stand in a firing line in a matter of weeks. When you have large, complex societies, as you had in India, cultural factors can overwhelm everything else.
Whilst this is true for later periods, I'm mainly interested in pre-Mughal medieval India. I thought at this point gunpowder weapons are mainly expert equipment, not weapons given to levies? As I understand it, the shift happened in the 16th century with improving powder and matchlocks, and formalised drills to reduce the risk of operator error.

rrgg
2018-02-11, 12:28 PM
Can anyone give me any info or sources about firearms in medieval India? I am mainly looking at pre-Mughal India.

From what I understand, the earliest gunpowder weapons were introduced around the time of the Mongol invasions of north India in the 13th century, like most of Eurasia, but then there doesn't seem to be much for the next few centuries until the Mughals. Mainly just bombs and other incendiaries, with firearms being uncommon. This is based mainly on Wikipedia to be honest, but I don't know where else to look for info on this in English. Also Wikipedia had much more info for other world regions. I have had someone tell me firearms were considered unpopular and poor military weapons in India.

This seems pretty odd to me, when firearms were generally taken up with great enthusiasm by just about every other region of the world, despite their drawbacks. The culturally close South East Asia used them, as did Central Asia, China and the Middle East. If all these peripheral regions made extensive use of firearms, why would highly developed India not?

"Gunpowder and Firearms: Warfare in Medieval India" by Iqtidar Alam Khan is an excellent book on the subject if you can find it in a library, however it is out of print now. If you have JSTOR access there are a number of articles by the same author if you do a search for his name.

It seems that india was starting to use early rockets, bombs, and projectile throwers based on gunpowder, especially during sieges, in the mid to late 14th century. But I don't think they were used nearly as extensively as they were in the ming empire, who found small cannons and handguns extremely effective against nomadic cavalry armies on the northern frontier.

During the 15th century though the most extensive development, including the invention of matchlocks and more modern artillery, seems to have been occurring in eastern europe with european and ottoman gunpowder technology advancing side by side. During the second half of the 15th century there is evidence that early matchlocks and cannons had made their way to India from the Ottoman empire, but they weren't as effective as the ones that the Mughals or Portuguese eventually brought with them.

Mendicant
2018-02-12, 12:03 AM
One thing to remember is that states had to have a certain scale and centralization before they started fielding significant gunpowder weaponry. Historians classify certain states, especially the Ottomans, Safavids and Mughals as the "gunpowder empires"--states that developed strong central imperial governments in conjunction with the advent and spread of firearms.

Basically, it's a self-reinforcing cycle where political centralization leads to better control of, and access to, the supply and industrial chains necessary to maintain gunpowder armies, and then those armies concentrate power even more heavily in the hands of the central government. I'm far from an expert in pre-mughal India, but if it had significant political fracturing and small/weak monarchies, they wouldn't have had the concentration of resources that say, the Ming or Ottomans did.

snowblizz
2018-02-12, 06:42 AM
I'm far from an expert in pre-mughal India, but if it had significant political fracturing and small/weak monarchies, they wouldn't have had the concentration of resources that say, the Ming or Ottomans did.
Which actually exactly describes the situation in India until British rule really starts cracking down on it in the later 1800s.

Off the cuff I can only recall the Guptas (300-600 CE abouts) and then Moghuls as empires that ruled over large part of the Indian subcontinent. I see now from wiki that the Delhi sultanate (1200s-1500s) and Mughals had a fair bit of territory. Interestingly THe Mughals repalced the Delhi sultante in part due to their firearms technology. There's the Maratha confederacy too later on competing with late Mughals. Generalizing the northern Ganges plains were easily controlled and invaded from outside the subcontinent and were where the Guptas and Mughals rules. Then you have the Deccan where the Maratha were focused which is sort of it's own thing. They'd usually be rival power centres. Between and around the Ganges plain and the Deccan you'd have loads of smaller kingdoms of various permanency all depending on the strength of the main powers.

Many of the larger states would have been working through ancient lines of power though and might not have more than nominal control of some areas. Some of the areas with petty kingdoms would have been geographically challenging to control and could easily resist larger foes for long periods. And of course tended to rebel eroding existing powers.

VoxRationis
2018-02-12, 01:14 PM
What sort of technological prerequisites are there to inventing gunpowder? Is there any reason other than chance that it was 9th-century China that discovered it, and not, say, BCE Rome, Egypt, or Babylon? The ingredients don't seem to be an issue, as both saltpeter and sulfur are naturally occurring. Is there some aspect of mixing and combining them that represents a hurdle only more advanced technology can overcome?

Max_Killjoy
2018-02-12, 01:42 PM
What sort of technological prerequisites are there to inventing gunpowder? Is there any reason other than chance that it was 9th-century China that discovered it, and not, say, BCE Rome, Egypt, or Babylon? The ingredients don't seem to be an issue, as both saltpeter and sulfur are naturally occurring. Is there some aspect of mixing and combining them that represents a hurdle only more advanced technology can overcome?

I asked a somewhat similar question I think towards the end of the prior incarnation of the thread.

http://www.giantitp.com/forums/showthread.php?532903-Got-a-Real-World-Weapon-Armor-or-Tactics-Question-Mk-XXIV/page43&p=22725461#post22725461

See forward from there for the answers to my version of that question.

Mike_G
2018-02-12, 03:08 PM
Does anyone here know why leaf-shaped swords stopped appearing? They seem very common in bronze age and early iron age weapons, but then they more of less vanish.

Did their advantages work better with bronze and less so with steel? Do they stop being useful once you get a longer blade?

I'm sure there's a reason, I just don't see what it was. The idea of more mass at the center of percussion and a nice acute point seems pretty solid. What was the advantage of the straighter migration-era pattern?

Galloglaich
2018-02-12, 03:38 PM
One thing to remember is that states had to have a certain scale and centralization before they started fielding significant gunpowder weaponry. Historians classify certain states, especially the Ottomans, Safavids and Mughals as the "gunpowder empires"--states that developed strong central imperial governments in conjunction with the advent and spread of firearms.

Basically, it's a self-reinforcing cycle where political centralization leads to better control of, and access to, the supply and industrial chains necessary to maintain gunpowder armies, and then those armies concentrate power even more heavily in the hands of the central government. I'm far from an expert in pre-mughal India, but if it had significant political fracturing and small/weak monarchies, they wouldn't have had the concentration of resources that say, the Ming or Ottomans did.

That is interesting - and while it may be true for the Ottomans, it's almost the opposite in Europe where the city-States and Free Cities (very small polities of anywhere from 10,000 to maybe 100,000 people) were way ahead of the larger Kingdoms on the use of gunpowder weapons. France eventually got up to speed probably much in the same way as the Ottomans did - due to contending for a long time with Flemish city-states (as part of the Burgundian Duchy).

But cities like Ghent created weapons in the very early 15th Century (like this beast (https://en.wikipedia.org/wiki/Dulle_Griet)) that France couldn't produce until 100 years later.

The City-State of Venice probably had the largest, most efficient, most technologically advanced and most productive arms industry in all of Latinized Europe, the famous "Arsenal", known chiefly for it's ability to produce a warship in a single day, but they also routinely produced large quantities of high quality firearms - by the hundreds when most polities of their day were producing guns in the scores or dozens.

https://en.wikipedia.org/wiki/Venetian_Arsenal#History

Nuremberg was another leading producer, making large quantities of the most advanced firearms in the 15th and 16th Centuries and leading the way with technology like wheel locks, rifling, and breach loading firearms.



I guess when it comes to arms production there are two ways to go about it - the large Centralized State or the small but free and self-administered city.


Regarding India, I do see some formidable cannon there by the 16th Century, such as this beast (https://en.wikipedia.org/wiki/Bir_Hambir#Dal_Madal_Kaman_.28Cannon.29) the Dal Madal Kaman.

I suspect maybe the reason that the South Asians didn't adapt gunpowder, aside from humidity and monsoon rains, might be the destabilizing devastation wrought by the Mongols and subsequent violent establishment of the Mughal State. The Chinese similarly had the Yuang dynasty but they seem to have assimilated to Chinese culture a bit quicker and so far as I know, were not of a different religion.

The Mughals seem to have retained a lot of their Mongol ways in terms of how they fought, and the Mongols never did adapt firearms beyond a kind of niche use. They preferred the horse-archer, which worked on the Steppe for probably at least 2,000 years so why not.

G

Sapphire Guard
2018-02-12, 04:48 PM
What would be the lowest plausible rank in the US army to be left in charge of retaking/quarantining a city from the zombie apocalypse? Assuming military structures are intact and it's a fairly low priority.

DrewID
2018-02-12, 04:55 PM
Does anyone here know why leaf-shaped swords stopped appearing? They seem very common in bronze age and early iron age weapons, but then they more of less vanish.

Did their advantages work better with bronze and less so with steel? Do they stop being useful once you get a longer blade?

I'm sure there's a reason, I just don't see what it was. The idea of more mass at the center of percussion and a nice acute point seems pretty solid. What was the advantage of the straighter migration-era pattern?

At least in the long term (and I'm moving to late medieval here), I know the goal for improved sword handling capability was to move the center of mass as close to the hilt as possible, so from that point of view at least, a leaf-shaped blade is counter-intuitive. While a sword with more mass towards the point might deliver a heavier chop, there are better weapons for chopping than a sword. And more mass towards the point would be of little to no benefit in delivering a thrust.

[hit send too soon] Also, for bronze and early steel blades, the weak spot for bending/snapping is the narrowest point. Which I suspect led to a cost/benefit move towards uniform width blades, as they were only as strong as that weak spot, but the leaf-shaped blade used more metal that one consistently of that narrowest width,

DrewID

Storm Bringer
2018-02-12, 05:00 PM
What would be the lowest plausible rank in the US army to be left in charge of retaking/quarantining a city from the zombie apocalypse? Assuming military structures are intact and it's a fairly low priority.

depends on the size of the force, but i'd go for brigadier general, or maybe full colonel. any lower than that and you have a force too small to do much more than secure a few blocks.

realistically, youd need thousands of troops to properly sweep a city, and it'd be a slow, slow process as youd have to clear every room of every building. s lot of your manpower would be just trying to maintain a cordon that can keep the dead out of the "Cleared" sectors of a city.

Mr Beer
2018-02-12, 05:12 PM
What would be the lowest plausible rank in the US army to be left in charge of retaking/quarantining a city from the zombie apocalypse? Assuming military structures are intact and it's a fairly low priority.

That's going to depend on how many troops are required to do the job, which in turn is going to depend a lot on the nature of the zombies. Slow, unintelligent zombies that can be killed by headshots are not particularly threatening to disciplined troops with good rifle training, unless in vast numbers.

In terms of retaking a city, can you do it slowly by house-clearing or do you have to basically take on a swarming horde of zombies in one go?

If the former, and it's low priority, you can assign a platoon commanded by a junior officer and they can take months to get it done.

If it's a swarm that you have to destroy in a single battle, well you need more guys. Say you want a 1:10 ratio or better, a small city with 20K original inhabitants that now consists of 10K zombies, you need 1K soldiers so that's a couple of battalions. My guess is that a colonel would be put in charge of the job.

Numbers could be wildly off though. If it's possible to channel zombies into a prepared killing ground, with the use of noise emitters to lure them in and chain link fences to contain them and a large shallow pit that they end up walking into, you could do the job with far fewer men.

As well as specialised tactics, you will end up with specialised gear. Slow dumb zombies are going to be useless against men with body-armour and skull-breaking weapons. If you're wiping out zombies in relatively small groups as opposed to giant swarms, you might have squads of guys kitted out in what looks like a Mad-Max/HEMA combination, who simply close and kill the enemy hand to hand.

Sapphire Guard
2018-02-12, 05:49 PM
Thank you, folks. Brigadier General sounds about right, the idea is that the commander can quarantine the city easily enough because zombies are an extremely poor threat against any kind of half baked fortification
unless something goes terribly wrong, but doesn't have enough crew to clear it so starts hiring contractors.

Haighus
2018-02-12, 07:13 PM
Which actually exactly describes the situation in India until British rule really starts cracking down on it in the later 1800s.

Off the cuff I can only recall the Guptas (300-600 CE abouts) and then Moghuls as empires that ruled over large part of the Indian subcontinent. I see now from wiki that the Delhi sultanate (1200s-1500s) and Mughals had a fair bit of territory. Interestingly THe Mughals repalced the Delhi sultante in part due to their firearms technology. There's the Maratha confederacy too later on competing with late Mughals. Generalizing the northern Ganges plains were easily controlled and invaded from outside the subcontinent and were where the Guptas and Mughals rules. Then you have the Deccan where the Maratha were focused which is sort of it's own thing. They'd usually be rival power centres. Between and around the Ganges plain and the Deccan you'd have loads of smaller kingdoms of various permanency all depending on the strength of the main powers.

Many of the larger states would have been working through ancient lines of power though and might not have more than nominal control of some areas. Some of the areas with petty kingdoms would have been geographically challenging to control and could easily resist larger foes for long periods. And of course tended to rebel eroding existing powers.
That is interesting - and while it may be true for the Ottomans, it's almost the opposite in Europe where the city-States and Free Cities (very small polities of anywhere from 10,000 to maybe 100,000 people) were way ahead of the larger Kingdoms on the use of gunpowder weapons. France eventually got up to speed probably much in the same way as the Ottomans did - due to contending for a long time with Flemish city-states (as part of the Burgundian Duchy).

But cities like Ghent created weapons in the very early 15th Century (like this beast (https://en.wikipedia.org/wiki/Dulle_Griet)) that France couldn't produce until 100 years later.

The City-State of Venice probably had the largest, most efficient, most technologically advanced and most productive arms industry in all of Latinized Europe, the famous "Arsenal", known chiefly for it's ability to produce a warship in a single day, but they also routinely produced large quantities of high quality firearms - by the hundreds when most polities of their day were producing guns in the scores or dozens.

https://en.wikipedia.org/wiki/Venetian_Arsenal#History

Nuremberg was another leading producer, making large quantities of the most advanced firearms in the 15th and 16th Centuries and leading the way with technology like wheel locks, rifling, and breach loading firearms.



I guess when it comes to arms production there are two ways to go about it - the large Centralized State or the small but free and self-administered city.


Regarding India, I do see some formidable cannon there by the 16th Century, such as this beast (https://en.wikipedia.org/wiki/Bir_Hambir#Dal_Madal_Kaman_.28Cannon.29) the Dal Madal Kaman.

I suspect maybe the reason that the South Asians didn't adapt gunpowder, aside from humidity and monsoon rains, might be the destabilizing devastation wrought by the Mongols and subsequent violent establishment of the Mughal State. The Chinese similarly had the Yuang dynasty but they seem to have assimilated to Chinese culture a bit quicker and so far as I know, were not of a different religion.

The Mughals seem to have retained a lot of their Mongol ways in terms of how they fought, and the Mongols never did adapt firearms beyond a kind of niche use. They preferred the horse-archer, which worked on the Steppe for probably at least 2,000 years so why not.

G

My thoughts were that Europe was really pretty similar to India in terms of a mix of large and small polities, with overlapping and changing allegiances, yet most states started to use gunpowder more and more in Europe. I suppose a combination of all of these factors is at play- whilst none of them alone seems to be a reason to avoid gunpowder, if you add fractured states, a humid environment, entrenched cultural ideals and foreign anti-gun influence, I can see that all working together to make firearms rare.

Whilst it is correct to say Venice was a city state, I think it is a bit unfair to use them as an example of a small state vs a big centralised one- they also operated a reasonably large empire to finance their production, and basically inherited a lot of the Byzantine empire. Whilst it is mighty impressive that such a small state was able to build such a powerful empire, at the point they were producing loads of gear, they had said empire.

I wouldn't be surprised if the Mongols didn't use firearms heavily because early firearms were not easy to use from horseback until the wheellock comes along, so gunners would not be easy to factor into their highly mobile style of warfare. At best, they would likely be used as mounted infantry.


As well as specialised tactics, you will end up with specialised gear. Slow dumb zombies are going to be useless against men with body-armour and skull-breaking weapons. If you're wiping out zombies in relatively small groups as opposed to giant swarms, you might have squads of guys kitted out in what looks like a Mad-Max/HEMA combination, who simply close and kill the enemy hand to hand.
I've always thought that in a zombie apocalypse, my first action is to the raid the nearest medieval museum for some armour (I am lucky enough to live in Europe, where such museums are not uncommon). Full plate or full mail with a face coif would leave you almost invulnerable from being infected, short of zombies smart enough to forceably remove segments of armour. Otherwise, it is massively repeated blunt trauma or suffocation to worry about, and bodily needs requiring some degree of exposure, like toileting, washing or eating.

snowblizz
2018-02-13, 04:09 AM
My thoughts were that Europe was really pretty similar to India in terms of a mix of large and small polities, with overlapping and changing allegiances, yet most states started to use gunpowder more and more in Europe. I suppose a combination of all of these factors is at play- whilst none of them alone seems to be a reason to avoid gunpowder, if you add fractured states, a humid environment, entrenched cultural ideals and foreign anti-gun influence, I can see that all working together to make firearms rare.
I'm always one for looking at a combination of factors for broad trends. I would also point out that the Indian subcontinent didn't have platearmoured knights to crack open with firearms either. I'm not sure about "foreign anti-gun influence" though, what do you mean by that? What we don't quite see, from what I know, in some of the broder culture on the continent is the kind of social dynamicism we see in Europe where friction between various ideas, social fabrics and cultures spur a lot of innovation.
At least we can't blame wheat, germs and steel.


I wouldn't be surprised if the Mongols didn't use firearms heavily because early firearms were not easy to use from horseback until the wheellock comes along, so gunners would not be easy to factor into their highly mobile style of warfare. At best, they would likely be used as mounted infantry.Firearms barely existed during the Mongol heyday so it's not exactly like they had opportunity to really deploy the technology. Though their successors did use them. The "problem" of all steppepeoples has always been one of production. The gunpowder using Mongol successors e.g. were much more settled peoples.



I've always thought that in a zombie apocalypse, my first action is to the raid the nearest medieval museum for some armour (I am lucky enough to live in Europe, where such museums are not uncommon). Full plate or full mail with a face coif would leave you almost invulnerable from being infected, short of zombies smart enough to forceably remove segments of armour. Otherwise, it is massively repeated blunt trauma or suffocation to worry about, and bodily needs requiring some degree of exposure, like toileting, washing or eating.
I think you'll find the odds of making use of the armour isn't as good as one might hope though. The best armour was rather formfitting and what museums got on display tend to be the best examples of fitted armour. As an example there's no way Henry VIII's armour would be of any use to me. You might also be lacking the padding to go with it. And will the straps hold up? You'd be unused to moving about in the armour and it won't be as flexible to you as it was it's original owner.
Theoretically you are right, in that if you are lucky you could find a suit of armour that fits. But then I have to ask, do you know how to put it on?:smallbiggrin:
As always, depending on what type of zombies. But I think being able to move normally and staying away is the best recourse usually.

Haighus
2018-02-13, 05:11 AM
I'm always one for looking at a combination of factors for broad trends. I would also point out that the Indian subcontinent didn't have platearmoured knights to crack open with firearms either. I'm not sure about "foreign anti-gun influence" though, what do you mean by that? What we don't quite see, from what I know, in some of the broder culture on the continent is the kind of social dynamicism we see in Europe where friction between various ideas, social fabrics and cultures spur a lot of innovation.
At least we can't blame wheat, germs and steel.
I was referring to Galloglaich's thoughts above regarding Mongol influence.

Lack of plate is true, but did India use the combined mail and plate style of armour we see in the Middle East? I am referring to the suits that look like a combo of mail and a coat of plates, with the plates connected together via mail. I wouldn't be surprised if the north of India did, but the south didn't. Armour seems to be unpopular in tropical regions, due to the climate and issues in overheating I assume.


Firearms barely existed during the Mongol heyday so it's not exactly like they had opportunity to really deploy the technology. Though their successors did use them. The "problem" of all steppepeoples has always been one of production. The gunpowder using Mongol successors e.g. were much more settled peoples.

Good point.


I think you'll find the odds of making use of the armour isn't as good as one might hope though. The best armour was rather formfitting and what museums got on display tend to be the best examples of fitted armour. As an example there's no way Henry VIII's armour would be of any use to me. You might also be lacking the padding to go with it. And will the straps hold up? You'd be unused to moving about in the armour and it won't be as flexible to you as it was it's original owner.
Theoretically you are right, in that if you are lucky you could find a suit of armour that fits. But then I have to ask, do you know how to put it on?:smallbiggrin:
As always, depending on what type of zombies. But I think being able to move normally and staying away is the best recourse usually.
I was actually thinking of looking for mail for pretty much all the reasons you point out! Much more forgiving in sizes, easier to put on and off, less reliance on old straps and should still be plenty resistant to bites. The padding for mail could also simply be thick clothing. The main weakness would be my face and head- mail face coifs were not common here. I would have to try to find a visored helmet to cover that gap.

gkathellar
2018-02-13, 07:54 AM
If you're giving serious consideration to zombies, I cannot endorse the book World War Z by Max Brooks strongly enough (the film has very little to do with its source material). Aside from being an entertaining read, it takes a serious look at the challenges zombies would present to modern entities, and the adaptations required to fight them effectively. It was, IIRC, at one point on the recommended reading list of the USAF Admiral of the Navy.

A few points of note:
Modern weaponry stressing rate of fire, incendiaries, shrapnel weapons and more are of little or no use against opponents that can only be stopped by massive head trauma. A steady, accurate semi-automatic rifle that never jams becomes the new standard. This is accompanied by the use of firing lines and rangefinders on prepared battlefields.
Troops becoming too psychologically or physically exhausted to continue needs to be treated as a given. Zombie engagements are long, disgusting, and may involve mercy killings.
CQC demands heavy, sturdy weapons that can cleave skulls without being worn down. Decapitation is not an option if a zombie's head can still bite. Reach is also a big advantage. Basically you want an overbuilt trench axe/halberd, supplemented by a variety of heavy knives and hatchets for when things get really ugly. Post CQC-hygiene is very important!
A damaged zombie is a hazard. Sever the legs and they will crawl. Sever the head and they will bite. Constant floor sweeps are a tactical necessity. Dogs are good for this, among other things.
Do zombies drown? No, and that means bodies of water become very, very bad places. Clearing out the ancient sewer and tunnel systems underneath some European cities is perhaps the single worst-case scenario for fighting zombies.

Haighus
2018-02-13, 09:54 AM
Why would the close combat weapons need to be overbuilt? A typical halberd is more than capable of cleaving unarmoured skulls- any heavier and it would be less effective as a weapon. Actually, in the context of unarmed, unarmoured zombies, they could likely make a halberd lighter and it would perform better with quicker follow-up attacks.

A greatsword could be effective as an area denial weapon in close combat to protect a squad- enough power to easily hack through skulls, and capable of holding back multiple enemies. Would require more training than halberds probably. I feel that poleaxes and hammers would also be good for crushing skulls without getting stuck.

gkathellar
2018-02-13, 10:56 AM
Why would the close combat weapons need to be overbuilt? A typical halberd is more than capable of cleaving unarmoured skulls- any heavier and it would be less effective as a weapon. Actually, in the context of unarmed, unarmoured zombies, they could likely make a halberd lighter and it would perform better with quicker follow-up attacks.

A greatsword could be effective as an area denial weapon in close combat to protect a squad- enough power to easily hack through skulls, and capable of holding back multiple enemies. Would require more training than halberds probably. I feel that poleaxes and hammers would also be good for crushing skulls without getting stuck.

Mainly because durability and ease of use become extremely valuable when facing down hordes of mindless, unskilled enemies who are not going to defend themselves meaningfully. It also helps if it can serve as a makeshift shovel.

Look at it this way: a sword's greatest assets are defensive versatility and the ability to deliver terrible wounds from a variety of angles due to structural advantages and a long cutting edge. None of that matters against zombies - you just need to put a hole in the thing's skull, and you may need to do that thousands of times without your weapon breaking. Think of it less as fighting and more as doing a really dangerous, unglamorous job, which demands a reliable, unglamorous tool. For this purpose, a broad-bladed spear, pole axe, or similar weapon is ideal because it keeps you far away from the zombie, punctures the skull with minimal fuss, and is easier to replace than a sword if it does happen to break.

Knaight
2018-02-13, 10:59 AM
Mainly because durability and ease of use become extremely valuable when facing down hordes of mindless, unskilled enemies who are not going to defend themselves meaningfully. It also helps if it can serve as a makeshift shovel.

With that said anything designed to deal with armor is already overbuilt for flesh.

Lapak
2018-02-13, 11:18 AM
Mainly because durability and ease of use become extremely valuable when facing down hordes of mindless, unskilled enemies who are not going to defend themselves meaningfully. It also helps if it can serve as a makeshift shovel.
It sounds like your ideal melee configuration is a rank of boar-spear-wielders who keep the horde at arm’s length and a rank of pole-hammer folks to bop the impaled zombies on the head and polish them off. I imagine you could go quite a while without significant casualties that way.

Haighus
2018-02-13, 03:37 PM
With that said anything designed to deal with armor is already overbuilt for flesh.

This is my thoughts- historical melee weapons are designed to cope with repeated contact with metal and wood, both of which are generally tougher than rotting flesh and bone. A weapon that breaks in the first couple of encounters in a battle is pretty crap unless it is secondary (javelins), there is an easy way to rearm, or the weapon gives some massive advantages otherwise (such as lances).

A heavier weapon is also going to be more tiring when repeatedly hacking at oncoming zombies, whilst basically uncessary for dealing with destroying heads. Also, I find stabbing skulls is likely to be less effective than chopping into or crushing skulls, but then the durability of zombies varies wildly depending on background.

Personally, I think fast, manouvrable cutting weapons like glaives, hewing spears and dane axes would be best, because they have enough leverage to slice through skulls, but are quick to move to the next zombie and have decent reach (not so much the dane axe for reach). Halberds probably don't need the extra spikes, just focus on an optimal cutting blade like a bardiche.

I like the idea of combining winged spears to hold the zombies at bay (with shields?) and hewing weapons to fell them for good. Backing this up with armour and firearms for ranged support could make a solid bastion of infantry to repel zombies.

I am also not sure why "rapid firing" weapons like the M16 wouldn't be effective- they do a lot of tissue damage with the tumbling and cavitation, even if the bullet itself isn't that big. 5.56 NATO is pretty good at depositing energy into tissue. Even without killing with headshots, damging the skeletal structure and tissue of zombies is going to slow them down and make them drastically more ineffective. I think shotguns firing frangible rounds could make good close-range support too.

Mr Beer
2018-02-13, 05:11 PM
Reading Black Hawk Down, it seems that the 5.56 round ultimately killed a lot of enemy combatants, but didn't tend to instantly knock them down or smash limbs off, which I think is what you want for zombies if they are not headshots. Note: I am aware that a single source written by a journalist is not a reliable metric for judging military efficacy so I'm not exactly dogmatic on this point.

I guess that heavy machine gun fire would shred zombies...I know by the book zombies walk off bullets but I still think an M2 is going to stop a pack in its tracks.

Shotguns can also fire a 'bolo round' I think it's called, which might be used to decapitate zombies.

Max_Killjoy
2018-02-13, 05:31 PM
Reading Black Hawk Down, it seems that the 5.56 round ultimately killed a lot of enemy combatants, but didn't tend to instantly knock them down or smash limbs off, which I think is what you want for zombies if they are not headshots. Note: I am aware that a single source written by a journalist is not a reliable metric for judging military efficacy so I'm not exactly dogmatic on this point.

I guess that heavy machine gun fire would shred zombies...I know by the book zombies walk off bullets but I still think an M2 is going to stop a pack in its tracks.

Shotguns can also fire a 'bolo round' I think it's called, which might be used to decapitate zombies.

To be blunt, the right 50 BMG ammo will turn a torso or head sized area of the body it hits into high-speed salsa. Some of the snipers who've scored long-range hits with 50s describe 4 limbs and a head sort of coming off in different directions.

Zombies? At half a mile? Not a problem.

Storm Bringer
2018-02-13, 05:55 PM
if we are going to be serious about zombies, the real question is how much of the area you want standing afterwards. depending on the exact nature of the strain, some weapons become more effective and others less so. since fat burns quite well, incendiaries would likely thin the horde out nicely, particularly as the zombies wont try and put themselves out. even if they don't suffer burns to the head that kills them, charred limbs are nearly useless if the muscles have been literally cooked. sure, you might end up with a pile of semi-charred corpses you need to pick though and spear the still going ones, but it would turn a battle into a clean up operation.



I am also not sure why "rapid firing" weapons like the M16 wouldn't be effective- they do a lot of tissue damage with the tumbling and cavitation, even if the bullet itself isn't that big. 5.56 NATO is pretty good at depositing energy into tissue. Even without killing with headshots, damging the skeletal structure and tissue of zombies is going to slow them down and make them drastically more ineffective. I think shotguns firing frangible rounds could make good close-range support too.


its not that they wont work, its just that a 7.62 battle rifle would do be better, assuming you can keep the troops rate of fire down.

I think shotguns are a poor choice, as you end up with crawling zombies, bits of head trying to bite, ect, or you end up with a slow firing gun that doesn't have more leathality than the issue rifle if you stick to headshots. I'd say that it complicates the cleanup, but realistically the troops are going to shoot every head they see, just in case, so not that much. if you could get some seriously wide spread type guns, you might be able to increase hit chance and even drop several close targets, but youd need a much wider spread than any normal shotgun.

.50 BMG might be a ok choice if you can keep the rounds at the right height. i know it can cause fatal concussions in targets even if it just passes by them, but i don't think it would work on zombies like that, and it wouldn't be worth the weight. you'd still need a rifle squad to pick off the stragglers and deal with the half-zombies and such.

Gnoman
2018-02-13, 07:36 PM
No matter what sort of pseudo-science virus or dark magic is animating the thing, one constant is that severed limbs and crushed bones don't grow back. Indeed, this is key to a lot of the classic portrayals.

Any modern rifle round (except the .22LR that at least one zombie-book author idiotically worshipped) is going to deal with a zombie effectively at a range far longer than the zombie can pose a threat. This is because a solid hit from such a round on a bone will shatter said bone, and a shooter with any skill is going to be able to deliver such hits against such easy targets at a range of at least 300 meters. It doesn't matter if the thing is still twitching. If a leg is shot off, or a pelvis shattered, it is neutralized, and can be dispatched at leisure.

Mr Beer
2018-02-13, 09:10 PM
.50 BMG might be a ok choice if you can keep the rounds at the right height. i know it can cause fatal concussions in targets even if it just passes by them, but i don't think it would work on zombies like that, and it wouldn't be worth the weight. you'd still need a rifle squad to pick off the stragglers and deal with the half-zombies and such.

A rifle squad is going to have to be one of the best possible choices for dealing with zombie hordes because accurate kills at distance is a tactically sound choice.

That said, I think any rifle group is going to want a BMG if possible*, compared to the option of not having a BMG. Probably not for picking off small groups but against swarms, so you can shatter attempts to swamp the soldiers' position and for quickly breaking large groups so they can be killed carefully later.

I don't think that accuracy would be too paramount with a BMG, if you just hose them down at upper thigh/abdominal height, you are going to shatter the femur/pelvis/spine area which will have an adverse effect on their ability to walk.

Slow, clustered zombie groups are going to be pretty much an amazing target for machine gun fire.

* Maybe not if they have to carry it everywhere by hand.

Carl
2018-02-13, 10:53 PM
No matter what sort of pseudo-science virus or dark magic is animating the thing, one constant is that severed limbs and crushed bones don't grow back. Indeed, this is key to a lot of the classic portrayals.

Any modern rifle round (except the .22LR that at least one zombie-book author idiotically worshipped) is going to deal with a zombie effectively at a range far longer than the zombie can pose a threat. This is because a solid hit from such a round on a bone will shatter said bone, and a shooter with any skill is going to be able to deliver such hits against such easy targets at a range of at least 300 meters. It doesn't matter if the thing is still twitching. If a leg is shot off, or a pelvis shattered, it is neutralized, and can be dispatched at leisure.

Nope not even close, so long as enough of one forearm or a certain amount of lower leg is left they can crawl at you. You might slow a zombie down that way, but your not going to hit bone on every shot, and your not going to stop it.

It'll stop a human sure because trying to move will likely compound the damage fatally and the sheer pain would be disabling. Neither is somthing a zombie has a problem with.


To be blunt, the right 50 BMG ammo will turn a torso or head sized area of the body it hits into high-speed salsa. Some of the snipers who've scored long-range hits with 50s describe 4 limbs and a head sort of coming off in different directions.

Zombies? At half a mile? Not a problem.

Oh absolutely, i man you now have 5 bits crawling towards you but you've hurt them bad. The real issue is sustained RoF. a 50 cal HMG just can't keep pouring the lead out for very long at high rate of fire.

I've actually been part of a discussion elsewhere in the past about this. There's some stuff the book definitely gets wrong, (a lot of it looks like bad knowledge of a thing or two on the authors part combined with arms tech marches on in some cases). At the same time the book combines some serious screw ups, (complacency on the military side), with a real issue that there's just so many of them. The first big battle of the book if i recall the details from said past discussion, (never read it, sounds way too depressing for my tastes), involved most of the population of new york, (our estimate was around 5 million as i recall, i don't think the book specifies), spread over a several mile wide front vs i believe a few thousand ground troops a couple of aircraft and a squadron of tanks. hey expected the aircraft to take out most of them in one pass and it didn't work out, (not surprising, you start dropping iron bombs in densely packed flesh and the sheer amount of high water content matter in there is really going to degrade the kill radius, same problem for any shrapnel producing ordnance really, it will only penetrate so far before all that flesh stops it).

Tiadoppler
2018-02-13, 11:30 PM
Nope not even close, so long as enough of one forearm or a certain amount of lower leg is left they can crawl at you. You might slow a zombie down that way, but your not going to hit bone on every shot, and your not going to stop it.


Although, to be fair, it'd take the one-armed zombie quite a while to drag itself 300 feet (100 yards, AKA close range) much less 1500 feet (500 yards, a reasonable distance for an intermediate cartridge) over rough terrain/the bodies of other zombies. Slowing down a zombie is a good thing to do.




Oh absolutely, i man you now have 5 bits crawling towards you but you've hurt them bad.


If dismembered zombie arms still function as independent units, then all the stuff re: destroying the brain/severing the head is irrelevant. Also, a severed head pulling itself along by the jaw isn't much of a threat to someone with boots, or a stepladder to stand on.




the aircraft to take out most of them in one pass and it didn't work out, (not surprising, you start dropping iron bombs in densely packed flesh and the sheer amount of high water content matter in there is really going to degrade the kill radius, same problem for any shrapnel producing ordnance really, it will only penetrate so far before all that flesh stops it).


Hmm. Wouldn't airbursting ordinance solve most of that?

Gnoman
2018-02-13, 11:41 PM
Nope not even close, so long as enough of one forearm or a certain amount of lower leg is left they can crawl at you. You might slow a zombie down that way, but your not going to hit bone on every shot, and your not going to stop it.


You don't have to stop it with one shot. Shatter a leg and the zombie's too slow to be a threat. Dispatch or cripple every threatening zombie, then walk up to point blank range and put them down with headshots. Or, for that matter, simply shuffle away, since the only way for it to catch you at this point is for you to let it.

gkathellar
2018-02-14, 07:21 AM
You don't have to stop it with one shot. Shatter a leg and the zombie's too slow to be a threat. Dispatch or cripple every threatening zombie, then walk up to point blank range and put them down with headshots. Or, for that matter, simply shuffle away, since the only way for it to catch you at this point is for you to let it.

This works if your goal is to escape, but if your objective is to clear the area, you are going to need to find all of those crippled zombies and deal with them individually at a less convenient range. That's not too bad in urban areas where they're easy to see on the ground, but it's not really a good approach if there's, say, tall grass.

Really, if you can prepare the battlefield at all (and you should be able to, zombies are dumb), consistent headshots should not be all that difficult. In general, the only challenges zombies would present are preparation and attrition. I'd even suggest that the absolute ideal solution to the standard zombie horde would be "build little wheeled combat drones, pick them off from the comfort and safety of your bunker."

Sapphire Guard
2018-02-14, 08:16 AM
No matter what sort of pseudo-science virus or dark magic is animating the thing, one constant is that severed limbs and crushed bones don't grow back. Indeed, this is key to a lot of the classic portrayals.

Resident evil (game) zombies can grow back limb and heads, it's why they're one of the few versions that are a convincing threat.


This works if your goal is to escape, but if your objective is to clear the area, you are going to need to find all of those crippled zombies and deal with them individually at a less convenient range. That's not too bad in urban areas where they're easy to see on the ground, but it's not really a good approach if there's, say, tall grass.

Depends. Seeing as they don't hide, they're not going to be that hard to find, and if you don't need to clear the area completely today, a broken leg means that the zombie is going to wear its fingers to nubs in short order just by moving around.

DrewID
2018-02-14, 10:31 AM
Resident evil (game) zombies can grow back limb and heads, it's why they're one of the few versions that are a convincing threat.

This illustrates one of the problems with this discussion (entertaining though it is): there is no single definition of "zombie". In fact, I suspect some later zombies were assigned new powers (like the aforementioned regrowth) just to eliminate a previously valid solution and present an ever-greater threat to the protagonists/players.

DrewID

Haighus
2018-02-14, 10:47 AM
This illustrates one of the problems with this discussion (entertaining though it is): there is no single definition of "zombie". In fact, I suspect some later zombies were assigned new powers (like the aforementioned regrowth) just to eliminate a previously valid solution and present an ever-greater threat to the protagonists/players.

DrewID
This is true, but then with the nature of the thread we should only really consider "plausible" zombies. So as an example, we can discount zombies that have limbs that continue to hunt you independently when detached, but die when the brain is destroyed (as mentioned above) because this set up basically requires magic to connect the limbs to the brain, rather than nerves.

We can also assume detached limbs should have no way of hunting by themselves anyway, even if technically functional, due to only having touch sensation available. So don't step on them, but they won't know a human is standing 100m away.

Further to this, as mentioned, zombies are flesh. Short of magic to hold them together, they are subject to the same limitations of other flesh creatures, and crawling across rough terrain is going to damage them and tear flesh away, wear down bone etc. Crawlers produced at sufficient distance are going to be no threat after some crawled distance in quite a few terrain types, like mountain scree and rocks, or tarmac. Dense barbed wire would also shred them.

Removing a zombies head should be pretty effective, because even if the head is still functionally able to fight, detaching it from the body would prevent it from doing almost anything. Adequately damaging/occluding it's ears and eyes (and maybe nose) should prevent it from even detecting an enemy at a distance. Perhaps extra powerful flashbangs and concussive weapons could be used to blind and deafen zombies.

Regrowth is maybe plausible if they eat stuff, but regrowing a head is pretty unlikely, and also not an option if the brain is the source of the control.

Frankly, I think zombies are a pretty weak threat in any plausible (non-magical) scenario beyond contaminating a city or two in the initial rush. The exception is if the infection is able to lie dormant and infect most the population before it activates. Then the survivors would be hugely outnumbered.

Tiadoppler
2018-02-14, 11:55 AM
This is true, but then with the nature of the thread we should only really consider "plausible" zombies. So as an example, we can discount zombies that have limbs that continue to hunt you independently when detached, but die when the brain is destroyed (as mentioned above) because this set up basically requires magic to connect the limbs to the brain, rather than nerves.

We can also assume detached limbs should have no way of hunting by themselves anyway, even if technically functional, due to only having touch sensation available. So don't step on them, but they won't know a human is standing 100m away.


How about a zombie in which radio-controlled nanotech replaces the nervous system? A grey-goo infiltrator might be perfectly able to control severed limbs with gps and remote footage.

Zombie is:
Living person with an incurable illness that causes cannibalism and aggressiveness? Normal firearms are still a good answer.
Magical undead? It all depends on what magic it is/how the magic works in the story.
Technologically controlled body? Depends on the tech. Could be a single implant controlling the physical motions of a living person or corpse, or anything up to a person-shaped mass of nanites.

Max_Killjoy
2018-02-14, 12:25 PM
Somehow I think this zombie thing has reached the point of needing its own thread.

Tiadoppler
2018-02-14, 12:49 PM
What do you think would be the ideal modern small arm for disabling/destroying a human-scale combat drone/robot?




By human scale I mean:


A ground vehicle that uses a reliable current-day technology for motive force (legs, wheels, treads, "hamsterball", wiggling or flipping, etc.)

Weight/size should be comparable to a human (although probably smaller and denser if it's using any amount of armor). Say 100 to 400 lbs and height 2'-5'

Power plant could be an engine or a battery system.

Control systems are optimized for durability and some redundancy. The brain isn't in the "head".



My thoughts are:
Many modern projectile weapons are designed for damaging soft, wet targets that transmit shock very well. A large part of the "damage" of a bullet is based on the shockwave of the bullet passing through soft tissue. A robot wouldn't be nearly as vulnerable to that, and a bullet might only damage the parts that are directly hit. Even a thin layer of hardened metal armor at a good deflection angle would reduce the effect of small arms fire.

Carl
2018-02-14, 01:00 PM
Although, to be fair, it'd take the one-armed zombie quite a while to drag itself 300 feet (100 yards, AKA close range) much less 1500 feet (500 yards, a reasonable distance for an intermediate cartridge) over rough terrain/the bodies of other zombies. Slowing down a zombie is a good thing to do.






If dismembered zombie arms still function as independent units, then all the stuff re: destroying the brain/severing the head is irrelevant. Also, a severed head pulling itself along by the jaw isn't much of a threat to someone with boots, or a stepladder to stand on.






Hmm. Wouldn't airbursting ordinance solve most of that?

1. Oh absolutely, but again you've got to remember at the first big battle it was several million vs several thousand, from what i understand dealing with that level of outnumbering was a consistent issue. In that situation it doesn't matter if it takes tem 4 times as long to close with you, you aren't going to have the sustained RoF for it to matter. An M16 for example is limited to 12-15rpm sustained according to a quick websearch. Even crawling at half shambling speed, (which based on most depictions is about as fast or slightly faster than human walking speed), they'd still cross 500 meters in 10-12 minutes. When your outnumbered several thousand to 1 thats a problem. Particularly in a containment situation where your trying to keep them from getting somwhere, which is basically what happened in the first book, the us military was tid uop elsewhere dealing with the whole nation going to bits and a small force was sent to keep the completely overrun city of NY quarantine.

2. Thinking about it in the context of world war z i believe you right, been a while and was tired when i wrote that. That said there's no such thing as a fully realistic zombie just due to energy problems.

3. Absolutely, and it was one of the authors gafes actually, there where times he treated airbursting weapons, (notably bomblet warheaded MLRS rockets), as if they where detonating on contact with the ground.


You don't have to stop it with one shot. Shatter a leg and the zombie's too slow to be a threat. Dispatch or cripple every threatening zombie, then walk up to point blank range and put them down with headshots. Or, for that matter, simply shuffle away, since the only way for it to catch you at this point is for you to let it.

Again depends on he numbers disparity, thousands to one where very round of ammo is precious and where thee are allways more coming on behind and it's really important you put the highest % possibble down.


This illustrates one of the problems with this discussion (entertaining though it is): there is no single definition of "zombie". In fact, I suspect some later zombies were assigned new powers (like the aforementioned regrowth) just to eliminate a previously valid solution and present an ever-greater threat to the protagonists/players.

DrewID

Indeed.


This is true, but then with the nature of the thread we should only really consider "plausible" zombies. So as an example, we can discount zombies that have limbs that continue to hunt you independently when detached, but die when the brain is destroyed (as mentioned above) because this set up basically requires magic to connect the limbs to the brain, rather than nerves.

We can also assume detached limbs should have no way of hunting by themselves anyway, even if technically functional, due to only having touch sensation available. So don't step on them, but they won't know a human is standing 100m away.

Further to this, as mentioned, zombies are flesh. Short of magic to hold them together, they are subject to the same limitations of other flesh creatures, and crawling across rough terrain is going to damage them and tear flesh away, wear down bone etc. Crawlers produced at sufficient distance are going to be no threat after some crawled distance in quite a few terrain types, like mountain scree and rocks, or tarmac. Dense barbed wire would also shred them.

Removing a zombies head should be pretty effective, because even if the head is still functionally able to fight, detaching it from the body would prevent it from doing almost anything. Adequately damaging/occluding it's ears and eyes (and maybe nose) should prevent it from even detecting an enemy at a distance. Perhaps extra powerful flashbangs and concussive weapons could be used to blind and deafen zombies.

Regrowth is maybe plausible if they eat stuff, but regrowing a head is pretty unlikely, and also not an option if the brain is the source of the control.

Frankly, I think zombies are a pretty weak threat in any plausible (non-magical) scenario beyond contaminating a city or two in the initial rush. The exception is if the infection is able to lie dormant and infect most the population before it activates. Then the survivors would be hugely outnumbered.

Well thats one aea from what i recall the book takes a few liberties, (zombies on fire for example will keep coming for a considerable length of time, and crawling doesn't seem to mess them up overmuch from what i recall of the last discussion).

Your last sentence is a variation on what happened. Everyone in NY where it started didn't realise what they where dealing with at first, and once they did where complacent about their ability to deal with them. By the time of the first big battle NY was lost and the infection had spread around the world. Leaving everyone's military overstretched as they tried to cope with the sheer scale of the problem.

Mike_G
2018-02-14, 01:59 PM
1. Oh absolutely, but again you've got to remember at the first big battle it was several million vs several thousand, from what i understand dealing with that level of outnumbering was a consistent issue. In that situation it doesn't matter if it takes tem 4 times as long to close with you, you aren't going to have the sustained RoF for it to matter. An M16 for example is limited to 12-15rpm sustained according to a quick websearch. Even crawling at half shambling speed, (which based on most depictions is about as fast or slightly faster than human walking speed), they'd still cross 500 meters in 10-12 minutes. When your outnumbered several thousand to 1 thats a problem. Particularly in a containment situation where your trying to keep them from getting somwhere, which is basically what happened in the first book, the us military was tid uop elsewhere dealing with the whole nation going to bits and a small force was sent to keep the completely overrun city of NY quarantine.

2. Thinking about it in the context of world war z i believe you right, been a while and was tired when i wrote that. That said there's no such thing as a fully realistic zombie just due to energy problems.

3. Absolutely, and it was one of the authors gafes actually, there where times he treated airbursting weapons, (notably bomblet warheaded MLRS rockets), as if they where detonating on contact with the ground.



Again depends on he numbers disparity, thousands to one where very round of ammo is precious and where thee are allways more coming on behind and it's really important you put the highest % possibble down.



Indeed.



Well thats one aea from what i recall the book takes a few liberties, (zombies on fire for example will keep coming for a considerable length of time, and crawling doesn't seem to mess them up overmuch from what i recall of the last discussion).

Your last sentence is a variation on what happened. Everyone in NY where it started didn't realise what they where dealing with at first, and once they did where complacent about their ability to deal with them. By the time of the first big battle NY was lost and the infection had spread around the world. Leaving everyone's military overstretched as they tried to cope with the sheer scale of the problem.

Explosives, including airstrikes, artillery, mines, even IEDs do a lot of damage to bodies. Not talking shrapnel, just like, bodies blow into bits and missing important pieces. A zombie with no arms can bite, maybe, but how easy is it to bite somebody when you can't grab onto them? Legless zombies are very much reduced as a threat, and if the concussion or heat of an explosion ruins the eyes or ears, well, how can it find you? The military has mortars at very low levels of organization, and grenade launchers at the fire team level.

Don't discount napalm for airstrikes either. Or, just saying, NYC is largely an island. Just write it off and treat it like Dresden in WWII.

As far as battlefield prep, if zombies are slow and dumb, that's easy. Something like barbed wire, or even just pits and snares and bear traps or an abbatis can slow down a horde. Put something flammable in the kill zone which is full of the obstacles. Even something as simple as straw or just use a field of dry grass if weather and conditions are right. Bait the horde into the area, let the obstacles slow them down and make them bunch up, then torch it.

Area denial and channeling would be huge factors in fighting a slow, dumb mob. As long as you can fall back, which our very very mechanized force could, you should be able to live to fight another day. Just slowly drive the Humvee or LAV or Bradley away at a walking pace while shooting behind you. And a dumb mob is easy to lure into ambush or make them chase your LAV 25 which is driving away at 5 mph hosing them with 25 mm fire.

As far as small arms, the 5.56 does not do a lot of structural damage. It can tumble around and wreck a lot of internal organs, but that's not an issue for zombies. So head shots it is. Now that almost everybody has optics, head shots aren't all that hard. Forget auto fire spray and pray. Not many of those hits would be disabling. I'd tell my troops to keep the selector on semi and only take head shots unless you have a grenade launcher. Even a bigger rifle round is only so good. A broken leg might not stop them, so just take the head shot. If you can't hit the head at 50 yards with modern rifles and ACOG sights, you need to go to bartending school. Make sure every soldier has a handgun and tomahawk or cutlass or something for melee than can split a skull or take off an arm in addition to the rifle and basic body armor.

Vehicle and crew served weapons like the Mk 19 automatic grenade launcher or the 25 mm gun on an APC would shred zombies, and vehicle mounted ones would be almost impossible to catch.

In conclusion, slow, dumb zombies wouldn't be a very serious threat to a modern military, even in huge numbers. Fast or smart zombies might be different

Now, bad things happen. Armies with breechloading rifles have been wiped out by armies with spears when they were dumb enough, so you can be too cocky and stupid, but I'd rather fight a few thousand zombies with my infantry company than 100 soldiers.

Mendicant
2018-02-14, 04:21 PM
That is interesting - and while it may be true for the Ottomans, it's almost the opposite in Europe where the city-States and Free Cities (very small polities of anywhere from 10,000 to maybe 100,000 people) were way ahead of the larger Kingdoms on the use of gunpowder weapons. France eventually got up to speed probably much in the same way as the Ottomans did - due to contending for a long time with Flemish city-states (as part of the Burgundian Duchy).

But cities like Ghent created weapons in the very early 15th Century (like this beast (https://en.wikipedia.org/wiki/Dulle_Griet)) that France couldn't produce until 100 years later.

The City-State of Venice probably had the largest, most efficient, most technologically advanced and most productive arms industry in all of Latinized Europe, the famous "Arsenal", known chiefly for it's ability to produce a warship in a single day, but they also routinely produced large quantities of high quality firearms - by the hundreds when most polities of their day were producing guns in the scores or dozens.

https://en.wikipedia.org/wiki/Venetian_Arsenal#History

Nuremberg was another leading producer, making large quantities of the most advanced firearms in the 15th and 16th Centuries and leading the way with technology like wheel locks, rifling, and breach loading firearms.



I guess when it comes to arms production there are two ways to go about it - the large Centralized State or the small but free and self-administered city.


That is an interesting distinction. I wonder if some of that had to do with how rich cities like Ghent were. In terms of administration, city states generally were much more cohesive than hodge-podge agrarian fiefdoms, so perhaps their wealth could substitute for the scale part of the equation.

Gnoman
2018-02-14, 04:31 PM
What do you think would be the ideal modern small arm for disabling/destroying a human-scale combat drone/robot?




By human scale I mean:


A ground vehicle that uses a reliable current-day technology for motive force (legs, wheels, treads, "hamsterball", wiggling or flipping, etc.)

Weight/size should be comparable to a human (although probably smaller and denser if it's using any amount of armor). Say 100 to 400 lbs and height 2'-5'

Power plant could be an engine or a battery system.

Control systems are optimized for durability and some redundancy. The brain isn't in the "head".



My thoughts are:
Many modern projectile weapons are designed for damaging soft, wet targets that transmit shock very well. A large part of the "damage" of a bullet is based on the shockwave of the bullet passing through soft tissue. A robot wouldn't be nearly as vulnerable to that, and a bullet might only damage the parts that are directly hit. Even a thin layer of hardened metal armor at a good deflection angle would reduce the effect of small arms fire.

Such a system would have very little "empty" space inside. Anything that isn't armor or weapon will be control boards, batteries, motors, or whatever. If you hit it, and penetrate it, you WILL hit something important.

Any system on that scale would be easily defeated by an anti-material rifle. You're not going to be able to get enough armor in a package that size to keep out a .50 BMG round or equivalent, and an Armor Piercing Incendiary round would be extremely effective against the internal components due to the incendiary component. One shot would probably be enough to "cook" the electronics (particularly since the armored case would trap the heat) and take out the unit.

Failing that, full-rifle Armor Piercing rounds would probably penetrate. You'd probably have to be a lot closer, but a typical such round will penetrate up to 7mm of hardened steel. For something the size you're talking about, that level of protection approaches the "prohibitive" category for the entire unit, although an armored "braincase" is plausible. Without a brain-kill, you'd have to put quite a few rounds into it for a full disable, but that's what automatic fire is for, and rifle-caliber machineguns are extremely common in combat zones.

Galloglaich
2018-02-14, 04:34 PM
Explosives, including airstrikes, artillery, mines, even IEDs do a lot of damage to bodies. Not talking shrapnel, just like, bodies blow into bits and missing important pieces.
Don't discount napalm for airstrikes either. Or, just saying, NYC is largely an island. Just write it off and treat it like Dresden in WWII.

As far as battlefield prep, if zombies are slow and dumb, that's easy. Something like barbed wire, or even just pits and snares and bear traps or an abbatis can slow down a horde. Put something flamable in the kill zone which is full of the obstacles. Even something as simple as straw or just use a field of dry grass if weather and conditions are right. Bait the horde into the area, let the obstacles slow them down and make them bunch up, then torch it.

I agree with this. People have a generally low grasp of military realities, and especially with the two key points of 1) how utterly devastating things like airstrikes and artillery are, and 2) how incredibly cunning people are in avoiding it. During Vietnam the NVA etc. were doing a large proportion of their fighting underground as an adaptation to American artillery and air power. Today Taliban and various Islamist groups in Afghanistan, Syria etc. are doing the same thing. This is how they have been able to drag the wars on so long.

If they were 'stupid' like the way most zombies are portrayed, even I think if they were fast (more on that in a second) I think modern armies could adapt to them very quickly and wipe out huge numbers of them - enough to account for vast populations of entire regions being 'zombified' - by things like channeling them into kill zones (with the help of concrete barriers, walls, barbed wire, pits dug by engineers and so on) where they could then be wiped out by artillery, air support, heavy cannon like on tanks and bradleys, .50 cals and so on.

I think artillery and mortars in particular would be one of the best ways to wipe out zombies. Having seen them in action (just in firepower demonstrations at ranges) I was shocked and so were most everybody else. Once you have the target range and distance dialed in, it's amazing how many shells they can pour down on them, and even a relatively small mortar shell (80mm) can pretty easily blow apart bodies. Even light (50-60mm) mortars can do this and from a surprisingly long range.

The cliche scene where you have some kind of fortified building or natural feature, with people besieged by zombies, would be very different if anyone involved on the human side had access to say a National Guard armory with mortars or artillery pieces. Even a few 60mm mortars with a reasonable amount of ammunition could fairly quickly reduce a mob of tens of thousands of zombies to a blown up mob of pieces ... presumably still dangerous and I would think a major disease risk, but not nearly as much of an emergency and much easier to deal with in an escape.

If you combine this with modern recon capabilities, drones for example or helicopters, you could figure out where the zombie mobs were well in advance (even 'fast' zombies move at the pace of a running human which is slow compared to a vehicle or a helicopter) and you could create kill-zones.

I think noisemakers would also be a fairly easy way to lure zombie hordes into kill zones, or even run them over cliffs and into bogs.

I think the main thing missing from the (I think now hundreds) of zombie movies and show episodes I have seen is that they tend to focus on the small caliber automatic weapons (because gamers and geeks love 'em) but they don't know about artillery so they don't include that. Artillery is what makes Max Brooks type scenarios seem unlikely to me.

G

Tiadoppler
2018-02-14, 05:00 PM
Such a system would have very little "empty" space inside. Anything that isn't armor or weapon will be control boards, batteries, motors, or whatever. If you hit it, and penetrate it, you WILL hit something important.

Any system on that scale would be easily defeated by an anti-material rifle. You're not going to be able to get enough armor in a package that size to keep out a .50 BMG round or equivalent, and an Armor Piercing Incendiary round would be extremely effective against the internal components due to the incendiary component. One shot would probably be enough to "cook" the electronics (particularly since the armored case would trap the heat) and take out the unit.

Failing that, full-rifle Armor Piercing rounds would probably penetrate. You'd probably have to be a lot closer, but a typical such round will penetrate up to 7mm of hardened steel. For something the size you're talking about, that level of protection approaches the "prohibitive" category for the entire unit, although an armored "braincase" is plausible. Without a brain-kill, you'd have to put quite a few rounds into it for a full disable, but that's what automatic fire is for, and rifle-caliber machineguns are extremely common in combat zones.


Unlike a living being, a robot does not go into shock when severely damaged. There's the possibility of using genetic algorithms to allow a robot to continue functioning (at decreasing effectiveness) as it is damaged. If you disable one weapons system, or one ammunition feed, or one turret motor, a well programmed combat robot might be able to continue fighting. Anti-material would definitely work. Do you think Armor Piercing Incendiary rounds would stay inside the robot long enough to cook the electronics? I'd be worried that they'd overpenetrate and shoot out the far side without spending enough time inside to transfer their heat.

A "tank-like" robot might be able to have a level of armor equivalent to 7mm of hardened steel over it's entire body without too much weight penalty. With properly angled composite armor plates, it might be very tough to damage even with AP small arms fire, but I agree that it's unlikely that too much emphasis would be placed on armoring a small, cheap, combat robot. Mass production, and mass deployment, is what would make them truly terrifying.

Think of a robot that's roughly the size of a car tire lying flat on the ground. It's got treads underneath, an articulated sensor "tower" that can be raised over obstructions, an array of cameras and sensors in the shell and an armored carbine-caliber turret on the top of the shell, and is deployed by the hundreds.

Mr Beer
2018-02-14, 05:20 PM
1. Re. zombies, yeah if they are traditional slow stupid zombies, armed forces will use kill zone tactics against them. You can pack an enormous number of humans into, say, a soccer stadium if they are just going to walk in there and not worry about personal space or the need to breathe. We can then bomb them apart or probably just as easy, simply set fire to them.

2. Re. armoured robots, I think drones are a much likelier battlefield robot, because a) they are already widely used and b) it's harder to hit a fast flying object than a slow ground one.

An actual tank that's fully robotic is going to happen at some time of course, as far fighter planes. Both of these suffer design issues because of the need to have humans in them.

Battlefield infantry is probably the last thing that will get replaced by robots, of course it will happen if technology progresses long enough.

Lapak
2018-02-14, 05:25 PM
Think of a robot that's roughly the size of a car tire lying flat on the ground. It's got treads underneath, an articulated sensor "tower" that can be raised over obstructions, an array of cameras and sensors in the shell and an armored carbine-caliber turret on the top of the shell, and is deployed by the hundreds.
I hope those were really cheap, because around here I’d just open up the fire hydrants to flood the streets and watch them short out / get washed away. ;)

Carl
2018-02-14, 06:09 PM
Explosives, including airstrikes, artillery, mines, even IEDs do a lot of damage to bodies. Not talking shrapnel, just like, bodies blow into bits and missing important pieces. A zombie with no arms can bite, maybe, but how easy is it to bite somebody when you can't grab onto them? Legless zombies are very much reduced as a threat, and if the concussion or heat of an explosion ruins the eyes or ears, well, how can it find you? The military has mortars at very low levels of organization, and grenade launchers at the fire team level.

Don't discount napalm for airstrikes either. Or, just saying, NYC is largely an island. Just write it off and treat it like Dresden in WWII.

As far as battlefield prep, if zombies are slow and dumb, that's easy. Something like barbed wire, or even just pits and snares and bear traps or an abbatis can slow down a horde. Put something flammable in the kill zone which is full of the obstacles. Even something as simple as straw or just use a field of dry grass if weather and conditions are right. Bait the horde into the area, let the obstacles slow them down and make them bunch up, then torch it.

Area denial and channeling would be huge factors in fighting a slow, dumb mob. As long as you can fall back, which our very very mechanized force could, you should be able to live to fight another day. Just slowly drive the Humvee or LAV or Bradley away at a walking pace while shooting behind you. And a dumb mob is easy to lure into ambush or make them chase your LAV 25 which is driving away at 5 mph hosing them with 25 mm fire.

As far as small arms, the 5.56 does not do a lot of structural damage. It can tumble around and wreck a lot of internal organs, but that's not an issue for zombies. So head shots it is. Now that almost everybody has optics, head shots aren't all that hard. Forget auto fire spray and pray. Not many of those hits would be disabling. I'd tell my troops to keep the selector on semi and only take head shots unless you have a grenade launcher. Even a bigger rifle round is only so good. A broken leg might not stop them, so just take the head shot. If you can't hit the head at 50 yards with modern rifles and ACOG sights, you need to go to bartending school. Make sure every soldier has a handgun and tomahawk or cutlass or something for melee than can split a skull or take off an arm in addition to the rifle and basic body armor.

Vehicle and crew served weapons like the Mk 19 automatic grenade launcher or the 25 mm gun on an APC would shred zombies, and vehicle mounted ones would be almost impossible to catch.

In conclusion, slow, dumb zombies wouldn't be a very serious threat to a modern military, even in huge numbers. Fast or smart zombies might be different

Now, bad things happen. Armies with breechloading rifles have been wiped out by armies with spears when they were dumb enough, so you can be too cocky and stupid, but I'd rather fight a few thousand zombies with my infantry company than 100 soldiers.

1. Oh absolutely, but the absolute minimum pulp radius is quite small, a 1000lb bomb only has a radius of 5m. Now given fles is less tough than soil i'd say it's probably greater than that, and all the body parts are going to become shrapnel. My point was more you can't expect a kill radius of 50m on a 155 shell, that value was derived against sensibly organised troops. Against tight packing the sheer amount of mass in the way is going to really degrade it, anything within close rnage of the blast is toast, but when you'v got carpets of zombies covering multiple square miles it's going to take a LOT of heavy ordnance to get them all.

2. I think most of the rest of what you have to say once again is just not getting your head around how outnumbered the military was. it wouldn't be your rifle company, (about 150-200 troops i believe from memory), vs a few thousand zombies. It would be your rifle company vs a half a million zombies spread across a 4 mile front stacked a third of a mile deep. Yeah you grenades and mortars and vehicle mounted weapons will tear them up, but how many can you kill before you start running out of ammo?


I hope those were really cheap, because around here I’d just open up the fire hydrants to flood the streets and watch them short out / get washed away. ;)

:biggrin: Lol, just Lol.

KiwiQuest
2018-02-14, 06:20 PM
Regarding the zombie apocalypse, it would require some equipment and munition changes, but I am fairly certain the military could adapt to it.

A lot of damage from artillery and mortars comes from shrapnel and not the actual explosion, because it’s the most efficient when it comes to regular warfare, but changing the ratio would not be particularly difficult. Not to mention that – to a person/zombie standing upright – the shrapnel damage is not insignificant (even if the only meaningful damage is completely disabling a limb or head trauma). However, delivering shrapnel is much more efficient in terms of grenade size, so I think we’d see a shift towards larger caliber artillery.
Bear in mind that as we speak, more effective types of explosives are being researched, partially driven by the need(/wish) to deliver them by drones. While they are more expensive, they are also expected to deliver up to 10 times the power/volume. This research could potentially become much more relevant in the zombie scenario, and it would radically increase the effectiveness of any indirect weapon system.

Thermobaric weaponry (fuel-air explosives) would probably work very well, as they deliver a larger shock wave than normal munitions. While the zombies would be indifferent to the rarefaction (creation of a vacuum which ruptures the lungs), the high temperatures and burning could potentially be significant as well.

IEDs and similar would also prove very effective if zombies could be herded or lured into appropriate areas. The real limit to explosive force is usually in the delivery method – you have to somehow propel it towards the target, and/or hide where you place from the enemy. With zombies neither is an issue, so the only limit to the size of your IED is the time available for digging and the amount of explosives at hand.

Finally, the zombies lack a real answer to even lightly armoured, tracked vehicles. I imagine existing vehicles could be adapted to simply run over a large amount of zombies with little to no threat to themselves – even if they’re stuck, they can realistically stay holed up inside the vehicle for days while awaiting rescue.
Consider something like a mine flail. An armoured vehicle with a spinning wheel of chains with fist-sized iron balls at the ends would probably make short work of a large amount of zombies if employed correctly. A main battle tank might even be better employed simply running zombies over than actually firing.

My initial reaction is I wouldn’t even bother with melee weapons at all. Unless the zombies are somehow sneakier than expected, or much faster than a human being, I simply don’t see large-scale melee combat breaking out. Small arms would probably see a shift towards higher caliber rounds with more kinetic energy delivered (so even if you don’t hit it in the head and kill it, you at least stop it momentarily, knock it down for a second or destroy a limb), but they’d still be much more relevant than duking it out face to face.

It is probably true that for a high enough ratio of zombies-to-soldiers, especially if preparation time is limited or non-existent, there’s a point where everyone gets overrun, but it requires a very specific scenario to get to that point I would say. I guess some sort of dormant virus that zombifies a huge part of the population at once. Anything less, or if it is isolated to a specific state or country, I think it would be possible to buy enough time to tailor a response.

Source: I’m an army officer and – believe me – it’s a regular debate at the office :P

Mike_G
2018-02-14, 06:36 PM
1. Oh absolutely, but the absolute minimum pulp radius is quite small, a 1000lb bomb only has a radius of 5m. Now given fles is less tough than soil i'd say it's probably greater than that, and all the body parts are going to become shrapnel. My point was more you can't expect a kill radius of 50m on a 155 shell, that value was derived against sensibly organised troops. Against tight packing the sheer amount of mass in the way is going to really degrade it, anything within close rnage of the blast is toast, but when you'v got carpets of zombies covering multiple square miles it's going to take a LOT of heavy ordnance to get them all.


Like I said, prepare the field, channel the enemy into where you want them, and use things like fire. And You are vastly underestimating the damage a 155 mm shell can do.




2. I think most of the rest of what you have to say once again is just not getting your head around how outnumbered the military was. it wouldn't be your rifle company, (about 150-200 troops i believe from memory), vs a few thousand zombies. It would be your rifle company vs a half a million zombies spread across a 4 mile front stacked a third of a mile deep. Yeah you grenades and mortars and vehicle mounted weapons will tear them up, but how many can you kill before you start running out of ammo?



Plenty.

And when we think we're getting low, we hop in out Bradleys or LAVs or AAVs and drive slowly away, taking casual headshots as we go. I don't see how they can counter that. Even if they catch up, how does a zombie open a Bradley?

Seriously, we could make such a mound of corpses that just the climb would slow down the zombie horde.

If you think the military would just stand and fight until they ran out of ammo and then go down under the claws and teeth, you've watched too many movies.

Airstrikes would decimate the horde, then artillery, then the infantry would do their part, inflicting the most damage the could to a crowd tangled in wire and channeled by obstacles and probably on fire, then fall back in vehicles to a prepared position that the engineers have been working on while we held the line and the aircrews restocked and rearmed and slaughter another few thousand zombies ten miles down the road when we made our next stand.

Unless zombies are smart enough to not make frontal assaults, or learn to drive, or the human general is dumb as a box of rocks with the smart ones taken out, this is no contest.

Galloglaich
2018-02-14, 07:22 PM
If zombies are attracted to loud sounds, just dropping shells into a big wide open field somewhere, repeatedly,


In WW1 they had artillery barrages that went on for days, and that's with thousands of guns firing. Again, with relatively light guns especially, you can keep shooting and shooting and shooting. One mortar company with a 'good' supply could keep dropping shells every few minutes into the same area for a long time, maybe weeks. If the crowd got thick enough you could drop a lot more.

Just to give you an idea of the scale, in one battle in WW2 US forces fired 75,000 shells in a half an hour. For zombies, you could do say, 75 shells in half an hour, and stretch that out for 500 hours - 20 days. With Mortars by the way, the launcher doesn't even necessarily make that much noise and the range is more than you would expect 5000 meters for a standard 81mm mortar. You could put a few on top of a tall building and muffle the sound in various ways, and just devastate.

Anyone who thinks the zombies would prevail in a situation like that is underestimating military capabilities.

G

Mike_G
2018-02-14, 07:25 PM
And I think you could just leapfrog by company or battalion more or less forever.

The zombie horde moves at like 3 miles an hour over good terrain. So you set up a defensive line ten miles away and one unit fights there, with air, arty, etc, while ten miles further back a second unit is preparing a defensive position. When the zombies get close, or ammo gets low, the first unit retreats twenty miles to a third position where your rea untis have stockpiled supplies.

Lather, rinse, repeat.

Funneling the attack to expected areas is easy if they are mindless or respond to something we'll known like noise or smell, and if we have air or drones to keep an eye on the path of the zombie horde.

Carl
2018-02-14, 09:12 PM
Like I said, prepare the field, channel the enemy into where you want them, and use things like fire. And You are vastly underestimating the damage a 155 mm shell can do.




Plenty.

And when we think we're getting low, we hop in out Bradleys or LAVs or AAVs and drive slowly away, taking casual headshots as we go. I don't see how they can counter that. Even if they catch up, how does a zombie open a Bradley?

Seriously, we could make such a mound of corpses that just the climb would slow down the zombie horde.

If you think the military would just stand and fight until they ran out of ammo and then go down under the claws and teeth, you've watched too many movies.

Airstrikes would decimate the horde, then artillery, then the infantry would do their part, inflicting the most damage the could to a crowd tangled in wire and channeled by obstacles and probably on fire, then fall back in vehicles to a prepared position that the engineers have been working on while we held the line and the aircrews restocked and rearmed and slaughter another few thousand zombies ten miles down the road when we made our next stand.

Unless zombies are smart enough to not make frontal assaults, or learn to drive, or the human general is dumb as a box of rocks with the smart ones taken out, this is no contest.

1. In the book they mobility kill AFV's by getting in the way till the ground up bits causes something to break, (thrown track in the case of an abrams).

2. I'm not remotely suggesting that they stand there till the zombies get them as such. That's not really realistic unless it's an objective that has to be defended. Which was sotr of what the first battle of the book was about, the zombies had started moving out of NY trying to break quarantine and they had to be stopped before they dispersed into the countryside or they'd be even harder to deal with. Even then most got away apparently. They just couldn't stop the zombie horde with what they had.

3. All of that prep sounds nice, if you can do it. The point about he WWZ scenario is that they couldn't. Again going from memory of the past discussion the timeline starts with the initial disease in NY with no one knowing it was a zombie plague, they try and fail to contain it and are slow to realise it is a zombie plague. By the time they do they've got outbreaks in every city, NY is overrun and it's gone global. Martial law has been declared everywhere with troops backing up the police and more troops around every military stockpile and base to defend against rouge zombie groups. But NY seems contained, the zombies aren't leaving so they put a small quarantine force in place, (few thousand troops, a company of tanks), to maintain it against the odd small zombie group and pour the rest of their resources on trying to piss on all the bonfires they have to deal with, (and be fair, in a modern western military on home soil there's going to be a very strong tendency to not abandon anyone by the higher ups, they're US/UK/French/German/Whoever citizens). When the zombies suddenly decided en mass to move out of the city all the brass could commit was a few planes.

The discussion focused on that battle so my knowledge of what happened afterwards is very hazy, but from what i do know it sounds like roughly the same pattern kept repeating with a severely overstressed military, (with an equally overstressed logistics system), trying to be strong everywhere kept getting overrun in small groups because no one group had enough men and heavy ordnance to make it work. They basically suffered a severe case of defeat in detail. At that point they have few cities, no real manufacturing, (or oil refining), pipeline and millions of refugees to look after with tens to hundreds of thousands of swarms of zombies numbering anything from a few tens of thousands to a million or so spread all over the US and wandering in every direction.


If zombies are attracted to loud sounds, just dropping shells into a big wide open field somewhere, repeatedly,


In WW1 they had artillery barrages that went on for days, and that's with thousands of guns firing. Again, with relatively light guns especially, you can keep shooting and shooting and shooting. One mortar company with a 'good' supply could keep dropping shells every few minutes into the same area for a long time, maybe weeks. If the crowd got thick enough you could drop a lot more.

Just to give you an idea of the scale, in one battle in WW2 US forces fired 75,000 shells in a half an hour. For zombies, you could do say, 75 shells in half an hour, and stretch that out for 500 hours - 20 days. With Mortars by the way, the launcher doesn't even necessarily make that much noise and the range is more than you would expect 5000 meters for a standard 81mm mortar. You could put a few on top of a tall building and muffle the sound in various ways, and just devastate.

Anyone who thinks the zombies would prevail in a situation like that is underestimating military capabilities.

G

Sure they can do that. If that have enough guns and shells and can concentrate adequately, the US only has so many guns though, (and much of the rest of the world proportionally is even worse off), and the point about the WWZ scenario was that they basically couldn't/didn't concentrate in the early stages trying to hold onto every city and town and protect every part of the US at the same time. And i don't think thats unrealistic. However practical abandoning some area to focus on some area's first then going back and clearing the abandoned area's afterwards may be i just can't see anyone doing it without a lot of messes when citizens of their own country are involved.

Mike_G
2018-02-14, 10:25 PM
1. In the book they mobility kill AFV's by getting in the way till the ground up bits causes something to break, (thrown track in the case of an abrams).


...Not buying it.

First, I don't think there is a maximum number of bodies you can run over with an Abrams, and second, why would you ever let them get that close?

If zombies move at walking speed, and you have GUNS that work at range, just keep backing away while you mow them down. This pisses me off and I've seen movie soldiers do this all the time. Like the guy in the Humvee with the .50 cal that drives TOWARDS the Hulk while shooting. Why? The damn thing has a range of a mile. Drive AWAY from the guy who can only hurt you in melee.

If your mechanized force winds up in melee with a guy who can only shamble, you deserve to get your brains eaten, because you weren't using them.





2. I'm not remotely suggesting that they stand there till the zombies get them as such. That's not really realistic unless it's an objective that has to be defended. Which was sotr of what the first battle of the book was about, the zombies had started moving out of NY trying to break quarantine and they had to be stopped before they dispersed into the countryside or they'd be even harder to deal with. Even then most got away apparently. They just couldn't stop the zombie horde with what they had.


But they could have retreated and fought another day. No US unit has so little transport that it couldn't retreat from a horde of movie zombies.




3. All of that prep sounds nice, if you can do it. The point about he WWZ scenario is that they couldn't. Again going from memory of the past discussion the timeline starts with the initial disease in NY with no one knowing it was a zombie plague, they try and fail to contain it and are slow to realise it is a zombie plague. By the time they do they've got outbreaks in every city, NY is overrun and it's gone global. Martial law has been declared everywhere with troops backing up the police and more troops around every military stockpile and base to defend against rouge zombie groups. But NY seems contained, the zombies aren't leaving so they put a small quarantine force in place, (few thousand troops, a company of tanks), to maintain it against the odd small zombie group and pour the rest of their resources on trying to piss on all the bonfires they have to deal with, (and be fair, in a modern western military on home soil there's going to be a very strong tendency to not abandon anyone by the higher ups, they're US/UK/French/German/Whoever citizens). When the zombies suddenly decided en mass to move out of the city all the brass could commit was a few planes.



Again, prep should have been done if they were trying to quarantine the city. Especially with limited troops. This is the cheap and easy way to multiply your forces. That's War 101.

You build obstacles, a mindless mob follows the path of least resistance, which is into your klll zone, you kill loads of them then you fall back if you run out of ammo.

This is Zulu War level training. This is the easy stuff. The average US general trained to face Soviet armor coming through the Fulda Gap and infiltration by Taliban insurgents. They aren't going to get overrun by mindless shamblers. They've heard of Isandlwana and Little Bighorn. They know to take precautions.

And NYC is on the ocean, so there's no excuse not to have some naval resources nearby, like a carrier (of which we have loads. Can't believe NYC isn't worth one.)





The discussion focused on that battle so my knowledge of what happened afterwards is very hazy, but from what i do know it sounds like roughly the same pattern kept repeating with a severely overstressed military, (with an equally overstressed logistics system), trying to be strong everywhere kept getting overrun in small groups because no one group had enough men and heavy ordnance to make it work. They basically suffered a severe case of defeat in detail. At that point they have few cities, no real manufacturing, (or oil refining), pipeline and millions of refugees to look after with tens to hundreds of thousands of swarms of zombies numbering anything from a few tens of thousands to a million or so spread all over the US and wandering in every direction.


If the zombies has already spread out and were everywhere before the big battle, that's a plausible tough scenario. Because you can't defend everywhere.

But a concentrated zombie horde in NYC is far less difficult than that. Circumvalation with much better boom than the Romans had and dumber enemies and the ability to drive away at 60 mph.




Sure they can do that. If that have enough guns and shells and can concentrate adequately, the US only has so many guns though, (and much of the rest of the world proportionally is even worse off), and the point about the WWZ scenario was that they basically couldn't/didn't concentrate in the early stages trying to hold onto every city and town and protect every part of the US at the same time. And i don't think thats unrealistic. However practical abandoning some area to focus on some area's first then going back and clearing the abandoned area's afterwards may be i just can't see anyone doing it without a lot of messes when citizens of their own country are involved.

We have a lot of stockpiled weapons, especially if we don;t need the most modern stuff, but can drag out the old stuff that may not kill a Russian T-80 but will rip up zombies all day long.

Mr Beer
2018-02-14, 10:53 PM
First, I don't think there is a maximum number of bodies you can run over with an Abrams

Really a shame that Mythbusters has stopped making new shows.

Gnoman
2018-02-14, 11:23 PM
Unlike a living being, a robot does not go into shock when severely damaged. There's the possibility of using genetic algorithms to allow a robot to continue functioning (at decreasing effectiveness) as it is damaged. If you disable one weapons system, or one ammunition feed, or one turret motor, a well programmed combat robot might be able to continue fighting. Anti-material would definitely work. Do you think Armor Piercing Incendiary rounds would stay inside the robot long enough to cook the electronics? I'd be worried that they'd overpenetrate and shoot out the far side without spending enough time inside to transfer their heat.

API would almost certainly stay in (by the time it penetrated the initial armor, went through whatever components are there, and struck the other side, it would have shed way too much velocity to keep going), and it wouldn't take that long to transfer the heat.

Secondly, you're seriously overestimating the durability here. Get a hit on the battery? The battery bursts into flames and cooks the robot (remember a few years ago when one of the Samsung phones was banned from aircraft because the battery was spontaneously combusting? Same principle applies here). Strike an ammo feed? The ammuntion is likely to chain-fires and combust, cooking the robot. Strike a motor? Might just disable it, might cause an arc that cooks the robot.

Looking at your idea in more specific detail:



Think of a robot that's roughly the size of a car tire lying flat on the ground. It's got treads underneath, an articulated sensor "tower" that can be raised over obstructions, an array of cameras and sensors in the shell and an armored carbine-caliber turret on the top of the shell, and is deployed by the hundreds.

There's simply no possible way you can construct this and have it armored against any real firepower, by simple size. If we are looking at it with a near-future technology bias (which is critical, otherwise the weapons used against it would be better as well), a car tire is about the minimum possible size for a turret, depending on what you intend the meaningless phrase "carbine caliber" to denote. So, best case scenario, you have a car-tire turret stacked on top of a car-tire body.

Let us assume that the designers are not stupid, and go for a round turret on top of a square body instead of making both round. This allows for much more efficient use of internal space. For a power supply, let us assume an electric drive. You'll need at least one, probably two Lithium-ion battery packs roughly the size of a car battery to get any kind of endurance. So two battery packs. Some research suggests that an electric motor roughly equivalent to 2 horsepower is also about the size of a car battery. So you get two of those, one to run each track. Now, you need a brain, which will require a CPU, a PLC, some communications equipment, and some relays (the motors will draw too much power to switch directly - a burning PLC taught me that). Since it's going into combat, put this in a 5mm armored box with data ports on the bottom (where fire is least likely to come from). Charitably, you'll wind up with a box about the size of a car battery. The result would be five equal-sized boxes all in a row. Almost forgot - you'll need a sixth box, because all of these components are going to generate a lot of heat and simple ventilation won't cut it. That means you'll need to install a radiator block and run cooling lines. Put that on the back. Once you've run all your wiring, installed the tracks, and run the cooling lines, you've used up pretty much the entire "car tire" worth of space, and put in between 100 and 150 pounds worth of weight. Ignore the body armor for now.

Your turret needs to carry the weapon itself (as I alluded to earlier, "carbine" is a very sloppily defined word that could mean anything from a long-barreled gun firing pistol cartridges, to a P90-style PDW, or simply a short assault rifle - given the need to balance size and firepower, I think a P90-analog is a good pick for your proposed idea), a plentiful supply of ammunition (500 rounds minimum if you want any kind of sustained combat capability), rotating gear, and at least two high-quality cameras for sighting and observation purposes. You'll probably also want another water-cooling system, but that's probably optional here. You won't be able to pack this in as tightly as the hull, because you'll need easy access to resupply ammunition. Before armor, let's call this fifty pounds with the bare minimum framework to hold it together.

Now, let us turn to armor. Assuming a square box, 5mm of hardened steel will weigh about 16 pounds per face. The rounded turret is more complicated, but assume similar weight for napkin calculating. This means that putting 5mm of armor on all sides of both hull and turrent will add about 128 pounds of weight. Alternatively, you could use composites. Each "face" would have about the same surface area of a chestplate, so we can use that as a basis. A chestplate rated to stop a single rifle-caliber AP round (it will stop all lesser rounds, but multiple AP shots will shatter it and allow penetration) weighs 7.5 pounds. Round that up to 8, and you can armor it for about 64 pounds.

So that puts us at 328 pounds for the steel-armored version, and 264 pounds for the composite-armor one. This would be more than adequate protection from intermediate rifle cartridges (such as an M4 or AK), and will even keep out battle-rifle or machine gun fire as long as they don't have AP rounds handy. It would be a wise idea to reduce the rear armor to boost the frontal armor - that could bring you high enough to keep out rifle-caliber AP rounds from the front, at the cost of opening up the rear to just about anything - this is offset by putting the radiator in the rear, which would be very likely to convert a penetration from a hard kill (the thing is destroyed completely) to a repairable mission-kill (radiator destroyed, unit goes into preventive shutdown).

If, however, the machine was penetrated, it would be disabled or destroyed easily. With everything packed so tightly, the bullets would have no place to go except through very valuable and volatile components. With that in mind, I'd suggest that thinking of them as disposable would be ideal, and don't bother increasing the armor further. This is particularly sensible because you can't armor the sensors (which is critical for a remote-operated vehicle, and even more critical for an autonomous one) much.

Galloglaich
2018-02-15, 12:14 AM
Sure they can do that. If that have enough guns and shells and can concentrate adequately, the US only has so many guns though, (and much of the rest of the world proportionally is even worse off), and the point about the WWZ scenario was that they basically couldn't/didn't concentrate in the early stages trying to hold onto every city and town and protect every part of the US at the same time. And i don't think thats unrealistic. However practical abandoning some area to focus on some area's first then going back and clearing the abandoned area's afterwards may be i just can't see anyone doing it without a lot of messes when citizens of their own country are involved.

Lets try to remember, WW Z isn't history, it's just some kids notion of how he imagined the zompoc to go down - some kid who obviously never experienced military life and didn't know much about military history.

I don't know what you mean by "the US only has so many guns though" but every major metropolitan area in the United States is reasonably close to a big military base, at the very least a national guard unit, which would have sufficient firepower in terms of mortars or artillery to do what i described, not to mention enough armored vehicles to do what Mike G described.

https://lokisrevengeblog.files.wordpress.com/2017/06/mil_install_map_sm.gif?w=679&h=474

Even the 'minor' bases on this map would have more than enough ordinance to deal with what I was referring to.

Plus things like bulldozers to dig trenches or build up walls, and claymore mines and C4 to set up 'IED' type defenses and much more besides that.

I don't think y'all realize how many military assets there are in the US, and the rest of the world too for that matter. We don't usually think about it, but it's there in a heartbeat if ...stuff goes sideways.

I'm not even mentioning here the massive firepower deployed in our naval fleets by the way which, most of them being off shore at any given moment, would presumably be immune to the zombies and would provide an obvious and easy staging area to recover from any surprise.

Nasty surprises are what wars are all about, it's how they typically start. See Pearl Harbor, Operation Barbarossa, etc. etc. So that is what military units prepare for.

G

Carl
2018-02-15, 03:35 AM
...Not buying it.

First, I don't think there is a maximum number of bodies you can run over with an Abrams, and second, why would you ever let them get that close?

If zombies move at walking speed, and you have GUNS that work at range, just keep backing away while you mow them down. This pisses me off and I've seen movie soldiers do this all the time. Like the guy in the Humvee with the .50 cal that drives TOWARDS the Hulk while shooting. Why? The damn thing has a range of a mile. Drive AWAY from the guy who can only hurt you in melee.

If your mechanized force winds up in melee with a guy who can only shamble, you deserve to get your brains eaten, because you weren't using them.





But they could have retreated and fought another day. No US unit has so little transport that it couldn't retreat from a horde of movie zombies.



Again, prep should have been done if they were trying to quarantine the city. Especially with limited troops. This is the cheap and easy way to multiply your forces. That's War 101.

You build obstacles, a mindless mob follows the path of least resistance, which is into your klll zone, you kill loads of them then you fall back if you run out of ammo.

This is Zulu War level training. This is the easy stuff. The average US general trained to face Soviet armor coming through the Fulda Gap and infiltration by Taliban insurgents. They aren't going to get overrun by mindless shamblers. They've heard of Isandlwana and Little Bighorn. They know to take precautions.

And NYC is on the ocean, so there's no excuse not to have some naval resources nearby, like a carrier (of which we have loads. Can't believe NYC isn't worth one.)




If the zombies has already spread out and were everywhere before the big battle, that's a plausible tough scenario. Because you can't defend everywhere.

But a concentrated zombie horde in NYC is far less difficult than that. Circumvalation with much better boom than the Romans had and dumber enemies and the ability to drive away at 60 mph.




We have a lot of stockpiled weapons, especially if we don;t need the most modern stuff, but can drag out the old stuff that may not kill a Russian T-80 but will rip up zombies all day long.

1. Your telling me you can't get an abrams so mired in mud it will throw a track? Pulped flesh and cartilage should create roughly the same issues AFAIK. The human body is mostly water after all.

2. Again AFAIK they weren't totally overrun at any point except possibly when defending civilians, (in which case you can hardly complain about them standing their ground).

3. I don't know NYC geography but the battle is known as the battle of junkers if memory serves me right so maybe that helps. All i really know is that wherever it took place it was the last place before the hordes could start to spread way out and go every which way. It's also important to note that once again everyone underestimated the zombies. They'd shown no sign of moving out en mass for some time since the city fell, so apparently everyone just assumed they'd stay there the odd stragglers aside and that was all that the deployed troops were assigned to cope with, the mass movement flat footed them because no one expected it. Weather that's completely realistic i can't say and i'd defer to your judgment on.

4. My understanding is they weren't so much spread out as the disease was and everyone kept underestimating how fast spreading and virulent it would be, and just how difficult the zombies would be to deal with.

5.Yeah but where are they stockpiled, if most of it's stockpiled at a limited rnage of locations it may not be available on hand at a moments notice. What about the munitions, are they stockpiled with the weapons or elsewhere, if it;s old enough do you have anyone who even knows how to oporate or maintain it.


Really a shame that Mythbusters has stopped making new shows.

So true, i've often though wwz would provide dozens of good myths for them.


Lets try to remember, WW Z isn't history, it's just some kids notion of how he imagined the zompoc to go down - some kid who obviously never experienced military life and didn't know much about military history.

I don't know what you mean by "the US only has so many guns though" but every major metropolitan area in the United States is reasonably close to a big military base, at the very least a national guard unit, which would have sufficient firepower in terms of mortars or artillery to do what i described, not to mention enough armored vehicles to do what Mike G described.

https://lokisrevengeblog.files.wordpress.com/2017/06/mil_install_map_sm.gif?w=679&h=474

Even the 'minor' bases on this map would have more than enough ordinance to deal with what I was referring to.

Plus things like bulldozers to dig trenches or build up walls, and claymore mines and C4 to set up 'IED' type defenses and much more besides that.

I don't think y'all realize how many military assets there are in the US, and the rest of the world too for that matter. We don't usually think about it, but it's there in a heartbeat if ...stuff goes sideways.

I'm not even mentioning here the massive firepower deployed in our naval fleets by the way which, most of them being off shore at any given moment, would presumably be immune to the zombies and would provide an obvious and easy staging area to recover from any surprise.

Nasty surprises are what wars are all about, it's how they typically start. See Pearl Harbor, Operation Barbarossa, etc. etc. So that is what military units prepare for.

G

What i mean by limited guns is the US only has so many tube artillery, so many SPG, so many rocket systems e.t.c. Start putting those into their organizational units and you actually can't go out and say every base has enough, and thats assuming none have been frittered away in penny packets. That same penny packet problem could also impact your recon ability curtailing your maximum effective rnage.

And thats just looking at he USm, an attempted search turned up a lot more info o the UK army which apparently has 7 regiments equipped with 105's, 3 with mixed 155/MLRS and 1 with pure MLRS at 32 units a regiment, thats a significantly lower number per pop than the US, (numbers i dug up said 2k 155, 1k MLRS, and unknown on 105 or HIMARS), with just 350 systems, most of which are either small calibre 105's or slow reloading MLRS systems.

That said a military base allowed to fort up could probably hold off a zombie attack no problem. But in a zombie apocalypse, you, (and for that matter the troops if their locals), are going to want to protect all the civilians that didn't get zombified in the surrounding area, (and beyond), and then we once again come back to the penny packet problem. Any single force might be able to handle X zombies, or run away all day, but if the zombie drift carries a force bigger than X to them and they've got civilians or some other must hold objective to defend then you've got a real problem.

That said i never claimed the book was 100% acurratte, if there aren't issues i'd be amazed, there's no such thing as perfection. For example AFAIK the bomblets of an MLRS are airbursting by default whilst i believe the book treated them as ground contact detonating, which changes one scene i had brought up in the last discussion significantly.